12.07.2015 Views

Revista (format .pdf, 1.4 MB) - RECREAÅ¢II MATEMATICE

Revista (format .pdf, 1.4 MB) - RECREAÅ¢II MATEMATICE

Revista (format .pdf, 1.4 MB) - RECREAÅ¢II MATEMATICE

SHOW MORE
SHOW LESS
  • No tags were found...

Create successful ePaper yourself

Turn your PDF publications into a flip-book with our unique Google optimized e-Paper software.

Semnificaţia formulei de pe copertă:iπÎntr-o formă concisă, formula e = −1leagă cele patru ramuri fundamentaleale matematicii:ARITMETICA reprezentată de 1GEOMETRIAreprezentată de πALGEBRAreprezentată de iANALIZA MATEMATICĂreprezentată de eRedacţia revistei :Petru ASAFTEI, Dumitru BĂTINEŢU-GIURGIU (Bucureşti), Cornelia - Livia BEJAN,Temistocle BÎRSAN, Dan BRÂNZEI, Cătălin - Cristian BUDEANU, AlexandruCĂRĂUŞU, Constantin CHIRILĂ, Eugenia COHAL, Adrian CORDUNEANU, MihaiCRĂCIUN (Paşcani), Gabriel DOSPINESCU (student, Paris), Marius FARCAŞ, ParaschivaGALIA, Paul GEORGESCU, Mihai HAIVAS, Gheorghe IUREA, Lucian - GeorgesLĂDUNCĂ, Mircea LUPAN, Gabriel MÎRŞANU, Andrei NEDELCU, Gabriel POPA, DanPOPESCU (Suceava), Florin POPOVICI (Braşov), Maria RACU, Ioan SĂCĂLEANU(Hârlău), Ioan ŞERDEAN (Orăştie), Dan TIBA (Bucureşti), Adrian ZAHARIUC (Bacău),Adrian ZANOSCHI.Adresa redacţiei:Catedra de Matematică – Universitatea Tehnică “Gh. Asachi” IaşiBd. Carol I, nr.11, 700506, IaşiTel. 032 – 213737 / int. 123E-mail: recreatii.matematice@gmail.comhttp://www.recreatiimatematice.uv.roCOPYRIGHT © 2006, ASOCIAŢIA “RECREAŢII <strong>MATEMATICE</strong>”Toate drepturile aparţin Asociaţiei “Recraţii Matematice”. Reproducerea integrală sauparţială a textului sau a ilustraţiilor din această revistă este posibilă numai cu acordul prealabilscris al acesteia.TIPĂRITĂ LA SL&F IMPEX IAŞIBd. Carol I, nr. 3-5Tel. 0788 498933E-mail: simonaslf@yahoo.com


Mendel Haimovici şi Şcoala matematică dinIaşiŞcoala matematică din Iaşi, fondată în 1910în cadrul Seminarului Matematic de către savantulAl. Myller, s-a dezvoltat şi consolidat prin opera unormari personalităţi, ca Octav Mayer, Mendel Haimovicişi prin contribuţia unor străluciţi continuatori ai lor.Departamentul de mecanică al acestei Şcoli a fostcreat de profesorul Mendel Haimovici, membrualAcademiei Române. Calităţile sale de Om, Savant şiProfesor, luptător neobosit pentru dreptate, au trezitadmiraţia opiniei publice şi a colegilor, colaboratorilorşi studenţilor săi. Contribuţiasaladezvoltareaideilormatematice moderne este cu totul remarcabilă. Întreagasa activitate poartă marca personalităţii sale,caracterizată prin inteligenţă, curiozitate ştiinţifică,creativitate, vastă culturăşi etică profesională.M. Haimovici s-a născut la Iaşi, în 30 noiembrie 1906. A făcut studiile liceale laColegiul (Liceul) Naţional. Şi-a continuat pregătirea la Facultate de Ştiinţe (secţiaMatematică) pe care a absolvit-o în 1930. A fost imediat numit asistent şi a devenitcolaborator al foştilor săi profesori. A făcut stagii de specializare la Roma, Paris şiLondra. În iulie 1933 şi-a luat doctoratul în Italia, cu o valoroasă teză de mecanicafluidelor, sub îndrumarea lui Tullio Levi-Civita. Întorsînţară, şi-a reluat munca deasistent. În 1940, după promulgarea legilor rasiale, a fost îndepărtat câţiva ani dinUniversitate. La încetarea războiului a fost numit profesor şi a devenit şeful catedreide mecanică, pe care a condus-o până lasfârşitul vieţii, adică timpde28deani.În 1948, M. Haimovici a fost ales membru corespondent, iar câţiva ani mai târziua devenit membru titular al Academiei Române. Când s-a înfiinţat Filiala Iaşi a Academiei,a fost numit director al Institutului de Matematică, pe care l-a condus pânăla sfârşitul vieţii. A fost invitat să ia parte la numeroase conferinţe internaţionale şia organizat la Iaşi manifestări ştiinţifice de mare prestigiu. A încetat din viaţă la31martie 1973, lăsând o operă valoroasă şi o amintire luminoasă de remarcabil savant.Cercetările lui M. Haimovici se raportează la trei domenii principale ale matematicii:geometrie, teoria ecuaţiilor cu derivate paţiale şi mecanică. Cu toată mareainfluenţă exercitatădeMyller şi Mayer în domeniul geometriilor cu grup fundamental,a abordat un nou subiect din geometria acelui timp. După vizita lui Elie Cartanla Iaşi, în 1931, atras de ideile acestuia a eleborat importante lucrări asupra unorprobleme dificile. Astfel, a descoperit clase remarcabile de spaţii Finsler şi ecuaţiilefundamentale ale hipersuprafeţelor din aceste spaţii. Mai târziu, matematicianuljaponez Makoto Matsumo a inclus lucrările lui M. Haimovici în celebra sa monografieasupra fundamentelor geometriei Finsler. M. Haimovici a fost pionier şi în domeniulgeometriei integrale, descoperind o formulă care generalizează teorema cunoscută alui Crofton. A descoperit conceptul de quasigrup diferenţiabil şiaconstruitoteoriecompletă şi unitară; a demonstrat că acesteobiectesecomportăfaţă de grupurileLie în aceeaşi manieră în care varietăţileplatesecomportă în raport cu cele curbate.Actualmente, noţiunea de quasigrup este studiată din punct de vedere algebric şi89


topologic în şcoli renumite de matematică dinSUAşi Rusia.Olungă perioadă de timp, M. Haimovici s-a ocupat de geometria varietăţilorneolonome. Importanţa lor constă înfaptulcă ele conduc la cel mai adecvat modelcare descrie mişcarea sistemelor mecanice neolonome. Se poate spune că geometriaintrinsecă avarietăţilor neolonome în spaţii Riemann este creaţia lui M. Haimovici.În 1998, a apărut volumul I al operelor sale complete, iar volumul al doilea va apăreacu prilejul Centenarului naşterii savantului.Prin contribuţiile sale, M. Haimovici ascoslalumină noi domenii de cercetare,în care au lucrat şi lucrează continuatorii operei sale, printre care şi semnatarulacestei Note, care este totodată şi primul cercetător care a obţinut doctoratul subîndrumarea lui M. Haimovici.Profesorul M. Haimovici aavutvocaţiedegeometru. Pentruceicarenusuntfamiliarizaţi cu personalitatea sa, poate părea curios cum a putut el deveni şi unspecialist în mecanică. Pentru a lămuri acest aspect, se cuvine să facem un scurtistoric al predării mecanicii la Universitatea din Iaşi. La înfiinţarea acesteia, exista şiocatedră de mecanică. Primul curs "Mecanicăelementarăşi raţională"afostţinut deNicolae Culianu. Ulterior, această catedrăafostilustratădeIoan Melik şi MiltiadeTzony (absolvent de la Sorbona), care a publicat acum 125 de ani o culegere deprobleme de mecanică în revista "Recreaţii Ştiinţifice". Manuscrisul cursului ţinutde Tzony se păstrează şi astăzi la Biblioteca Seminarului Matematic. Tot lui i sedatoreazăşi înfiinţarea Laboratorului. Tzony rămâne unul dintre cei mai importanţioameni care au pus bazele unui învăţământ de înaltă valoare în ţara noastră. Acondus catedra de mecanică până în anul 1898. În 1907, Dimitrie Pompeiu, doctorîn matematică de la Sorbona, a preluat această catedră. El era un excelent analist,dar a fost şi primul în Iaşi, care a publicat lucrări originale de mecanică. Dupăcâţiva ani (cinci) a fost chemat la Bucureşti, pentru a lua postul rămas vacant prinretragerea lui Spiru Haret. Ulterior, catedra de mecanică afostilustrată succesiv deVictor Vâlcovici, Simion Sanielevici şi Ioan Plăcinţeanu.În 1945, M. Haimovici adevenitşefulcatedreidemecanică. Având vaste cunoştinţede geometrie, ecuaţii diferenţiale şi cu derivate parţiale, domenii în care a aduscontribuţii de valoare, a găsit acum oportunitatea de a materializa în mecanică osinteză a experienţei predecesorilor şi a sa personală. A ţinut cursuri de mecanică,hidrodinamică şi mecanica mediilor continue. A publicat un curs de elasticitateşi a organizat seminarii de specialitate şi conferinţe naţionale. A condus grupulde mecanicieni de la Institutul de Matematică. A condus teze de doctorat, avândsubiecte din teoria mediilor continue, hidrodinamică, magneto-hidrodinamică, elasticitateisotropică şi neisotropică în medii omogene şi neomogene, termoelasticitate,plasticitate, medii continue cu microstructură şi mecanică analitică.Această activitateintensă care a dus la crearea unei Şcoli originale de mecanicăeste, fără îndoială, rezultatul muncii de o viaţă a profesorului M. Haimovici. Dinnefericire, activitatea neîntreruptă, plină de abnegaţie şi permanentul său efort de asatisface toate exigenţele au contribuit la sfârşitul său prematur.Dăruireasapentruştiinţă este concretizată în cele două marirealizări ale sale:Şcoala de Mecanică dinIaşi şi Institutul de Matematică (care a renăscut după 1989).90Acad.Radu MIRON


adouazi. Cândsocoteacă dialogul nu are pe cine tulbura, împărtăşea şi altorabucuriile ei: iată ceideegrozavăaavutX,sauiată cefrumosospuneY.Osupăraustângăciile: cum poate Z să scrie despre asta fără a-l cita pe...? Pentru contrast, amvăzut frecvent mulţi care se încruntau când descopereau idei prin reviste: cum numi-a venit mie ideea asta?Mai cu seamă, Florica a înţeles şi iubit adolescenţii în strădaniile lor de a seapropia de matematică. Din dragoste faţă de ei a fost o excelentă profesoară întâila Liceul "Oltea Doamna", unde fusese şi eleva primei femei din România doctor înmatematică, Silvia Creangă. Mai apoi a fost conferenţiar şi profesor la Facultatea dematematică dinIaşi. Din aceeaşi dragoste a scris şi cunoscutele şi mult-cititile salecărţi. Alţii au conceput cărţi asemănătoare nu din dragoste faţă decititor,cispreazgândări glorii; să nemirăm că nu au avut ecou? Parcă spuneacinevacă dragosteaeste ca o minge pe care cu cât o arunci mai departe de tine, revine mai repede înapoi.Un mic dar gingaş, semn că ieşenii nu au uitat-o pe Florica, sunt cele cinci ediţii(judeţene şi interjudeţene) ale Concursului de matematică "FloricaT.Câmpan".Şirândurile de mai sus doresc a semnala adâncă şi neîntinată preţuire.Prof. dr. Dan BRÂNZEI92


Eclipsele de SoareSe spune că un corp ceresc fără luminăproprieesteeclipsat atunci când acestaintră în conul de umbră al unei planete, fiind astfel lipsit de lumina Soarelui şi nupoate fi văzut. Aşa se produc eclipsele de Lună sau eclipsele sateliţilor planetelor.Fenomenul de dispariţie a unui astru din câmpul nostru de vedere, când între acel astruşi observator se interpune unaltastru,senumeşte ocultaţie. Astfel,înmişcareasa în jurul Pământului, Luna poate să acopereparţial sau total Soarele. Fenomenulprodus este impropriu spus eclipsă deSoare, când de fapt este vorba de ocultaţiaSoarelui de către Lună. Trecând peste aceste mici neînţelegeri lingvistice să urmărimcum se produce o eclipsă de Soare.Dispariţia treptată a Soarelui din câmpul de vizibilitate are loc atunci cândSoarele, Luna şi Pământul sunt aproximativ în vecinătatea aceleiaşi drepte, cândau loc eclipse parţiale, totale sau inelare. Întrucât Luna se roteşte în jurul Pământuluide aproximativ 12,368 ori pe an, ar rezulta că înfiecarelună s-ar produce oeclipsă de Soare şi una de Lună. Totuşi, nu este aşa şi iată dece: Pământul seroteşte în jurul Soarelui pe o elipsă, Soarele aflându-se într-unul din focare (legea atreia a lui Kepler). Elipsa, curba plană descrisă dePământ, este foarte aproape deun cerc; distanţa minimă aPământului faţă deSoare(Pământul la periheliu) este de147 099 000 km, iar cea maximă (Pământul la afeliu) este de 152 097 090 km. Pe dealtă parte,Lunaseroteşte în jurul Pământului tot pe o elipsă, planul orbitei Lunii(planul elipsei) face, însă, cu planul orbitei terestre un unghi de 5 grade, 8 minute şi48 secunde, aşa că, nu la fiecare rotaţie cele trei obiecte pot fi coliniare.În fig.1, SA reprezintă raza Soarelui, egală cu 696 260 km, LB raza Lunii, egalăcu 1738 km, iar V un punct situat în vârful conului de umbră. Din calcule, rezultăcă înălţimea conului variază în limitele 363 200 km şi 375 900 km, în timp ce distanţade la Pământ la Lună rămâne cuprinsă între 363 100 km (perigeu) şi 405 500 km(apogeu). Rezultă deaicică, în cazul în care Luna se află ladistanţă maimarede375 900 km, vârful conului de umbră nu poate atinge nici un punct de pe suprafaţaPământului şi, în consecinţă, nu poate avea loc nici o eclipsă totală. Din contra,93


momentul primului contact exterior. Urmează acoperirea treptată a Soarelui pânăcând acesta nu se mai vede, momentul primului contact interior, după care Soarelestă în totalitate un timp, care nu poate fi mai mare de 8 minute. În continuarediscul Lunii devine tangent exterior, moment numit ultimul contact interior, dupăcare Soarele iese din totalitate. După momentul când cele două discuri devin dinnou tangente exterior, ultimul contact exterior, eclipsa ia sfârşit. Ansamblul tuturorpunctelor de pe suprafaţa Pământului pe unde trece pata de umbră senumeştebanda de totalitate, acărei lăţime maximă poate ajunge la 260 km, când Luna este laperigeu. În afara benzii de totalitate, unde eclipsa este parţială, cele două momenteinterioare lipsesc.Determinarea momentelor caracteristice, la care se adaugă un al cincilea, numitmomentul fazei maxime, oferăinformaţii importante pentru mecanica cerească.Acum circa 3000 de ani, caldeenii au notat datele la care observau fiecare eclipsăşi au constatat că eclipsele de Soare şi Lună sereproducîncondiţii identice într-uninterval de timp de 18 ani, 11 zile şi 8 ore. Această perioadă de repetare a eclipselora primit numele de ciclu Saros (saros în limba arabă înseamnă repetiţie). Într-uncicluSarosauloc71deeclipsedincare43deSoaresi28deLună. Ulterior, dupăîmbunătăţirea mijloacelor de observare şi dezvoltarea mecanicii cereşti, s-a constatatcă perioada ciclului Saros nu a fost determinată cu suficinentă precizie ceea ce acondus la abateri mari în calcularea datei celei mai mari sărbători a creştinătăţii,sărbătoarea Paştelui. Caexemplu,înanulacestacreştinii catolici au sărbătoritPaştele la data de 16 aprilie, iar ortodocşii la 23 aprilie. Diferenţele dintre cele douădate pot depăşi uneori şi o perioadă sinodicăaLunii(perioada sinodică reprezintătimpul necesar Lunii ca sătreacăprindouă faze succesive de acelaşi fel şi are o duratăde29zile,12ore,44minuteşi 3 secunde).În ziua de 29 martie a acestui an a avut loc o eclipsă totalădeSoare, vizibilăla Iaşi ca eclipsăparţială, pentru care momentele caracteristice calculate (pentru Iaşi)sunt: începutul eclipsei parţiale - 12h 50m 53s, faza maximă - 14h 02m 02s, sfârşituleclipsei parţiale —15h 12m 01s, iar acoperirea maximă a discului solar a fost de 73,3%.Chiar şi acum observarea eclipselor prezintă interesştiinţific pentru o cunoaşteremai bună a drumului parcurs de Pământ în jurul Soarelui precum şi pentru studiulunor fenomene solare ce nu pot fi observate decât în timpul eclipselor. Toate observaţiilefăcute cu mare rigurozitate sunt binevenite.Iulian BREAHNĂMembru al Uniunii Astronomice InternaţionaleEx-director al Observatorului Astronomic din Iaşi95


OLIMPIADA 57Cel care însăilă aceste rânduri socoteşte (de multişor) că olimpiadele de matematicăsuntevenimentemajore,plinedesemnificaţii pentru multă lume: pentru tineriieroi care îşi încearcă puterile în munci mai relevante decât cele ale lui Hercules, pentru(veşnic tinerii) care şlefuiesc cu migală subiecte sau evaluează redactări, pentru inimoşiicare asigură detalii organizatorice, pentru sponsorii care au avut înţelepciune săplaseze bine bănuţi trebuitori. Lista de mai sus a fost nevoită săsarăpesteforţe carepregătesc bune prestaţii ale elevilor în şcoalăşi acasă: profesori (în general) care respectăeforturi, profesori de matematică aprinzători de scântei, părinţi care vegheazăiubitori, editori care adună strădanii trecute spre a fi de folos celor care urmează.Să mai spunem că apelativul de olimpiadă s-a conturat prin anii ’50, mai ales prinstrădanii ale neasemuitului Grigore C. Moisil. (În 2006 s-a evocat în lumea largăcentenarul naşterii sale.) Exista buna experienţă a concursurilor gazetei matematicecu suflet, inimă şi minte donate de Gheorghe Ţiţeica. Dar a existat şi vântpotrivnic dinspre răsărit care ar fi preferat un nume de spartakiadă. Instituţia carea asigurat geneza acestor olimpiade a fost Societatea de Ştiinţe Matematice şi Fizice.Este benefic să neminunăm de vigoarea cu care au crescut şi s-au maturizatolimpiadele. Erau 50, apoi o sută, apoi două definalişti; acum ar veni (eventual pecheltuială proprie) cam 10 mii de finalişti; spre a preveni auto-sufocări s-a conturatprin anul 2000 algoritm prin care să nusetreacă de 600 de participanţi.Buna experienţă a olimpiadelor naţionale a îndreptăţit România săiniţieze OlimpiadeInternaţionale, apoiOlimpiade Balcanice. Acumfinala naţională nu mai apare caultim scop; urmează îndată prime baraje, calificări la loturi (de seniori şi de juniori),alte baraje, calificări în echipe naţionale, variate întreceri internaţionale.Dulcele Târg al Ieşilor se întâlnise cu Olimpiada Naţională de Matematică în 1975.(A fost atunci creditat la treabăşi semnatarul acestor rânduri.) Au mai existat cereride reîntâlnire; ce susţineau unii, boicotau alţii. Iaşul se afirmase prin anii ’90, în specialprin trio-ul de elevi Turinici—Gheba—Bîrsan, iar prin ’99 prin multiplu campionMarius Beceanu. Dintre vechi olimpici (dar nu numai) fuseseră invitaţi la ComisiiNaţionale distinşi profesori ieşeni. Există o familie olimpică ieşeană; cităm aici doarpe ultimul: campionul Pachi. Loturi naţionale se pregătiseră înBucium. Intraserăîn calendar naţional şi Tabere organizate de ieşeni. Se aprobase şi se desfăşurase cusucces Concursul Naţional "Al. Myller". Factori diriguitori susţinuseră OlimpiadăNaţională de Matematică laIaşi în 2005. A fost aprobare, dar s-a acordat prioritateOlimpiadei Balcanice; se ştia că Olimpiada Naţională din 2006 va fi la Iaşi.Afostşi a ieşit bine. Climatul intelectual al Iaşului a constituit un fond favorabil.Profesori dotaţi şi inimoşi au contribuit cu capacitate şi dăruire. Colegiile Negruzzişi Naţional au colaborat excelent ca gazde. Facultatea de matematică aasiguratgiracademic; preşedinte de onoare a fost academicianul Radu Miron. Aufostcreditaţica propunători sau evaluatori cam 15 profesori ieşeni. A funcţionat impecabil unsecretariat ce a însumat cam 30 de profesori destoinici. Elevii ieşeni au materializatceva din avantajul terenului, cifra pentru olimpiada naţională crescânddela16la20.Încă nu putem face un bilanţ alcomportărilor internaţionale, dar suntem optimişti.96Prof. dr.Dan BRÂNZEI


Pseudoinversă şi inversă generalizatăale unei aplicaţii liniareAdrian REISNER 11. Pseudoinversă a unui endomorfism într-un spaţiu vectorial de dimensiunefinită. Fie S un R-spaţiu vectorial de dimensiune finită. Are locTeorema 1. Fiind date două endomorfisme u, v ale spaţiului S, dacădouădincele trei condiţii următoare:a) uvu = u, b) vuv = v, c) rangu =rangvsunt verificate, atunci a treia este de asemenea verificată.Demonstraţie. a) şi b) ⇒ c) Avem: rang u =rang(uvu) ≤ rang(uv) ≤ rang vdin condiţia a), rang v =rang(vuv) ≤ rang(vu) ≤ rang u din condiţia b). Deducemcondiţia c).a) şi c) ⇒ b) Condiţia a) conduce, înmulţind la dreapta cu v, la egalitateauv =(uvu)v = u(vuv).(∗)Ţinând seama de a), avem, pe de altă parte, că rang u ≤ rang(uv) şi, fiindcărang(uv) ≤ rang u, deducem egalitatea rang(uv) =rangu. Dar rangul endomorfismuluiuv este rangul restricţiei endomorfismului u la subspaţiul Im v. Această restricţieu / Im v este deci injectivăşi egalitatea (∗) poate fi simplificată cuu la stânga,ceea ce închide demonstraţia.b) şi c) ⇒ a) rezultă imediat, endomorfismele u şi v având roluri simetrice în relaţiilea), b), c). Teorema este demonstrată.Corolar. Fiind dat un endomorfism u al spaţiului vectorial S, existăunendomorfismv (nu unic) verificând condiţiile a), b) şi c).Demonstraţie. Ţinând seama de teoremă, ne propunem să construimendomorfismulv verificând condiţiile b) şi c). Fie v un astfel de endomorfism. Avempentru ∀y ∈ Im v : vu(y) =y. Deducem că, dacă x ∈ Im v ∩ Ker u, x =0şi deciIm v ∩ Ker u = {0}. Suma acestor două spaţii vectoriale este deci directă . Folosindcondiţia rang u =rangv, deducem atunci că Im v ⊕ Ker u = S.Fiind dat endomorfismul u, fieF un suplementar al subspaţiului vectorial Ker uşi G un suplementar al spaţiului Im u. Aplicaţia u 0 : F → Im u, x 7→ u(x) este unizomorfism. Definim aplicaţia v prin restricţiilesalelaceledouăsubspaţii suplementareIm u şi G prin:• v / Im u =(u 0 ) −1 ,• v /G =0.Această aplicaţie v este evident liniară şi verifică:− Im v =Im(u 0 ) −1 = F , deci rang v =dimF =rangu şi rezultă că v verifică c);−∀x ∈ S, v(x) ∈ F , deci vuv(x) =v {u 0 [v(x)]} =(vu 0 )[v(x)] = v(x), i.e. vverifică vuv = v, adică b).Corolarul este astfel stabilit.1 Cercetător, Centrul de Calcul E.N.S.T., Paris97


Observaţie. Endomorfismul v nu este unic: el depinde de alegerea subspaţiilorF şi G din demonstraţia corolarului.Definiţii. Fiind dat endomorfismul u, orice endomorfism verificând condiţia a) senumeşte inversă generalizată aluiu. Fiind dat endomorfismul u, orice endomorfismverificând condiţiile a), b) se numeşte pseudoinversă aluiu.Cu aceleaşi notaţii ca în corolarul precedent, o inversă generalizată v a endomorfismuluiu este definită prinrestricţiile sale la subspaţiile suplementare Im u şi Gprin:• v / Im u =(u 0 ) −1 ,• v /G morfism oarecare aparţinând spaţiului L(G, S).Oinversă generalizată a endomorfismului u depinde deci de alegerea subspaţiilor F ,G şi de alegerea morfismului v /G .VomnumiIng(u) ={v ∈ L(S) |uvu = u}.Observaţii. 1) Inversa generalizată aunuiautomorfism u este unică şi coincidecu inversa u −1 aluiu :Ing(u) = © u −1ª .2) Fie v ∈ Ing(u). Avem:endomorfismul w = uv este un proiector (w 2 = w) de imagine Im u:w 2 =(uv)(uv) =(uvu)v = uv = w,endomorfismul w 0 = vu este un proiector (w 02 = w 0 )denucleuKer u:w 02 =(vu)(vu) =v(uvu) =vu = w 0 .Aplicaţie. Vom nota cu aceeaşi literă un endomorfism al spaţiului vectorialR n şi matricea acestui endomorfism în baza canonică aluiR n . Fiind date treiendomorfisme A, B, C ale spaţiului vectorial R n ,fieà ∈ Ing(A) şi ˜C ∈ Ing(C). ArelocTeorema 2. Ecuaţia matriceală AXC = B admite o soluţie X dacă şi numaidacă avemAÃB ˜CC = B.Demonstraţie. Presupunând că AÃB ˜CC = B, deducem că matriceaX = ÃB ˜Cverifică AXC = B. VomnotaX 0 = ÃB ˜C.Invers, dacăexistă X ∈ L(R n ) verificând AXC = B, atunci avem (AÃA)X(C ˜CC)=B şi, în consecinţă, B = AÃ(AXC) ˜CC = AÃB ˜CC.à şi ˜C fiind două matrice oarecare aparţinând respectiv la Ing(A) şi Ing(C),notămnK(A, C) = Y − ÃAY C ˜Co| Y ∈ M n (R) .Teorema 3. Soluţia generală aecuaţiei matriceale AXC = B este X = ÃB ˜C +Z, undeZ ∈ K(A, C).Demonstraţie. Soluţia generală a ecuaţiei matriceale AXC = B este X =X 0 + Z, unde X 0 = ÃB ˜C (v. demonstraţia Teoremei 2) şi Z este soluţia generală aecuaţiei AZC = O. Ne propunem să justificăm echivalenţa următoare:a) AZC = O ⇔ b) Z ∈ K(A, C).a) ⇒ b) Z = Z − O = Z − Ã(AZC) ˜C;98


) ⇒ a) ∀Y ∈ M n (R), matricea Z = Y − ÃAY C ˜C verificăAZC = AY C − AÃAY C ˜CC = AY C − (AÃA)Y (C ˜CC)=Oşi Teorema 3 este demonstrată.Exemplu. Fie ecuaţia matriceală⎛⎝ 0 1 0 ⎞ ⎛−1 0 0⎠ · X · ⎝ 1 0 0 ⎞ ⎛0 0 0⎠ = ⎝ k 0 0 ⎞0 0 0⎠ , unde k ∈ R. (1)0 0 0 0 0 0 0 0 0Să găsim soluţia ⎛generală X ∈ M 3 (R) a acestei ecuaţii.Avem: A = ⎝ 0 1 0 ⎞ ⎛−1 0 0⎠ şi à = ⎝ 0 −1 0 ⎞⎛1 0 0⎠ ∈ Ing(A); C = ⎝ 1 0 0 ⎞0 0 0⎠0 0 0⎛0 0 00 0 0şi ˜C = C ∈ Ing(C). Cum B = ⎝ k 0 0 ⎞0 0 0⎠ = AÃB ˜CC, ecuaţia (1) admite soluţii,0 0⎛0osoluţie fiind matricea X = ÃB ˜C = ⎝ 0 0 0 ⎞k 0 0⎠. Ţinând seama de Teorema 3,0 0 0soluţia generală aecuaţiei (1) este⎛X = ÃB ˜C + Y − ÃAY C ˜C = ⎝ 0 0 0 ⎞k 0 0⎠ + Y − ÃAY C ˜C,0 0 0unde Y este o matrice oarecare aparţinând spaţiului de matrice M 3 (R).⎛ Considerând a, b, c, d, e, f, g, h, i numere reale oarecare, obţinem, cu Y =⎝ a b c ⎞⎛d e f⎠, căsoluţia generală aecuaţiei matriceale (1) este X = ⎝ 0 b c ⎞k e f⎠,g h ig h iunde b, c, e, f, g, h, i sunt numere reale oarecare.2. Pseudoinversa unui endomorfism într-un spaţiu euclidian. E fiindun spaţiu euclidian, produsul scalar va fi notat h, i. [Aceleaşi definiţii şi rezultatesunt valabile dacă spaţiul E este un spaţiu hermitian, adică unC -spaţiu vectorialînzestrat cu un produs scalar, formă hermitianăpozitivdefinităpeE.] Amintimdefiniţia şi teorema următoare:Definiţie şi teoremă. Fiind dat un endomorfism al spaţiului euclidian E, existăun singur endomorfism u ∗ al spaţiului E verificând∀x, y ∈ E, hu(x),yi = hx, u ∗ (y)i .Acest endomorfism se numeşte adjunctul endomorfismului u.Avem Ker u ∗ =(Imu) ⊥ şi Im u ∗ =(Keru) ⊥ .Demonstrăm că Ker u ∗ =(Imu) ⊥ ;într-adevăr,y ∈ (Im u) ⊥ ⇔ ∀x ∈ E, hu(x),yi =0 ⇔ ∀x ∈ E, hx, u ∗ (y)i =0 ⇔ y ∈ Ker u ∗ .Asemănător procedăm pentru egalitatea Im u ∗ =(Keru) ⊥ .99


Fie u un endomorfism al spaţiului euclidian E .Notăm K =(Keru) ⊥ ortogonalulnucleului lui u şi I =(Imu) ⊥ ortogonalul imaginii endomorfismului u.Notăm P proiecţia ortogonală de imagine K (de nucleu Ker u), P 0 proiecţiaortogonală de imagine Im u (de nucleu I = (Imu) ⊥ ). Avem: P 0 u = u = uP ;P 0 (y) ∈ Im u, ∀y ∈ E; u (−1) P 0 (y) (imaginea reciprocă a elementului P 0 (y)) esteoclasă modulo Ker u aspaţiului euclidian E (atenţie: u (−1) nu este o aplicaţie!).Imaginea acestei clase prin proiecţia P este un element unic al spaţiului K.Definiţie. Aplicaţia liniară u + = Pu (−1) P 0 se numeşte pseudoinversă aendomorfismuluiu.Teorema 4. Aplicaţia pseudoinversă u + a endomorfismului u verifică relaţiile:a) uu + u = u,b) u + uu + = u + ,c) uu + şi u + u sunt endomorfisme ortogonale, i.e. (uu + ) ∗ = uu + , (u + u) ∗ = u + u.Mai mult, u + este singurul endomorfism al spaţiului E verificând aserţiunile a), b)şi c).Demonstraţie. P şi P 0 fiind proiectori, avem Pu + = u + P 0 = u + . Atunci:u + u = Pu (−1) P 0 u = Pu (−1) u = P, proiector ortogonal,uu + = uP u (−1) P 0 = uu (−1) P 0 = P 0 , proiector ortogonal,şi, în consecinţă, uu + u = uP = u; u + uu + = u + P 0 = u + .Înplus,Im u + =ImP = K =(Keru) ⊥ =Imu ∗ , Ker u + =KerP 0 = I =(Imu) ⊥ =Keru ∗ .Demonstrăm acum unicitatea endomorfismului u + . Fie u 0 un alt endomorfismverificând condiţiile a), b) şi c). Avem:u 0 = u 0 uu 0 = u 0 u 0∗ u ∗ = u 0 u 0∗ (u ∗ u +∗ u ∗ )=u 0 (uu 0 )(uu + )=u 0 uu + ;u 0 uu + = u ∗ u 0∗ u + =(u ∗ u +∗ u ∗ )u 0∗ u + =(u + u)(u 0 u)u + = u + uu + = u + .Deci u 0 = u + , c.c.t.d.Observaţie. Am văzut în prima parte că opseudoinversăaluiu depinde înparticular de alegerea subspaţiilor suplementare F şi G ale spaţiilor Ker u şi respectivIm u. În cazul în care E este un spaţiu euclidian F şi G sunt unic definite prinF =(Keru) ⊥ şi G =(Imu) ⊥ , de unde unicitatea pseudoinversei.Dacă y este un vector oarecare al spaţiului euclidian E, notăm x 0 = u + (y). AvemTeorema 5. Vectorul x 0 = u + (y) verifică:a) kux 0 − ykeste minimum, i.e. kux 0 − yk ≤ kux − yk, ∀x ∈ E;b) kx 0 k este minim din toate elementele x verificând kux − yk = kux 0 − yk.°Demonstraţie. a) kux 0 − yk = °uu + y − y° = kP 0 y − yk,kux − yk 2 = ku(x − x 0 )+ux 0 − yk 2 = ku(x − x 0 )+P 0 y − yk 2 == ku(x − x 0 )k 2 + kP 0 y − yk 2 ≥ kux 0 − yk 2 .b) Avem kux − yk = kux 0 − yk numai când u(x − x 0 )=0. În cazul acesta,x = x 0 + z, unde x 0 ∈ K =(Keru) ⊥ şi z ∈ Ker u. Deci kxk 2 = kx 0 k 2 + kzk 2 ≥ kx 0 k 2şi deducem b).100


Observaţie. Din teorema precedentă deducem că x 0 = u + (y) este cea mai bunăsoluţie apropiată înnormă a ecuaţiei y = ux.Pseudoinversa unei matrice. Dacă alegem în spaţiul euclidian E o bazăortonormată B,obţinem U-matricea endomorfismului u în aceastăbază: Mat(u ∗ ,B)=U ∗ = U T .[În cazul în care E este hermitian avem Mat(u ∗ , B) =U ∗ = U T .]Traducând cele de mai sus în limbaj matriceal, obţinemFiind dată o matrice U ∈ M n (R), existăosingurămatriceV verificând condiţiileurmătoare:UVU = U, V UV = V, (UV) ∗ = UV, (VU) ∗ = VU.Această unică matrice V se numeşte pseudoinversa matricei U şi se notează U + .X 0 = U + Y este cea mai bună aproximaţie cuadratică aecuaţiei UX = Y .Exemplu de calcul al pseudoinversei unei matrice. Fie matricea⎛U = ⎝ 1 0 1 ⎞0 1 0⎠ ∈ M 3 (R).1 0 1Să calculăm matricea U + .Matricea U fiind simetrică, avem Ker u =(Imu) ⊥ .Fie B = {e 1 ,e 2 ,e 3 } baza canonică aspaţiului R 3 înzestrat cu produsul scalarcanonic. Pentru endomorfismul u asociat matricei U în această bază, avem:Im u =Vect{e 2 ,e 1 + e 3 } , Ker u = R(e 1 − e 3 ).Endomorfismul v de matrice U + în baza canonică B verifică atunciKer v =(Imu) ⊥ =Keru; Imv =(Keru) ⊥ =Imu.Pe de altă parte:uvu(e 2 )=u(e 2 ) ⇒ uv(e 2 )=u(e 2 ) ⇒ v(e 2 )=e 2 + λ(e 1 − e 3 ); v(e 2 ) ∈ Im v =Im u ⇒ λ =0,deciv(e 2 )=e 2 ;uvu(e 1 )=u(e 1 ) ⇒ uv(e 1 + e 3 )=e 1 + e 3 = u( e 1 + e 3) ⇒ v(e 1 + e 3 )= e 1 + e 3+22µ (e 1 − e 3 ); v(e 1 + e 3 ) ∈ Im u ⇒ µ =0,deciv(e 1 + e 3 )= e 1 + e 3. Dat fiind că2v(e 1 − e 3 )=0[Ker v =Keru], deducem că v(e 1 )=v(e 3 )= e 1 + e 3.4Finalmente matricea U + este⎛U + = 1 ⎝ 1 0 1 ⎞0 4 0⎠ .41 0 1101


Osoluţie parţială a unei probleme a lui N. PapacuCătălin ŢIGĂERU 1Profesorul Nicolae Papacu analizează în[1] recurenţa pătratică, subliniind faptulcă naturaşirului este dictată dealegereatermenuluiiniţial şi lasă caproblemădeschisă cazul (III.3.d). Ne propunem să dăm un răspuns parţial acestei probleme.I. O problemă deschisăşi câteva rezultate generale. Recurenţax n+1 = ax 2 n + bx n + c, x 0 ∈ R, a 6= 0,se reduce, cu substituţia y n = ax n + b 2 , la recurenţa y n+1 = yn 2 + α, unde α =12 (2b − ∆), cu∆ = b2 − 4ac. Situaţia propusă caproblemădeschisă, notată (III.3.d)în articolul citat, se referă lacazulîncareα


N ∗ ,considerăm mulţimile M0n = © x ∈ [−L 2 ,L 2 ] | f [n] (x) =0 ª Sşi fie M 0 = ∞ M0 n .n=1Să notăm faptul că, pentru fiecare n ∈ N ∗ ,avem0 ∈ M0 2n2n−1, {±1} ⊂ M0 . Maimult, dacă β ∈ M 0 , atunci avem şi incluziunea © x ∈ [−L 2 ,L 2 ] | f [n] (x) =β ª ⊂ M 0 .Teorema 2. Dacă δ =1,atuncişirul½(y n ) n∈Neste convergent dacă şi numaidacă y 0 ∈ [−L 2 ,L 2 ] \ M 0 şi lim y L1 ; yn =0 ∈ (−L 2 ,L 2 ) \ M 0 ,n→∞ L 2 ; y 0 ∈ {−L 2 ,L 2 } .Dacă y 0 ∈ M 0 , atunci (y n ) n∈Nnu are limită şi există n 1 ∈ N ∗ astfel încâty n = 1 2 ((−1)n − 1), ∀n ≥ n 1 .Demonstraţie. Se demonstrează prin inducţie că, dacă y 0 ∈ (−1, 1), atunci y n ∈(−1, 0), ∀n ≥ 1. Mai departe, judecând ca în Teorema 1, deducem că lim y n = L 1 .n→∞Să presupunem că y 0 ∈ (−L 2 , −1) ∪ (1,L 2 ) \ M 0 ;conformLemei1,există n 0 ≥ 2astfel ca y n0 < 0. Deoarece y 0 /∈ M 0 ,rezultăcă f [n0] (y 0 ) 6= 0, deci y n0 +1 6= 0,deunde −1 0, ∀n ∈ N, deducem, pe baza Lemei 1, că ȳ 0 = L 2 ,adică y n0 = L 2 ; dar aceasta conduce la y 0 ∈ M L2 , fals. Presupunem prin absurdcă există n 0 ∈ N ∗ astfel ca y n < 0, ∀n ≥ n 0 . Luând în considerare şirul (ȳ n ) n∈Ndefinit de (∗∗), deducem că ȳ n = y n+n0 < 0, ∀n ∈ N, ceea ce, în virtutea Lemei 2,conduce la ȳ 0 = L 1 ,adică y n0 = L 1 , aceasta însemnând că y 0 ∈ M L1 ,falsdinnou.S-a demonstrat astfel că, dacă y 0 ∈ [−L 2 ,L 2 ] \{M L1 ∪ M L2 },atuncişirul (y n ) n∈Neste mărginit, nu are limită şi conţineoinfinitatedetermenipozitivişi o infinitatede termeni negativi. Dacă y 0 ∈ M L1 ∪ M L2 , deducem că lim n ∈ {L 1 ,L 2 }.n→∞Propoziţia2semnaleazăfaptulcă, în acest caz, există şiruri periodice sau, maigeneral, şiruri cu mulţimea valorilor finită. Demonstrăm în continuare că putem alege105n=1n=1


y 0 astfel încât mulţimea valorilor şirului să fie infinită. Pentru aceasta, considerămaplicaţia A : Ñ → [X], unde R [X] este inelul polinoamelor cu coeficienţi reali şiÑ = N × N \{(n, n) | n ∈ N}, definită prinA (n 1 ,n 2 )=f [n1] (X) − f [n2] (X) ∈ R [X].Considerăm mulţimea S care are ca elemente toate rădăcinile din intervalul [−L 2 ,L 2 ]ale polinoamelor ce aparţin imaginii funcţiei A. Cum S este numărabilă, alegemy 0 = b ∈ [−L 2 ,L 2 ] \ S; demonstrăm că funcţia Y : N → [−L 2 ,L 2 ], Y (n) =y n esteinjectivă: în adevăr, dacă arexistan 1 ,n 2 ∈ N, n 1 6= n 2 ,astfelcaY (n 1 )=Y (n 2 ),atunci am avea f [n1] (b) =f [n2] (b), ceea ce ar însemna că polinomul g ∈ A(Ñ),g (X) =f [n1] (X) − f [n2] (X), arerădăcina b ∈ S, ceea ce este fals. Q.e.d.III. Completări şi comentarii finale. 1. Propoziţia 2 asigură, pentru 1 ≤ δ,existenţa şirurilor periodice, de perioadă 2. Fără a intra în detalii, precizăm doar căse pot construi şiruri care au perioade mai mari decât 2.2. Pentru δ = √ 2 se cunoaşte forma analitică a termenului general, aceastăsituaţie făcând obiectul mai multor probleme de concurs (vezi [1], [3] ).3. În cazul δ>1, putem constata: (a) dacă y 0 ∈ M L1 ∪M L2 ,atunci lim y n = L 1n→∞dacă y 0 ∈ M L1 şi = L 2 dacă y 0 ∈ M L2 .(b) Conform propoziţiei 2, există şiruri periodice sau şiruri periodice de la un locîncolo, deci şiruri mărginite, cu un număr finit de valori.(c) Există posibilitatea de a alege y 0 ∈ [−L 2 ,L 2 ] astfel încât şirul (y n ) n∈Nsăaibălimita egală cu∞; de exemplu, dacă estey 0 =0, atunci y 1 = −δ 2 , y 2 = δ 4 − δ 2 >L 2 ,de unde lim y n = ∞. Ceea ce nu putem preciza este dacă există posibilitatea den→∞aalegey 0 ∈ [−L 2 ,L 2 ],astfelîncâtşirul să fiemărginit şi mulţimea valorilor şiruluiinfinită. În legătură cu acest subiect propunem · conjectura:Şirul (y n ) n∈N, definit de recurenţa y 0 ∈ − 1 ³1+p1+4δ 2´, 1 22y n+1 = yn 2 − δ 2 , δ> √ 2,estemărginit dacă şi numai dacă esteperiodicdelaunlocîncolo.³1+p1+4δ 2´¸,4. Sintetizăm concluziile articolului în următorul tabel:Valorile lui δ Tipul de şiruriδ ∈ (0, 1) ; (i) , (ii);δ =1;(i) , (ii) , (iii);δ ∈ ¡ 1, √ 2 ¤ ; (ii) , (iii) , (iv);δ ∈ ¡√ 2, ∞ ¢ ; (ii) , (iii) , (v) , (iv−?) .5. Dl prof. Gheorghe Marchitan a citit cu atenţie materialul, îmbunătăţindu-lprin observaţii. Acelaşi efect benefic l-au avut şi discuţiile purtate cu dl prof. MariusMarchitan. Amândurora, autorul le mulţumeşte şi pe această cale.Bibliografie1. N. Papacu - Asupra unui şir recurent, Arhimede, 5-6/2000, 1-4.2. Gh. Oprişan - On a class of real sequences defined by recurence, Gazeta Matematică(seria informare ştiinţifică şi perfecţionare metodică), XIV(XCIII), 2/1996, 82-90.3. C. Ţigăeru - Recurenţa a n =2a 2 n−1 − 1 şi problema XI.3 a Concursului Revistei"Sinus", Sinus, II.1(4)/2006.106


Criterii de congruenţă a triunghiurilorMarius APETRII 1 şi Cristian-Cătălin BUDEANU 2În RecMat - 2/2005, elevilor de clasa a VII-a le este propusăurmătoarea problemă:VII.63. Fie triunghiurile ABC şi A 0 B 0 C 0 cu AB = A 0 B 0 , AC = A 0 C 0 şi bisectoarele[AD], [A 0 D 0 ] congruente. Să searatecă 4ABC ≡ 4A 0 B 0 C 0 (enunţ adaptat).Petru Asaftei, IaşiÎn cele ce urmează, dorim să stabilim condiţii suficiente pentru congruenţa a douatriunghiuri care au două bisectoare congruente. Vom considera mereu îndeplinităipotezaA(I) Triunghiurile ABC şi A 0 B 0 C 0 au bisectoarele [AD]şi [A 0 D 0 ] congruente, cu D ∈ (BC), D 0 ∈ (B 0 C 0 ), iarm(\ADB) ≤ 90 ◦ , m( A\0 D 0 B 0 ) ≤ 90 ◦ .Propoziţia 1. În ipoteza (I), dacă \BAC ≡ B\0 A 0 C 0 şiBDC\BDA ≡ B\0 D 0 A 0 ,atunci4ABC ≡ 4A 0 B 0 C 0 .A′Demonstraţie. Din congruenţa triunghiurilor ADB şiA 0 D 0 B 0 (U.L.U.) rezultăcă AB = A 0 B 0 şi \ABD ≡ A\0 B 0 D 0 ,de unde concluzia urmează conformU.L.U.Propoziţia 2. În ipoteza (I), dacă \BAC ≡ B\0 A 0 C 0 şiAB = A 0 B 0 ,atunci4ABC ≡ 4A 0 B 0 C 0 .Demonstraţie.A\0 B 0 C 0 . Folosind U.L.U., deducem că 4ABC ≡ 4A 0 B 0 C 0 .Propoziţia 3. În ipoteza (I), dacă \BAC ≡ B\0 A 0 C 04ABC ≡ 4A 0 B 0 C 0 .Demonstraţie. Deoarece \BAD ≡ \B′ D′ C′Obţinem că 4ABD ≡ 4A 0 B 0 D 0 (U.L.U.), de unde \ABC ≡şi BD = B 0 D 0 ,atunciB 0 A 0 D 0 , AD = A 0 D 0 , BD = B 0 D 0 şi \ADB,A\0 D 0 B 0 sunt ambele neobtuze, conform L.L.U. rezultă că 4ADB ≡ 4A 0 D 0 B 0 şiacum concluzia este imediată.Propoziţia 4. În ipoteza (I), dacă AB = A 0 B 0 şi \ABC ≡ A\0 B 0 C 0 , atunci4ABC ≡ 4A 0 B 0 C 0 .Demonstraţie. Deoarece AB = A 0 B 0 , AD = A 0 D 0 , \ABC = A\0 B 0 C 0 şi \ADB,A\0 D 0 B 0 sunt ambele neobtuze, conform L.L.U. rezultă că 4ADB ≡ 4A 0 D 0 B 0 ,deunde concluzia anunţată.Propoziţia 5. În ipoteza (I), dacă \ADB ≡ A\0 D 0 B 0 şi BC = B 0 C 0 ,atunci4ABC ≡ 4A 0 B 0 C 0 .Demonstraţie. Considerăm punctul A 00 în acelaşi semiplan cu A faţă deBC,astfel încât 4A 00 BC ≡ 4A 0 B 0 C 0 .Fie[A 00 D 00 bisectoarea lui \BA 00 C, D 00 ∈ (BC), iar{E} = A 00 B ∩ AD, {F } = A 00 D 00 ∩ AC. Din\ADB ≡ A\00 D 00 B rezultă că AD k A 00 D 001 Asist. drd., Facultatea de Matematică, Univ. Al. I. Cuza, Iaşi2 Profesor,C.N.EmilRacoviţă, Iaşi107


şi cum AD = A 00 D 00 ,obţinem că ADD 00 A 00 este paralelogram. Astfel, AA 00 k BC şideci A 00 ∈ Ext ABC, de unde m( \A 00 CB) ≥ m(\ACB) şi m(\ABC) ≥ m( \A 00 BC) sauinvers; ne plasăm în prima situaţie.Din AD k A 00 D 00 obţinem că m( [EAC) =A A′′m( \AF A 00 ) = α şi m( \AEA 00 ) = m( \EA 00 F ) = β.ββ βAtunci m(\ABE) = 180 ◦ − α − (180 ◦ − β) =β − α şim( \A 00 CF) = 180 ◦ − β − (180 ◦ − α) =α − β. CumE Fα−β ≥ 0, deducem că α = β. Astfel,m( \A 00 CA)=0,deci C, A 00 , A sunt coliniare şi m( \ABA 00 )=0, deciB, A 00 , A sunt coliniare. În concluzie, A 00 = A, de B D D′′ Cunde 4ABC ≡ 4A 0 B 0 C 0 .Propozţia 6. În ipoteza (I), dacă AB = A 0 B 0 şi AC = A 0 C 0 ,atunci4ABC ≡4A 0 B 0 C 0 . (Problema VII.63 din RecMat - 2/2005)Demonstraţia 1. Considerăm punctul C 00 în acelaşisemiplan cu C faţă deAB, astfelîncât4ABC 00 ≡C C′′4A 0 B 0 C 0 . Fie [AD 00 bisectoarea lui \BAC 00 , D 00 ∈D D′′(BC 00 ). Din teorema bisectoarei avem că BDCD = ABACBD 00şiC 00 D 00 = ABAC 00 şi cum AC 00 = AC, deducem căABBDCD =BD00C 00 D 00 , deci DD00 k CC 00 . Triunghiurile ACC 00 şi ADD 00 sunt isoscele şiatunci A se află atât pe mediatoarea lui [DD 00 ],câtşipeceaalui[CC 00 ].Însăceledouă mediatoare sunt paralele sau coincid; rămâne cea de-a doua variantă şi astfelrezultă că A aparţine medianei din B în 4BCC 00 ,fals. Deducemcă C = C 00 ,adică4ABC ≡ 4A 0 B 0 C 0 .Observaţia 1. În această demonstraţie nu folosim explicit faptul că AD şi A 0 D 0sunt bisectoare, ci doar egalitatea rapoartelor BDDC = B0 D 0D 0 . Astfel, autorul problemei,Petru Asaftei, observăcărămâne adevărată concluzia dacă înlocuim în ipotezăC0 bisectoarele [AD] şi [A 0 D 0 ] cu două ceviene ce determină rapoarte egale pe latura pecare cad. În particular, concluzia rămâne adevărată pentru[AD], [A 0 D 0 ] medianesau simediane.Observaţia 2. Am putea denumi rezultatul Propoziţiei 6 cazul de congruenţăLatură-Bisectoare-Latură. Claudiu-Ştefan Popa ridică problemafuncţionării unorcazuri Bisectoare-Latură-Bisectoare sau Bisectoare-Bisectoare-Bisectoare; propunemcititorului demonstrarea sau invalidarea acestora. Pentru o eventuală justificare,maipromiţătoare pare calea urmată înDemonstraţia 2. Cu notaţiile uzuale, l a =2bcb + c cos A 2 şi l0 a = 2b0 c 0 A0b 0 cos+ c0 2 .Cum l a = la, 0 b = b 0 , c = c 0 ,obţinem cos A 2 =cosA0 2 , de unde A = A0 . Concluziarezultă din L.U.L.Propoziţia 7. În ipoteza (I), dacă \ABC ≡ A\0 B 0 C 0 şi BC = B 0 C 0 , atunci108


4ABC ≡ 4A 0 B 0 C 0 .Demonstraţie. Maiîntâi,săobservăm că:(i) m(\BDA) < 90 ◦ ⇔ AB < AC. Într-adevăr:m(\BDA) < 90 ◦ ⇔ BD > ccos B ⇔acb + c > a2 + c 2 − b 2⇔2ah2a 2 c>a 2 (b + c)+(c − b)(b + c) 2 ⇔ (b − c) (b + c) 2 − a 2i > 0 ⇔ b>c.(ii) În 4ABC, AB < AC, considerăm A 00 cu A ∈ (BA 00 ) şi fie [AD, [A 00 D 00bisectoarele unghiurilor \BAC, respectiv \BA 00 C, D, D 00 ∈ (BC); pesegmentul(BC)avem atunci ordinea B − D − D 00 − C. Într-adevăr, dacă AC = b, BC = a, AB = c,A 00 B = c 00 , A 00 C = b 00 şi m(\ABC) =α, avemcă a c + a c 00 > 2cosα. Relaţia esteevidentă pentruα ≥ 90 ◦ ,iarpentruα 2cosα ⇒ a c + a > 2cosα.c00 Atunci ³ ac − a ´³ ac 00 c + a ´ ³ ac 00 >c − a ´c 00 · 2cosα ⇒a 2c 2 > b002c 002 ⇒ b c > b00c 00 ⇒ D00 CD 00 B < DCDB ⇒ D ∈ (BD00 ) .Revenim la demonstraţia propoziţiei; putem presupunefără a restrânge generalitatea că AB ≤ A 0 B 0 . EA′′Considerăm A 00 ∈ [BA astfel încât 4A 00 BC ≡ 4A 0 B 0 C 0 A(deci A 00 B = A 0 B 0 ). Cum A ∈ (BA 00 ), conform observaţiei(ii), pe(BC) avem ordinea B−D−D 00 −C şi atuncim(\BAC) >m( \BA 00 C), adică m(\BDA) 90 ◦ şi deci ED > AD. Din DE k D 00 A 00 rezultă căCDED 00 A 00 =BEBA 00 < 1, deundeDE < D 00 A 00 . Obţinem astfel că AD < A 00 D 00 ,fals. Rămâne căAB = A 00 B, prin urmare 4ABC ≡ 4A 0 B 0 C 0 .Propoziţia 8. În ipoteza (I), dacă AB = A 0 B 0 şi BC = B 0 C 0 ,atunci4ABC ≡4A 0 B 0 C 0 .Demonstraţie. Presupunem prin absurd că m(\ABC)


Asupra problemei 3639din Gazeta Matematică, v. XXXIII (1927—1928)D. M. BĂTINEŢU-GIURGIU 1În volumul XXXIII (1927-1928) al Gazetei Matematice, la pagina 280, D. V.Ionescu 2 a propus problema 3639 cu următorul enunţ:Să se demonstreze călimm→∞limm→∞µmµmlimm→∞µmµ µµm 1+m 1 m − e = − e 2 , (1)µmµµ1+m 1 m − e + e = 11e2µ µµm 1+m 1 m − e + e − 11e2 2424 , (2)= − 21e48 , (3)e fiind baza logaritmilor neperieni.Soluţia acestei probleme a fost dată ded-niiG. G. Constantinescu, Ştefan E.Olteanu, George Silaş în Gazeta Matematică, vol. XLII (1936-1937), pag. 36-37.Mai întâi se arată că:µe m = 1+m 1 m µ= e 1 − 12m + 1124m 2 − 2148m 3 + a mm 4 , (4)undelimm→∞a m=0.Din(4) rezultă limitele din enunţ.m4 În această Notă ne propunem să generalizăm limita (1). Fie(x n ) n≥1un şir convergentde numere reale cu lim x n = x not= D 0 (x n ) ∈ R.n→∞Definiţie [1]. Spunem că şirul (x n ) n≥1admite o derivată dacăexistălim (n (x n − x)) not= D (x n ) ∈ R. Dacă D (x n ) ∈ R, spunem că şirul (x n )n→∞ n≥1estederivabil.³Propoziţia 1. Şirul (u n ) n≥1, u n = 1+n´s, r unde r ∈ R∗+ , s ∈ R + estederivabil.Demonstraţie. Dacă s = 0, atunci u n = 1, ∀n ∈ N; deci n (u n − 1) = 0,∀n ∈ N ∗ , deci limn→∞ (n (u n − 1)) = 0.³Dacă s ∈ R ∗ +,atunciD 0 (u n ) = lim u n = lim 1+ r =1. Decin→∞ n→∞ n´sµ lim (n (u un − 1³ ´n − 1)) = lim ln u n n =1· limn→∞ n→∞ ln u (ln nn→∞ un n)=ln limn→∞ un n =³ ³=ln lim 1+ r ´sn´µ µ ³=ln lim 1+ r ´ n rs r=lne rs = rs ∈ R,n→∞ nn→∞ n1 Profesor, Colegiul Naţional "Matei Basarab", Bucureşti2 Matematician român (1901—1985)110


ceea ce demonstrează propoziţia.Propoziţia 2. Dacă (x n ) n≥1este un şirconvergentcuD 0 (x n )= lim x n =n→∞³= x ∈ R ∗ xn´n,atuncişirul (x n ) n≥1este derivabil dacăşi numai dacăexistă lim =n→∞ x= d ∈ R ∗ +.Demonstraţie. Este evident căó µxn´nlim = lim 1+ x x! n(x n−x)xxn−xn − x= e 1 x lim (n(x n−x))n→∞ . (5)n→∞ x n→∞ xRelaţia (5) ne aratăcă, dacăexistă limn→∞³ xn´n= d ∈ R∗ xx+ , atunci d = e 1 lim (n(x n−x))n→∞ .Prin urmare există şi D (x n ) = lim (n (x n − x)) = x ln d ∈ R, adicăşirul (x n )n→∞ n≥1este derivabil.Reciproc, relaţia (5) ne arată că, dacă şirul (x n ) n≥1este derivabil, atunci existăD (x n )= lim (n (x n − x)) ∈ R, deciexistăşi d = limn→∞ n→∞e x 1 D(xn) ∈ R ∗ +. Cu aceasta propoziţia este demonstrată.³ xnx´n= e1xlim n→∞ (n(xn−x)) =În continuare vom nota lim x n = D 0 (x n )=x ∈ R şi lim y n = D 0 (y n )=y ∈ R.n→∞ n→∞Propoziţia 3 [2]. Dacă şirurile (x n ) n≥1, (y n ) n≥1sunt derivabile, atunci şirul(x n − y n ) n≥1este derivabil şi D (x n y n )=D (x n ) y + xD (y n ).Demonstraţie. Conform enunţului avem:D 0 (x n )=x = lim x n, D 0 (y n )=y = lim y n ∈ Rn→∞ n→∞şi, de asemenea,D (x n )= lim (n (x n − x)) , D(y n )= lim (n (y n − y)) ∈ R.n→∞ n→∞Prin urmare, trebuie de arătat căD (x n y n ) = lim (n (x ny n − xy)) ∈ R.n→∞Într-adevăr,n (x n y n − xy) =n (x n y n − xy n + xy n − xy) =y n n (x n − x)+xn (y n − y) , ∀n ∈ N ∗ ,de unde, prin trecere la limită cun →∞, deducem:D(x n y n )= lim (n (x ny n − xy)) = lim (n (x n − x)) lim y n + x lim (n (y n − y)) =n→∞ n→∞ n→∞ n→∞= D (x n ) D 0 (y n )+D 0 (x n ) D (y n ) .Cu aceasta propoziţia este demonstrată.În continuare, fie r ∈ R ∗ +, s ∈ R + şi vom adopta notaţiile:³e n (r, s) = 1+n´n+s r ³, en (r) =e n (r, 0) = 1+n´n r ,³u n = 1+n´s r µ, en = 1+n 1 n, ∀n ∈ N ∗ .111


³Se observă că lim e n (r, s) =e r , lim e n (r, 0) = lim 1+ r = en→∞ n→∞ n→∞ n´n r , lim u n =1,n→∞lim e n = e.n→∞Propoziţia 4. Şirul (e n (r)) n≥1este derivabil şiD (e n (r)) = − r2 e r2 . (6)Demonstraţie. Conform inegalităţilor Bencze-Tóth din [3] aveme ran + b


Oproblemă despre suma cifrelor unui număr naturalîn baze de numeraţie oarecareAdrian ZAHARIUC 11. Introducere. Studiul cifrelor puterilor unui număr întreg este o problemădificilă de teoria numerelor. Pentru ultimele progrese în acest sens, cititorul poateconsulta [2]. Există multe probleme, evidente din punct de vedere intuitiv, careîncă sunt departe de a fi rezolvate. Chiar şi faptul că suma cifrelor lui a n tinde lainfinit nu este atât de evident, fiind un rezultat binecunoscut al lui Schnirelman.Printre întrebările ce apar, una pare adevărată dincolo de orice îndoială, dar aproapeimposibil de atacat.Propoziţia 1. a şi b sunt numere naturale nenule astfel încâts(a n )=s(b n ) (1)pentru orice n. Atuncilg a − lg b ∈ Z. (s (N) notează suma cifrelor numărului N.)Observaţie. Desigur, lg a − lg b =lga/b, deci lg a − lg b ∈ Z înseamnă defaptcăa = b sau că numărul mai mare dintre a şi b este obţinut din celălalt prin adăugareacâtorva 0-uri la sfârşit. Este evident că reciproca acestui fapt este adevărată.Propoziţia 2. a şi b sunt numere naturale nenule astfel încâts(an) =s(bn) (2)pentru orice n. Atuncilg a − lg b ∈ Z.Este clar că aceasta este o variantă mai slabă a problemei anterioare. De fapt,(2) este mult mai restrictivă decât (1); încazcă (2) este adevărată, avems(a n )=s(a n−1 b)=s(a n−2 b 2 )=···= s(ab n−1 )=s(b n ),deci (1) este de asemenea adevărată. Astfel, am devenit interesaţi de această întrebare.Am crezut că nuavemdeverificat decât un lucru simplu, dar adevărul s-adovedit a fi altul.2. Observaţii preliminare. O modalitate naturală deaatacaproblemaestesăvedemcaretermeniaişirurilor s(an) şi s(bn) pot fi calculaţi uşor. Această metodănu se va dovedi eficientă, dar ne va da o imagine asupra problemei. Este cunoscutăurmătoareaLemă. Dacă N ≤ 10 k − 1, atuncis ¡¡ 10 k − 1 ¢ N ¢ =9k.Demonstraţie. Putem presupune că 10 nu divide N. FieN = b 1 b 2 ...b k , undeprimele cifre pot fi şi nule. Este mai convenabil şi mai uşor să calculăm o diferenţă1 Elev, cl. a XI-a, Colegiul Naţional "Ferdinand I", Bacău113


decât un produs:s ¡¡ 10 k − 1 ¢ N ¢ = s ¡ 10 k N − N ¢ = s(b 1 b 2 ...b k 00|...00{z }− b 1 b 2 ...b k )=k cifre³´= s b 1 b 2 ...b k−1 (b k − 1)(9 − b 1 )(9 − b 2 ) ...(9 − b k−1 )(10 − b k ) =9k,deci lema este demonstrată.Ce se întâmplă dacă folosim aceasta în problema noastră? Dacă 10 k − 1 >aşi b,atuncis ¡¡ 10 k − 1 ¢ a ¢ =9k şi s ¡¡ 10 k − 1 ¢ b ¢ =9k,deci, de fapt, s(an) =s(bn) pentru toţi n =10 k − 1 suficient de mari, ignorândipoteza noastră. Aşadar, din aceastăobservaţie, împreunăcuobservaţia căadăugarea0-urilor în numerele n nu are efect, concluzionăm că toate şirurile de forma s(an) austructuri similare şi chiar valori comune pentru anumite valori ale lui n. Mai mult,este clar că alegerea unor valori "frumoase" ale lui n nu este o tehnică utilă.Dar ce s-ar întâmpla dacă amreuşi să găsim forme simple pentru an sau bn?Aceasta este prima idee utilă.Desigur, putem tăia 0-urile de la sfârşitul numerelor a şi b, deci 10 nu divide nicia nici b, şi ne ramâne să arătăm că a = b. Presupunem că nuesteadevărat şi luăma>b. Atunci, evident, pentru orice n, an > bn. Vrem săgăsim un număr an careare suma cifrelor mai mare decât cea a tuturor numerelor mai mici decât el. Evident,aceasta va contrazice s(an) =s(bn) şi problema este rezolvată. Dar care sunt acestenumere? Sunt numerele care conţin doar cifra 9, eventual, cu excepţia primei cifre.Nu este greu să vedem că a are un astfel de multiplu dacă şi numai dacă (a, 10) = 1.Deci, în acest caz, problema este rezolvată. Din păcate, nu putem rezolva pe aceeaşiidee cazul în care a sau b sunt divizibile cu puteri mari ale lui 2 sau 5. Oricum,dacă am fi folosit o bază primăînlocullui10, problema ar fi fost rezolvată. Dinacest motiv considerăm potrivit să generalizăm problema pentru o bază de numeraţieoarecare.Propoziţia 3. Fie b ≥ 2, iarm şi n două numere naturale nenule astfel încâts b (km) =s b (kn),pentru orice k. Atuncilog b m − log b n ∈ Z.3. Soluţia problemei. Pasul 1. Vom arăta ca fracţia n/m are un număr finitde cifre după virgulă. Pentru fiecare i, existăunk i ∈ Z astfel încâtb i ≤ k i m ≤ b i + m − 1.Atunci, deoarece funcţia s b este subaditivă,s b (k i m) ≤ s b (b i )+s(k i m − b i )=1+s b (k i m − b i ) ≤ 1+k i m − b i ≤ m.Să fixăm j ∈ N, foarte mare. Vom demonstra că pentru orice i suficient de mare,primele j − 1 cifre ale lui k i n coincid cu primele j − 1 cifre ale lui n/m. Primacifră114


aluin/m este prima cifră din stânga diferită de0 indiferent dacă este înainte saudupă virgulă. Fiej C = b −j nm bjk .AvemC ≤ n m = k ink i m


Înlocuind k din ipoteza s b (km) =s b (kn) cu kk 1 ,obţinem!tYs b (k 1 km) =s b (k 1 kn) ⇔ s bÃk p fα i−s i +r i¡i N = s b kb f N ¢ = s b (kN), ∀k ∈ NPentru simplitate, fie1K =tY1p fαi−si+rii.Atunci, s b (kNK) =s b (kN) pentru toţi k ∈ N, care reduce problema cu mult. Esteimportant să ţinem minte că (N,b) =1.Pasul 3. Să arătăm că log b K ∈ Z. Deoarece s b (kNK) =s b (kN) pentru toţik ∈ N, deducem căs b (N) =s b (KN)=s b (K 2 N)=s b (K 3 N)=...În particular, aceasta înseamna că mulţimea {s b (K u N):u ∈ N} este marginită.Dacă log b K ∈ R \ Q, vom arăta că pentru orice înşiruire de cifre, există unuastfel încât K u N începe exact cu acea secvenţă de cifre, ceea ce ar conduce la ocontradicţie. Pentru aceasta, este suficient să arătăm că mulţimea{{log b K u N} : u ∈ N},unde {x} = x−bxc, estedensăîn[0, 1]. Dar, deoarece log b K u N = u log b K +log b N,aceasta rezultă imediat din lema lui Kronecker. Amintim că lemaluiKroneckerafirmă faptulcăpentruoriceα ∈ R \ Q, mulţimea {{αn} : n ∈ N} este densă în(0, 1). Aşadar, am arătat că log b K ∈ Q.Să presupunem, prin absurd, că log b K ∈ Q\Z. Este foarte interesant săobservămcă acest lucru nu este posibil decât atunci când b este o putere netrivială şi cădemonstraţia este făcută în toate celelalte cazuri. Observăm că log b K ∈ Q înseamnăde fapt că, pentru orice p, exp p K =log b K exp p b.Fier = blog b Kc; atunci exp p K ≥r · exp p b, deci b r | K. Fie K 0 = K/b r ;esteclarcă K 0


Generalizări ale unor inegalităţi din RecMatAlexandru NEGRESCU 1Ne propunem să generalizăm următoarele inegalităţi apărute în revista "RecreaţiiMatematice":X.53. (nr. 2/2004, p. 155). Fie a, b, c ∈ (1, ∞) astfel încât a + b + c =9.Săsearate că ¡log a 2b 3 + c 3¢ ¡+log b 2c 3 + a 3¢ ¡+log c 2a 3 + b 3¢ ≥ 12.Angela ŢigăeruIX.48 (nr. 1/2004, p.77). Fie a, b, c ∈ (0, ∞) cu a + b + c + √ abc =4. Săsearate căa 2a + √ bc + b 2b + √ ca + c 2c + √ ab ≥ 3 2 .Cezar LupuVII.17 (nr. 1/2001, p. 74). a) Fie x, y, z ∈ [2, ∞). Arătaţi că¡x 2 + y ¢¡ y 2 + z ¢¡ z 2 + x ¢ ≥ 27xyz.b) Fie x, y, z ∈ [3, ∞). Arătaţi că ¡ x 2 + y ¢¡ y 2 + z ¢¡ z 2 + x ¢ ≥ 64xyz.Lucian TuţescuSoluţii ale acestor probleme pot fi găsite în numerele 2/2005, 1/2005 şi respectiv1/2002 ale revistei.Problema 1. Date numerele a 1 ,a 2 ,...,a n ∈ (1, ∞), n ≥ 2, fie S = a 1 + a 2 ++ ···+ a n .Săsearatecălog a1(2a n 2 + a n 3 + ···+ a n n)+log a2(a n 1 +2a n 3 + ···+ a n n)+···+¡+log an 2an1 + a n 2 + ···+ a n ¢ nn−1 ≥log n S − 1 + n2 ,cu egalitate dacă şi numai dacă a 1 = a 2 = ···= a n = S n .Soluţie. Notăm cu E membrul stâng al inegalităţii. Utilizând inegalitatea mediilor,avem:¡ ¢ ¡E ≥ log a1 na22 a 3 ···a n +loga2 na1 a 2 ¢ ¡ ¢3 ···a n + ···+logan na21 a 2 ···a n−1 =nX= log ain +2 ¡ ¢log a1a 2 +log a2a 3 + ···+log ana 1 +Dar≥i=1+ £ log a1(a 3 ···a n )+log a2(a 1 a 4 ···a n )+···+log an(a 2 ···a n−1 ) ¤ ≥nXlog ain +2n + nqlog n a1(a 3 ···a n ) ···log an(a 1 ···a n−1 ).i=1log a1(a 3 ···a n )= ln a 3 +lna 4 + ···+lna n≥ (n − 2) (ln a 3 ···ln a n ) 1n−2.ln a 1 ln a 11 Elev, cl. a XI-a, Colegiul Naţional "A. T. Laurian", Botoşani117


Cu această inegalitate şi analoagele ei, vom aveanXnXE ≥ log ain +2n + n (n − 2) ==i=1n 2log n (a 1 a 2 ···a n ) + n2 ≥i=11log n a i+ n 2 ≥n 2log nµa1 + a 2 + ···+ a nnn 2P ni=1 log n a i+ n 2 = n + n 2 =nlog n S − 1 + n2şi astfel inegalitatea este dovedită. Cazul în care are loc egalitate se obţine cu uşurinţă.Observaţie. Inegalitatea din Problema X.53 se obţine pentru n =3şi S =9.Problema 2. Date numerele a 1 ,a 2 ,...,a n ∈ (0, ∞), n ≥ 3, fie S = a 1 + a 2 ++ ···+ a n + n−1√ a 1 a 2 ···a n .Săsearatecăa 2 1a 2a 1 + k n−1√ 2+a 2 a 3 ···a n a 2 + k n−1√ + ···+a 1 a 3 ···a n³1 ´1+a 2 n SS −nn+1+a n + k n−1√ ≥,a 1 a 2 ···a n−1 k +1cu egalitate dacă şi numai dacă a 1 = a 2 = ···= a n =1şi S = n +1.Soluţie. Notând cu E primul membru al inegalităţii şi utilizând inegalitateaCauchy-Buniakowski-Schwarz şi cea a mediilor, obţinemE ≥ (a 1 + a 2 + ···+ a n ) 2P ¡a1 + k n−1√ a 2 a 3 ···a n¢ ≥(a 1 + a 2 + ···+ a n ) 2P µ a 1 + k a =2 + a 3 + ···+ a nn − 1= (P ni=1 a i) 2 P n(k +1) P i=1ni=1 a =a ii k +1 .(∗)Pe de altă parte, pornind de la S = a 1 + a 2 + ···+ a n + n−1√ a 1 a 2 ···a n şi folosinddin nou inegalitatea mediilor, vom aveaS ≥ (n +1) n+1 v uut nYi=1a in−1snQa i ,de undeµ Ã n+1 n!nSY n − 1≥ a i .n +1i=1Ca urmare,v uut nXYn µ n +1S na i = S − n−1 a i ≥ S −.n +1i=1i=1Introducând în (∗), vom obţine inegalitatea din enunţ. Egalitate avem dacăşi numaidacă a 1 = a 2 = ···= a n = n−1√ a 1 ···a n =Sn +1 ,adică a i =1, i ∈ N şi S = n +1.118i=1


Observaţii. 1) Mai general, putem considera a 1 ,a 2 ,...,a n ∈ (0, ∞), n ≥ 3, şiS = a 1 + a 2 + ···+ a n + p√ a 1 a 2 ···a n , p ≥ 3; sevaobţine⎡⎤E ≥ 1 µ Sp(n+1)p+1⎣S −⎦ .k +1 n +12) Pentru n =3şi k =1regăsim inegalitatea din Problema IX.48.În privinţa Problemei VII.17, vom spune mai întâi că în nr. 1/2002, la pag. 66,sunt prezentate două demonstraţii pentru următoarea generalizare a acesteia:Fie x, y, z ∈ [n, ∞), n ∈ N. Arătaţi că ¡ x 2 + y ¢¡ y 2 + z ¢¡ z 2 + x ¢ ≥ (n +1) 3 xyz.O inegalitate şi mai generală estedatădeProblema 3. Fie x 1 ,x 2 ,...,x n ∈ [k, ∞), k, n ∈ N ∗ şi n ≥ 2. Arătaţi că¡ ¢¡ ¢ ¢xn−11 + x 2 xn−12 + x 3 ···¡xn−1n + x 1 ≥ (k +1) n (x 1 x 2 ···x n ) k(n−2)+1k+1,Cu egalitate dacă şi numai dacă x i = k =1, i = 1,n,saux i = k, n =3, i = 1, 3.Soluţie. Să notăm din nou cu E membrul întâi al inegalităţii de demonstrat.Observăm că x 1 ≥ k implică x n−11 ≥ kx k−21 şix n−11 + x 2 ≥ kx n−21 + x 2 = x n−21 + x n−21 + ···+ x n−21 + x 2 .| {z }qkDeci x n−21 + x 2 ≥ (k +1) k+1 x k(n−2)1 x 2 . Utilizând această inegalitate şi analoageleei, obţinem căqE ≥ (k +1) n k+1x k(n−2)+11 x k(n−2)+12 ···x k(n−2)+1n =(k +1) n (x 1 x 2 ···x n ) k(n−2)+1k+1,cu egalitate aşa cum este specificat în enunţ.Observaţie. Inegalităţile din Problema VII.17 se obţin pentru k =2, n =3şirespectiv k =3, n =3, iar generalizarea lor, dată înrevistă, pentru n =3.Când se naşte, omul zice AA. . . . Când moare, zice MOR... .Aşadar, intervalulvieţii este [A, MOR], adică AMOR.Vizitaţi pe Internet revista "Recreaţii Matematice" la adresahttp://www.recreatiimatematice.uv.ro119


Cea mai bună inegaliate de acest tip ...Marian TETIVA 11. Introducere. Aţi întâlnit cu siguranţă asemeneaenunţ(maialesdacăvăpreocupă şi probleme mai grele, "de olimpiadă"). Iată treiastfeldeexemple:1. Cea mai bună inegalitate de tipul(x + y + z) 3 − 27xyz ≥ k[(x + y + z)(xy + xz + yz) − 9xyz], ∀x, y, z > 0se obţine pentru k =4(este aşa-numita inegalitate a lui Schur).2. Cea mai bună inegalitate de tipulp ≤ kR + hrcare are loc în orice triunghi se obţine pentru k =2şi h =3 √ 3 − 4 (se numeşteinegalitatea lui Blundon în acest caz).3. Cea mai bună inegalitate de formaa 2 + b 2 + c 2 ≤ kR 2 + hr 2şi care are loc, iarăşi, în orice triunghi este cea obţinută pentru k =8şi h =4.Cum se abordeazăoasemeneaproblemă? Răspundem imediat la această întrebarerezolvând-o pe prima (a se vedea şi [2], problema 9.22, un pic altfel formulată acolo);celelalte sunt soluţionate în [3], respectiv [1]. Iată mai departe soluţia problemei 1.Trebuie săînţelegem bine enunţul; observăm că expresia din membrul drept(x + y + z)(xy + xz + yz) − 9xyz,are numai valori nenegative, conform unei inegalităţi cunoscute. De aceeak 1 [(x + y + z)(xy + xz + yz) − 9xyz] ≤ k 2 [(x + y + z)(xy + xz + yz) − 9xyz]pentru x, y, z > 0 şi k 1 ≤ k 2 ,aşa că problemanoastră este, de fapt, problema găsiriicelui mai mare număr k astfel încât inegalitatea(x + y + z) 3 − 27xyz ≥ k[(x + y + z)(xy + xz + yz) − 9xyz]săfieadevărată pentru orice x, y, z > 0. De obicei într-o asemenea chestiune, se cautămai întâi cea mai mare valoare posibilă aluik (cea mai mică, în alte cazuri), dândvalori particulare variabilelor, apoi se demonstrează inegalitatea rezultată pentruvaloarea extremă depistatăpentruk. Cel mai comun (şi comod) este să egalămvariabile; în cazul de faţă, săpunemy = z =1şi x>0 arbitrar. Obţinem inegalitatea(x +2) 3 − 27x ≥ k[(x +2)(2x +1)− 9x] ⇔ (x − 1) 2 (x +8)≥ 2k(x − 1) 2 ,care trebuie să aibălocpentruoricex>0 (pentru că inegalitatea iniţială vremsăaibă locpentruoricex, y, z > 0). De fapt asta înseamnă x +8≥ 2k, pentruoricex 6= 1şi, în ultimă instanţă, pentru orice x>0. Aici are loc de obicei un procesde trecere la limită, pentru a obţine cel mai bun k. Limita se calculează, uneori, înpuncte care sunt capete ale intervalelor pe care rezultă inegalitatea finală; în cazulde faţă, pentru x → 0, obţinem k ≤ 4. (Evident, aici trecerea la limită în0nuera1 Profesor, Colegiul Naţional "Gheorghe Roşca Codreanu", Bârlad120


strict necesară, căci x, y, z pot fi considerate ≥ 0 -nuneapărat strict pozitive - şiraţionamentul funcţionează la fel). Astfel am aflat că, probabil, cea mai bună valoarealuik este 4; numai dacă reuşim să demonstrăm inegalitatea pentru k =4putemtrage concluzia finală, cum că "ceamaibună inegalitate de tipul"(x + y + z) 3 − 27xyz ≥ k[(x + y + z)(xy + xz + yz) − 9xyz], ∀x, y, z > 0se obţine pentru k =4. Ori, înlocuind această valoare a lui k, ajungem lax 3 + y 3 + z 3 +3xyz ≥ x 2 y + xy 2 + x 2 z + xz 2 + y 2 z + yz 2 , ∀x, y, z > 0,inegalitate bine cunoscută (numită uneori "inegalitatea lui Schur", aşacumammaispus), care apare în multe cărţi (inclusiv în [2]), deci nu o mai demonstrăm aici.2. Din nou despre normare. Iniţial această lucrare s-a vrut o continuare anotei [4]. Între timp, s-a ivit această nouă inegalitate (vom arătalaurmăprinceconcurs de împrejurări am ajuns la ea) care s-a lăsat rezolvată cumetodanormării(nu ezitaţi să o folosiţi şi pentru inegalitatea lui Schur! de fapt, aşa e făcută şi în[2]) şi, în plus, ilustrează bineşigenuldeproblemepecarel-amdescrissus;iatăde ce am decis să amânăm publicarea soluţiilor exerciţiilor din [4] (maialescă, nu-iaşa?, "cititorii noştri sunt la fel de inteligenţi ca şi noi", ba chiar mai) în favoareaproblemei pe care o prezentăm chiar acum.Problema 1. Care este cea mai bună inegalitate de tipul(a 2 + b 2 + c 2 )(a 2 + b 2 + c 2 − ab − ac − bc) ≥≥ k(a + b + c)(a 2 b + ab 2 + a 2 c + ac 2 + b 2 c + bc 2 − 6abc), ∀a, b, c > 0?Cu alte cuvinte, deoarece şi aici a doua paranteză din membrul drept are valorinenegative şi a, b, c sunt strict pozitive, trebuie să determinăm cea mai mare valoarepe care o poate lua k astfel încât inegalitatea de mai sus să fieadevărată pentru oricea, b, c > 0. Şi de această dată începem cu particularizarea b = c =1care ne conducela (a 2 +2)(a − 1) 2 ≥ 2k(a +2)(a − 1) 2 ,deci,pânălaurmă a 2 +2≥ 2k(a +2),carear trebui să fieadevărată pentru orice a>0, a 6= 1,adică(pebazacontinuităţii)pentru orice a>0. Facemiarpea să tindăla0pentruagăsi 2k ≤ 1 dar, surpriză!Pentru k =1/2 şi b = c =1inegalitatea devine a(a − 1) 3 ≥ 0 şi, prin urmare, estedeparte de a fi adevărată pentru orice a, b, c > 0. Trebuie deci să rafinăm evaluarealui k, observând că, de fapt, am obţinut2k ≤ a2 +2, ∀a >0;a +2ceea ce înseamnă că, în mod necesar, 2k este cel mult egal cu valoarea minimă afuncţiei f(x) = x2 +2pe intervalul (0, ∞). (De aici şi deosebirea faţă decazulx +2studiat anterior: minimul nu se atinge în capătul intervalului.) Aceasta se dovedeştea fi ceva mai mică decât limita funcţiei în origine, adică decât 1; valoarea minimăafuncţiei f este 2 √ 6 − 4 (se realizează pentrux = √ 6 − 2), astfel că ar trebui săavem k ≤ √ 6 − 2 (fără să fie deloc clar deocamdată dacă aceasta este, într-adevăr,valoarea care produce cea mai bună inegalitate). Oricum, putem considera maideparte k ∈ [0, √ 6−2] (în mod clar valorile negative ale lui k nu pot fi luate în discuţiedrept candidate pentru "cel mai bun" k); în particular, avem k ≤ √ 6 − 2 < 1/2 ⇒121


1 − 2k >0. Mai departe încercăm, cu metoda normării, să vedem ce se întâmplă cuinegalitatea noastră pentru asemenea k şi a, b, c numere pozitive oarecare. Datorităsimetriei putem presupune c =min{a, b, c}; notăm atunciac =1+x, bc =1+y,unde x şi y trebuie să fie nenegative. Inegalitatea mai poate fi scrisă înformaXa 4 +(2− 2k) X a 2 b 2 +(4k − 1) X a 2 bc ≥ (k +1) X (a 3 b + ab 3 )(toate sumele sunt ciclice) sau înlocuind a c cu 1+x şi b cu 1+y,c(1 + x) 4 +(1+y) 4 +1+(2− 2k) £ (1 + x) 2 (1 + y) 2 +(1+x) 2 +(1+y) 2¤ ++(4k − 1) £ (1 + x) 2 (1 + y)+(1+x)(1 + y) 2 +(1+x)(1 + y) ¤ ≥≥ (k +1) £ (1 + x) 3 (1 + y)+(1+x)(1 + y) 3 +(1+x) 3 +(1+y) 3 +1+x +1+y ¤ .Acum urmează parteaneplăcută (dar inevitabilă când folosim metoda normării!)în care trebuie să facem calculele; pe care nu le vom reproduce aici (dar vă sfătuimsă le verificaţi!. . . ). Inegalitatea de demonstrat capătă formax 4 − (k +1)x 3 y +(2− 2k)x 2 y 2 − (k +1)xy 3 + y 4 ++(2 − 2k)x 3 − 3x 2 y − 3xy 2 +(2− 2k)y 3 +(3− 6k)(x 2 − xy + y 2 ) ≥ 0,sau încă12 (x − y)4 + 1 2 (x2 − y 2 ) 2 +(1− k)xy(x − y) 2 +2(1− 2k)x 2 y 2 ++2(1 − k)(x + y)(x − y) 2 +(2− 5k)xy(x + y)+3(1− 2k)(x − y) 2 +3(1− 2k)xy ≥ 0.Acum putem demonstra inegalitatea pentru k ≤ √ 6 − 2; oricum, ea este evidentăpentru k 0.În membrul stâng al inegalităţii de demonstrat toţi termenii sunt în mod evidentnenegativi, cu excepţia lui (2 − 5k)xy(x + y) care poate fi luat în grupul2(1 − 2k)x 2 y 2 +(2− 5k)xy(x + y)+3(1− 2k)xy == xy [2(1 − 2k)xy +(2− 5k)(x + y)+3(1− 2k)] ,pe care-l mai putem scrieà µ !2 x + yxy 2(1 − 2k) +2(2− 5k) x + y +3(1− 2k) − 1 − 2k xy(x − y) 2 .222Dar paranteza mare este nenegativă, conform observaţiei referitoare la trinomul degradul al doilea, iar ultimul termen este "anihilat" de (1 − k)xy(x − y) 2 :(1 − k)xy(x − y) 2 − 1 − 2k xy(x − y) 2 = 1 22 xy(x − y)2 ≥ 0122


şi inegalitatea este demonstrată; valoarea cea mai bună (maximă) a lui k este, întradevăr,√ 6 − 2.3. Poate că nu e lipsit de interes să vedem cum am ajuns la această inegalitate:desigur, din întâmplare (multe ies astfel . . . )! Am vrut să vedem ce se obţinedacă aplicăm inegalitatea lui Jensen funcţiei radical în felul următor:a p b p c p(b + c)2 +(a + c)2 +(a + b)2 ≤a + b + ca + b + ca + b + cra≤a + b + c (b + bc)2 +a + b + c (a + cc)2 +a + b + c (a + b)2 .(Asta mai rezultă şi folosind inegalitatea Cauchy-Buniakowski-Schwarz). După ridicarela pătrat şi încă câteva calcule ajungem la4(ab + ac + bc) 2≤ 4 a + b + c 9 (a + b + c)3 + 30abc − 4 P a 3 − 3 P a(b 2 + c 2 )9sau, totuna,4(a 2 + b 2 + c 2 +5ab +5ac +5bc)(a 2 + b 2 + c 2 − ab − ac − bc) ==4 £ (a + b + c) 4 − 9(ab + ac + bc) 2¤ ³≥ (a + b + c) 4 X a 3 +3 X ´a(b 2 + c 2 ) − 30abc .Mai ţinemcontacumdedouă lucruri: prima paranteză din membrul stâng nu depăşeşte6(a 2 + b 2 + c 2 ),şi a doua este nenegativă conform inegalităţiia 2 + b 2 + c 2 ≥ ab + ac + bc,pe când cea de-a doua paranteză din membrul drept se poate minora (pe baza inegalităţiilui Schur) cu4 X a(b 2 + c 2 ) − 12abc +3 X ´a(b 2 + c 2 ) − 30abc =7³Xa(b 2 + c 2 ) − 6abc ;astfel suntem conduşi la inegalitatea24(a 2 + b 2 + c 2 )(a 2 + b 2 + c 2 − ab − ac − bc) ≥≥ 7(a + b + c)(a 2 b + ab 2 + a 2 c + ac 2 + b 2 c + bc 2 − 6abc), ∀a, b, c > 0şi întrebarea din problema 1 vine în mod natural (după ce am demonstrat inegalitateapentru k = 7 ). Putem vedea acum că valoarea lui k de la care am pornit este destul24de departe de cel mai bun k; avem,totuşi, o demonstraţie fără preamultecalculeaunei inegalităţi de acest tip, care ne poate da speranţe că s-arputeafaceaşa şi "ceamai bună dintreacesteinegalităţi". De asemenea, vă propunem spre rezolvareProblema 2. Găsiţi cea mai bună inegalitate de forma(x + y) 2 (x + z) 2 (y + z) 2 +(27k − 64)x 2 y 2 z 2 ≥ kxyz(x + y + z) 3 , x,y,z >0.Bibliografie1. L. Panaitopol - O inegalitate geometrică, Gazeta Matematică seria B, 4/1982.2. L. Panaitopol, V. Băndilă, M. Lascu - Inegalităţi, EdituraGIL,Zalău, 1996.3. V. Vâjâitu - Asupra unei inegalităţi, Gazeta Matematică seria B, 7/1984.4. M. Tetiva - Metoda normării, RecMat - 1/2006, 30-34.123


Societatea de Ştiinţe Matematice din R. MoldovaAnul 2006 aduce cu sine două date marcante în cercetarea şi în învăţământulsuperior din Republica Moldova. Două instituţii ce constituie imaginea Basarabieipostbelice în ştiinţa contemporană -Academia de Ştiinţe a Moldovei şi Universitateade Stat din Moldova - împlinesc 60 de ani de la fondare. Şcoalamatematică din R. Moldova datorează în mare parte acestora evoluţia sa. Date relevantedespreistoriaacestorinstituţiipot fi găsite în volumele: Institutul de Matematicăşi In<strong>format</strong>ică. File din istorie, editat de Acad. de Şt. a Moldovei în 2004, şiIstoria Universităţii de Stat din Moldova (1946-1996), editat de USM în 1996.La afirmarea şcolii matematice basarabene au contribuit un şir de personalităţide autentică valoare, fondatori de şcoli ştiinţifice: Vladimir Andrunachievici,Valentin Belousov (algebră), Constantin Sibirschi (ecuaţii diferenţiale), Izrael Gohberg(analiză funcţională), Alexandru Zamorzaev (geometrie), Alexandru Cuzneţov(logică matematică) ş.a. Şcolile ştiinţifice create în Republica Moldova au reuşit săse impună în lumea matematică prin realizări valoroase, devenind centre de pregătireaspecialiştilor din republică şidepestehotaretimpdecâtevadecenii.Societatea Matematică afostprezentă în Basarabia în diferite forme de organizare,reunind în rândurile sale cercetători, profesori de matematică din universităţişi şcoli şi chiar elevi. În actualul său statut Societatea Matematică din RepublicaMoldova (SMM) activează din anul 1993. În prezent SMM întruneşte 180 de membri,incluzând matematicieni consacraţi, tineri cercetători, profesori de la instituţiisuperioaredeînvăţământ, licee şi şcoli. În anul 2004 Adunarea Generală aSMMl-areales în calitate de preşedinte pe academicianul Mitrofan Ciobanu, iarvicepreşedinţipe profesorii universitari Nicolae Jitaraşu şi Alexei Caşu.Activitatea de bază aSMMţine de promovarea cercetărilor ştiinţifice în domeniulmatematicii şi al in<strong>format</strong>icii, de monitorizarea sistemului de învăţământ matematicla toate nivelele şi de consolidarea comunităţii matematice.Un element important în consolidarea comunităţii matematice din RepublicaMoldova, pe parcursul anilor, au fost conferinţele ştiinţifice. În ultimii 50 au fost organizatepeste 50 conferinţe ştiinţifice, inclusiv ediţii ale unor conferinţe internaţionaleex-sovietice şi europene. La organizarea acestor manifestări ştiinţifice aportul SocietăţiiMatematice a fost esenţial, cu toate acestea prima conferinţă SMMafostorganizată abia în anul 2001. Beneficiind de sprijinul financiar al UNESCO, primaconferinţă aSocietăţii Matematice din Republica Moldova a reuşit să întruneascăpeste 160 de participanţi, inclusiv 40 de oaspeţi de peste hotare, specialişti în domeniulmatematicii şi al in<strong>format</strong>icii din România, Franţa, Germania, Rusia, Polonia,Ucraina, Belarus, Serbia şi Muntenegru. La conferinţă au fost prezentate 148 derapoarte iar ulterior au fost publicate materialele conferinţei în două numerealerevistei“Buletinul Academiei de Ştiinţe a Republicii Moldova. Matematică”, unnumăral revistei “Computer Science Journal of Moldova” şioculegeredearticoleîndomeniuldidacticii matematicii şi al in<strong>format</strong>icii. În anul 2004 a fost organizată a douaconferinţă SMM,fiind dedicată aniversării a 40-a de la fondarea Institutului deMatematică şi In<strong>format</strong>ică alA.Ş. R.M. La această conferinţă auparticipat124


peste 170 de specialişti în matematică şi in<strong>format</strong>ică, inclusiv 30 de invitaţi de pestehotare (SUA, România, Germania, Ucraina, Rusia, Israel, Canada ş.a.).În anul 2002, susţinând iniţiativa societăţilor matematice din Grecia, România şiBulgaria, SMM a participat la fondarea Societăţii Matematice din Sud-EstulEuropei (MASSEE) cu sediul la Atena, Grecia. Fondarea MASSEE a constituit onouă etapă în cooperarea regională a matematicienilor din Balcani.Criza extinsă prin care a trecut R. Moldova în primii săi ani de existenţă aavutrepercursiuni negative asupra tuturor sferelor de activitate şi în primul rând asupradezvoltării ştiinţelor fundamentale. Astfel, SMM şi-a propus drept scop identificareaunor modalităţi de depăşire a dificultăţilor ce afectează cercetarea şi educaţia înR. Moldova. Prima conferinţă SMMaadresatconduceriiR.Moldovaunapelîncare au fost formulate propuneri concrete vizând probleme de stringentă actualitateîn domeniile cercetare-dezvoltare şi învăţământ. Iniţiativele SMM, incluse în acestapel, au influenţat politica Guvernului şi a altor structuri statale abilitate, avânddrept urmare anunţarea unor concursuri naţionale de granturi, inclusiv granturi decercetare în matematică şi în in<strong>format</strong>ică, granturi pentru organizarea conferinţelorşi a simpozioanelor ştiinţifice, pentru editarea revistelor de specialitate etc.În anul 2004 a intrat în vigoare Codul cu privire la ştiinţă şi inovare al R. Moldovace reglementează raporturile juridice ce ţin de elaborarea şi promovarea politicii destat în sfera ştiinţei şi inovării, de acreditarea organizaţiilor din sfera ştiinţei, deatestarea cadrelor ş.a. Codul admite participarea comunităţii ştiinţifice la elaborareapoliticii de stat în domeniul ştiinţei şi inovării. Şi învăţământul, în special celuniversitar şi postuniversitar, la etapa actuală esteînprocesdereformareînconformitatecu cerinţele Programului de la Bologna, la care recent a aderat R. Moldova.În contextul celor menţionate conducerea SMM îşi propune monitorizarea procesuluide reformare în domeniul matematicii şi al in<strong>format</strong>icii, promovând valorileacceptate de comunitatea matematică internaţională.Comunitatea matematică din Republica Moldova marchează în anul 2006 organizareaediţiei a 50-a a Olimpiadei republicane de matematică pentru elevi. Acesteveniment este deosebit de semnificativ şi graţie rezultatelor Lotului olimpic al RepubliciiMoldova, care a obţinut doar în ultimii cinci ani 66 de medalii la competiţiiinternaţionale, inclusiv trei medalii de aur la olimpiadele internaţionale de matematicăşi nouă medalii de aur la olimpiadele balcanice. În anul 2006 Chişinăul va figazdă a Olimpiadei Balcanice de Matematică pentru juniori.Evoluţia ştiinţei şi a învăţământului în Basarabia după 1990 a fost puternic influenţatăde deschiderea culturală către România. Un loc aparte îl ocupă contribuţiadezinteresată a României în oferirea de burse pentru liceeni, studenţi, masteranzi şidoctoranzi din R. Moldova. Participarea largă laconferinţe ştiinţifice, inclusiv organizareaunora din ele în comun, a extins şi a concretizat cadrul de colaborare, avânddrept urmare susţineri de teze de doctorat, schimb de profesori invitaţi, pregătirealotului olimpic, tabere de vară, editare de carte şi multe alte activităţi deosebit debenefice pentru cultura, cercetarea şi educaţia din Basarabia.Florin DAMIAN, Parascovia SÂRBUFac. de Mat. şi Inf., Univ. de Stat din Moldova125


Concursul de Matematică “Al. Myller”EdiţiaaIV-a,IaşiClasele IV-VI, aprilie 2006Notă. Toate subiectele sunt obligatorii. Timp efectiv de lucru: 90 min. Se acordă dinoficiu 30 de puncte, câte 6 puncte pentru problemele 1-5, câte 8 puncte pentru problemele6-10 şi câte 10 puncte pentru problemele 11-15.Clasa a IV-a1. Ce număr trebuie scăzut din 9 pentru ca diferenţa obţinută înmulţită cu8 sădevină 40?2. Într-o clasă suntbăieţi şi fete. Numărul băieţilor este cu 3 mai mare decâtnumărul fetelor. Dacă înclasăarveni4 băieţi şi ar pleca 4 fete, atunci numărulbăieţilor ar fi de 2 ori mai mare decât numărul fetelor. Câţi elevi sunt în clasă?3. Se dau numerele: a = 1+2+3+··· +9; b = 11+22+33+··· +99;c = 111 + 222 + 333 + ···+ 999. Să se calculeze câtul împărţirii lui a + b + c la 123.4. Suma dintre un număr şi succesorul său este cu 2006 mai mare decât predecesorulsău. Care este numărul?5. Fie pătratul magic (sumele elementelor de pe linii, coloaneşi diagonale sunt egale). Aflaţi ce număr trebuie înscris în căsuţamarcată cuX?X10 129 76. Împărţind numărul natural a la numărul natural b, obţinem câtul 6 şi restul31. Săseaflea ştiind că a − b


14. M-am născut în secolul XX. Dacă în1999 am avut o vârstă egală cusumacifrelor anului meu de naştere, ce vârstă am acum?15. Sony cară în fiecare zi câte o piatră din vârful muntelui. În prima zi a petrecut7 ore urcând şi coborând, a doua zi a petrecut 8 ore urcând şi coborând. În fiecare ziurcă dedouă ori mai încet decât în ziua precedentă, dar coboară de2 ore mai repede.Cât timp va munci în cea de-a treia zi?Clasa a V-a1. Care este cel mai mare număr natural impar de 4 cifre distincte?2. Determinaţi x ∈ N pentru care 5 · 3 x − 2 · 3 x =81.3. Câţi termeni are suma 26 + 32 + 38 + ···+ 2006?4. Câte pătrate perfecte conţine mulţimea {x ∈ N | x ≤ 1000}?5. Câte mulţimi X verifică {1, 2} ∪ X = {1, 2, 3, 4, 5}?6. Câte numere xy scrise în baza 10 verifică relaţia x + xy = y + yx +50?7. Dacă a · b + a · c = b · c + c 2 , b + c = 1003, calculaţi a +2b + c.8. Să se determine un număr de 4 cifre al cărui produs cu 9 se termină cu3755.9. Dacă restul împărţirii lui 2a +3b +5c la 13 este 7, aflaţi restul împărţirii lui34a +51b +85c la 13.10. Câte numere care se scriu cu 4 cifre în baza 10 au suma cifrelor 3?11. Se consideră 5 numere naturale distincte având suma 60. Găsiţi numereledacă suma diferenţelor dintre cel mai mare şi fiecare dintre celelalte 4 numere este10.12. Care sunt ultimele două cifrealelui5 +7 2006 ?n a13. Fie M =b | a, b ∈ N∗ ,a+ b = 2006o. CareesteprodusulelementelorluiM?14. Suma a două numere naturale este 3n +5, n ∈ N ∗ . Împărţind un număr ladublul celuilalt obţinem câtul n şi restul 0. Care sunt numerele?15. Se scriu în ordine crescătoare toate numerele nenule din baza 10 în a cărorscriere nu apar alte cifre afară de0, 1, 2 şi 3. Careestecelde-al2006-lea număr?Clasa a VI-a1. Care este cel mai mic număr natural de nouă cifre divizibil cu trei?2. Fie a, b, c, d măsurile a 4 unghiuri <strong>format</strong>e în jurul unui punct. Să seafleaceste măsuri dacă a 1 = b 2 = c 3 = d 4 .3. Preţul unui obiect se micşorează cu20%. Cu cât la sută trebuie să semăreascănoul preţ pentru a se ajunge la preţul iniţial?4. Calculaţi măsura unui unghi ştiind că triplul complementului său este cu 42 ◦30 0 mai mare decât jumătatea suplementului său.127


5. Care este cea mai mare valoare pe care o poate lua numărul n = a 1 −a 2 + a 3 −a 4 + ···+ a 9 − a 10 , unde a 1 ,a 2 ,...,a 10 sunt numere naturale distincte din mulţimea{1, 2, 3,...10}?6. Fie A, B, C, D puncte pe o dreaptă, în această ordine, astfel încât ACBD =1,ADBD = 7 BC.Calculaţi5 AD .7. Se dau două vasecuapă astfel încât dacă turnăm jumătate din primul în aldoilea şi jumătate din cantitatea de apă ceseaflăacumînaldoileaoturnăm înprimul şi luând apoi jumătate din cantitatea aflată înprimulşi o turnăm în al doileaobţinem (în al doilea vas) 10 litri apă. Aflaţi câţi litri de apă seaflă în fiecare vas,ştiind că ambelecantităţi sunt numere întregi.8. Arătaţi că numărul a =3+3 2 +3 3 + ···+3 4n se divide cu 120, ∀n ∈ N ∗ .9. Să se calculeze a 2007-ea zecimalăanumărului n =0, 00 (300)+0, 00 (3000)+1.10. Să seaflex, ştiind căµ µ µ1 2 32 %din 3 %din 4 %din ...µ nn +1 %din x ... =1100 n (n +1) .222 ...2311. Fie n un număr natural, n>2. Numărătorii şi numitorii fracţiilor333 ...34555 ...56şi au câte n cifre. Care este fracţia mai mare?666 ...6712. Câte unghiuri cu măsurile numere naturale consecutive se pot forma în jurulunui punct?13. Măsurile unghiurilor unui triunghi sunt direct proporţionale cu trei numereinvers proporţionale cu trei numere direct proporţionale cu trei numere puteri consecutiveale lui 3. Găsiţi măsurile unghiurilor.14. Fie S =1+2+3+···+10 2006 . Laceputereapare2 în descompunerea înfactori primi a lui S?15. Să se afle cardinalul mulţimii A =n! =1· 2 · 3 · ··· ·n.½¾2006!(x, y) ∈ N × N |7 x · 11 y ∈ N , undeJunioriClasele VII-XII, 19 aprilie 20061. Să searatecăecuaţia 1 a + 1 b + 1 ab = 1 c are o infinitate de soluţii în (N∗ ) 3 .2. Fie 4ABC dreptunghic în C şi D ∈ (BC), E ∈ (CA) astfel încât BDAC =AECD = k. DrepteleBE şi AD se intersectează înO. Săsearatecă m( \BOD) =60 ◦dacă şi numai dacă k = √ 3.128


3. Se dau în plan 5 puncte cu proprietatea că oricare dintre cele 10 triunghiuricu vârfurile în aceste puncte are aria cel mult egală cu1. Săsearatecăexistăuntrapez de arie cel mult 3 ce conţine în interior sau pe laturi cele 5 puncte.Seniori1. Fie p ≥ 3 număr prim. Demonstraţi că p−1 Pk=1k p − kp≡ p +12(mod p).2. Fie p ≥ 5 număr prim. Determinaţi numărul polinoamelor de forma X p +pX k + pX l +1, k>l, k, l ∈ {1, 2,...p− 1} care sunt ireductibile în Z [X].3. Mediana AM a triunghiului ABC taie cercul înscris în K şi L. Dreptele duseprin K şi L paralele la BC taie a doua oară cercul înscris în X şi Y . Fie P , Qintersecţiile dreptelor AX şi AY cu BC. Arătaţi că BP = CQ.8 11 ARITMOGRIF 7 123 6 1 21 6 1?9 2 6 12 14 9 4 15 9 210 5 3 2 9 12 8 7 18 19 8 12 4 6 1 2 13 19A 1 2 3 1 2 4 5 6 6 * 7 4 5 2 7 4 5 6 3 2 B6 1 8 12 13 1 15 10 10 6 2 3 6 1713 8 12 15 13 2 8 1 19 15 13 172 16 17 8 8 7 15 20Înlocuind numerele cu litere veţi obţine numele a 18 matematicieni.Pe orizontală A-B: numele unei reviste de matematică pentruelevişi profesori.Prof. Valeriu BraşoveanuC. N. "Gh. Roşca Codreanu", Bârlad(Rezolvarea aritmogrifului se găseşte la pag. 162)Vizitaţi pe Internet revista "Recreaţii Matematice" la adresahttp://www.recreatiimatematice.uv.ro129


Concursul de matematică “Florica T. Câmpan”Etapa judeţeană, 18 februarie 2006Clasa a IV-a1. Să seîmpartă la trei persoane 24 sticle de suc identice ca mărime, din care 5sunt pline, 11 umplute pe jumătate şi 8 goale, încât fiecare să aibă acelaşi număr desticle, dar şi aceeaşi cantitate de suc.Înv. Lina Huzum2. Între cele 9 numere de mai jos există un "intrus". Acesta nu respectă relaţiadintrecifreceexistă la fiecare din celelalte opt numere. Descoperă şi scrie relaţia,precum şi numărul "intrus": 9334, 4862, 6148, 5132, 7835, 3524, 9963, 9782, 8133.Înv. Fănică Dragnea3. Cântarul pe care vor să secântărească treicopiinumăsoară mase mai mici de40 kg. Fiecare din cei trei copii cântăresc între 25 şi 30 kg.Cum a reuşit fiecare copil să secântărească?Inst. Iulian CristeaClasa a V-a1. Cifrele care alcătuiesc vârsta bunicului reprezintă vârstele celor doi nepoţi.Dacă împărţim vârsta bunicului la suma vârstelor nepoţilor, se obţine câtul 4 şirestul 12. Aflaţi vârsta bunicului şi vârstele nepoţilor.Mihaela Cianga2. Se consideră înmulţirea următoare, unde literele nu reprezintăobligatoriu cifre distincte:a) Determinaţi e.b) Arătaţi că bd =63.c) Reconstituiţi înmulţirea.Gabriel Popaa 3 bcdef3 ghik2 np33. Se consideră mulţimea A = {2, 3, 4,...,13}.a) Determinaţi B, C disjuncte astfel încât B ∪ C = A şi suma elementelor din Beste egală cusumăelementelordinC.b) Arătaţi că nuexistă M, N disjuncte cu M ∪ N = A şi produsul elementelordin M egal cu produsul elementelor din N.c) Găsiţi două mulţimi X, Y disjuncte cu X ∪ Y = A, X având două elemente,cu produsul elementelor lui X egal cu suma elementelor lui Y .Ionel NechiforClasa a VI-a1. Pe o tablă s-au scris trei numere naturale. Când în locul lor s-au scris: suma,produsul şi suma produselor câte două, s-a văzut că petablăauapărut aceleaşinumere ca şi cele iniţiale. Care este produsul lor? Explicaţi!2. Pe o tablă estescrisnumărul 12. La fiecare minut numărul se înmulţeşte sause împarte fără rest fie la 2, fie la 3, iar rezultatul se scrie pe tablă înloculnumăruluiiniţial. Să searatecănumărul scris pe tablă după exactooră nu poate fi 54.3. Unghiurile proprii \AOB şi \BOC sunt adiacente suplementare. Fie [Ox şi [Oybisectoarele acestora. Dacă m([BOy) ∈ N ∗ şi m( [COx) =p · m([BOy), unde p este130


număr prim, aflaţi numărul p.Vasile NechitaClasa a VII-a1. Fie a un număr natural arbitrar, divizibil prin 9, având2007 cifre. Notăm cus (a) numărul care reprezintă sumacifrelorluia. Găsiţi s (s (s (a))).Gabriel Mîrşanu2. Vârfurile unui cub se notează cu8 numere întregi consecutive, iar centrulfiecărei feţe se notează cumediaaritmeticăavârfurilorfeţei respective.a) Să se demonstreze că suma centrelor oricăror două feţeopuseesteaceeaşi şi săse afle cât este aceasta în funcţie de cel mai mic dintre numerele din vârfuri.b) Să segăsească încecondiţii centrul unei feţe este număr întreg. Să sescrievârfurile feţelor ale căror centre sunt numere întregi.c) Arătaţi că, dacă centrele a trei feţe care au un vârf comun sunt notate cunumere întregi, atunci şi centrele celorlaltor feţe sunt notate tot cu numere întregi.Julieta Grigoraş3. În ţara TI a triunghiurilor isoscele era împărat, firesc, triunghiul echilateral.El decretase că este singurul care binemerită numeledePrearostogolibil; supuşii săitrebuiau să fienumiţi ţepoşi dacă auolaturămaiscurtă decât cele egale, respectivturtiţi dacă auolaturămailungă decât cele egale. (Vorba congruent era socotită deocară pe acele meleaguri.) Nişte unghiuri umblau venetice prin TI căutând fiecaretriunghi isoscel la al cărui vârf să slujească.— Ţeposule, zise un unghi α. Eu şi vecinii mei de pribegie bâlbâitul de β şinemăsuratul de γ ne căutăm stăpâni în TI. Ne-ai fi de mare folos dacă aibinevoisăne spui dacă nu cumva ai o bisectoare interioară ataexactatâtdelungăcaolatură.—După vorbire se cunoaşte că veniţi de pe coclauri unde lucrurile nu sunt făcutedin linii drepte bine limitate. Întrebi de lucruri la care nu gândeşte nimeni fiindcănu sunt de niciun folos. Dar, până cercetez pentru răspuns, fii bun măi crăcănatuleşi spune-mi dacă aşa se obişnuieşte pe la voi: să-ţi ponegreşti colegii cu vorbenecuviincioase?— Nu e necuviinţă, prea-limitatule. Eu, α, mă exprim frumos în grade, de aceeasunt purtător de cuvânt; β nu cunoaşte fracţii ordinare ci doar zecimale şi se bâlbâiegrozav când încearcă să spună câte grade are; γ încă nuştie dacă estemăsurabil îngrade. Dar bag seamă căîntârziicurăspunsul la întrebarea mea; o fi capul tău maimult ascuţitdecâtîncăpător?—Bine,măi vorbăreţule. Am cercetat şi răspund precis: am exact două bisectoareinterioare exact aşa de lungi ca laturile mele egale.—Amînţeles. Te rog să măierţi că ţi-am zis ţepos; înţeleg că eşti turtit. Miepersonal nu îmi eşti de folos, dar iată căpentruβ eşti bun de stăpân. Dacă bisectoareletale egale erau cât latura ta scurtă, te recunoşteam de stăpân. Dacă osingurăbisectoare a ta era cât latura ta scurtă, te-ar fi slujit γ cu credinţă.a) Exprimaţi cu fracţii ordinare gradele lui α şi β.b) Exprimaţi cu fracţii zecimale numărul de grade, minute şi secunde ale lui β.c) Argumentaţi că triunghiurile isoscele care convin lui α, respectiv β, suntţepoase,respectivturtite.131


d) Desenaţi un triunghi isoscel cu unghiurile de la vârf γ. Esteelţepos sau turtit?Dan BrânzeiClasa a VIII-a1. Într-o clasă sunt20deelevi. Fiecarefatăoferăfiecărui băiat trei flori şi fiecareifete o floare, iar fiecare băiat oferă câte trei flori fiecărei fete şi câte o floare fiecăruibăiat.a) Arătaţi că numărul maxim de flori oferite este 780.b) Câte fete ar trebui să fieînclasă, astfel încât să fie oferite exact 780 de flori?Monica Nedelcu, Iaşi2. Pe fiecare faţă a unui cub este scris câte un numărnaturalnenul,iarfiecăruivârf îi corespunde produsul numerelor de pe cele trei feţe adiacente acestuia. Dacăsuma numerelor corespunzătoare tuturor vârfurilor este 2006, arătaţi că există celpuţin două feţe pe care este scris acelaşi număr.3. Fie paralelipipedul dreptunghic ABCDMNP Q şi punctele E ∈ (BN), F ∈(DQ) astfel încât suma AE + AF + PE + PF este minimă. Arătaţi că EF ⊥ APdacă şi numai dacă ABCDMNP Q este prismă regulată.Valentina Blendea şi Gheorghe Blendea, IaşiFaza interjudeţeană, 20 mai 2005Clasa a IV-a1. Lungimea laturii unui pătrat este de 17 m. O persoană pleacădintr-unvârfalpătratului şi, mergând în acelaşi sens pe laturile acestuia, parcurge o distanţă de 637m. Din punctul în care a ajuns se întoarce şi parcurge 773 m. Aflaţi la ce distanţăse va situa în final persoana, faţă de punctul de plecare.RecMat - 1/2005, P.902. Avem mai multe vase identice. Ştim că 50vaseplinecuapăcântăresc 600 kg,iar 10 vase goale cântăresc cu un kilogram mai puţin decât apa dintr-un vas plin. Săse afle cât cântăresc 100 de vase goale.Petru Asaftei3. În câte moduri diferite pot fi scrise numerele 1, 2, 3, 4 în pătratele dinfigura alăturată, câte unul în fiecare pătrat, astfel încât să nuexistedouăpătrate alăturate în care suma să fie 5? Justificaţi. (Pătratele care au doarunvârfcomunnusuntconsideratealăturate, iar pătratul mare este fix.)Petru AsafteiClasa a V-a1. Din produsul tuturor numerelor naturale de la 1 la 2006 se exclud toatenumerele care se divid cu 5. În ce cifră setermină produsul celorlaltor numere?2. Pe o tablă sunt scrise numerele 1,3,4,6,8,9,11,12,16. Doi copii au şters câtepatru numere şi s-a observat că suma numerelor şterse de unul este de trei ori maimare decât suma numerelor şterse de celălalt. Ce număr a rămas scris pe tablă?3. Fie numărul 123456789. O operaţie înseamnă să alegem două cifrealăturatecărora să lisescadăounitateşi să li se schimbe locurile (de exemplu: 123456789 →123436789 → ...). Care este cel mai mic număr ce se poate obţine ca rezultat alacestor operaţii? După câteoperaţii se obţine cel mai mic număr?132


Clasa a VI-a1. Se pot aşeza pe muchiile unui cub numerele 1, 2, 3,...,12 (câte un număr pefiecare muchie) astfel încât suma numerelor aflate pe cele trei muchii care pleacă dinacelaşi vârf să fie aceeaşi pentru fiecare vârf al cubului?Constantin Chirilă2. Se consideră triunghiul ABC cu [AB] ≡ [AC] şi m(\BAC) = 20 ◦ . FieM,P ∈ (AC) astfel încât şi m( \ABM) =m(\CBP) =20 ◦ şi N ∈ (AB) astfel încâtm(\BCN) =50 ◦ .Săseaflem( \AMN).3. Numerele naturale 22, 23, 24 au următoarea proprietate: descompunerile în factoriprimi ale numerelor din şir au exponenţii factorilor numere impare (22 = 2 1 · 11 1 ,23 = 23 1 , 24 = 2 3 · 3 1 ). Care este cel mai mare număr de numere naturale consecutivecare au această proprietate?Cristian - Cătălin BudeanuClasa a VII-a1. a) Arătaţi că din oricare 3 numere naturale, putem alege două astfelîncâtsuma lor să fieunnumăr par.b) Fiind date şapte numere naturale, arătaţi că putem alege patru dintre ele astfelîncât suma lor să fie divizibilă cu4.Cristian Lazăr2. Fie ∆ABC isoscel cu BC =2a, AB = AC = b, a, b ∈ N ∗ . Să se determinetoate triunghiurile ABC, dacă a =2r, unde r este raza cercului înscris în ∆ABC, şiapoi găsiţi ∆ABC cu aria minimă.Dan Brânzei3. Un trapez ABCD are baza mare [AB] şi [AC] ∩ [BD] ={O}. Linia mijlociea trapezului intersectează peAC în E şi pe BD în F .a) Demonstraţi că ABCD este trapez isoscel dacă şi numai dacă [OE] ≡ [OF].b) Vârfurile trapezului şi punctul O reprezintă 5 oraşe, iar laturile şi diagonalelesale sunt şosele de legătură. Două maşini pleacă dinD, respectiv C pe ruta cea maiscurtă spreA, respectiv spre B şi alte două maşini pleacă dinA respectiv B spre D,respectiv C, trecândprinO pe ruta cea mai scurtă. Cele 4 maşini au aceeaşi viteză,constantă, pe întreg parcursul. Demonstraţi că primele2 maşini ajung simultan înD, respectivC. Pot ajunge, toate patru, în acelaşi timp la destinaţie?Claudiu-Ştefan PopaClasa a VIII-a1. Aflaţi perechile de numere întregi cu proprietatea că diferenţa cuburilor loreste egală cupătratul diferenţei lor.2. Un poliedru are 17 muchii şi 9 feţe.a) Desenaţi două astfel de poliedre diferite.b) Dacă feţele poliedrului sunt doar triunghiuri echilaterale sau pătrate, iar toatemuchiile sunt de lungime 1 cm, calculaţi aria sa totală.Gabriel Popa3. Un cub cu latura de n cm (n ∈ N ∗ ) se împarte în cuburi cu latura de 1 cmşi se colorează toate cuburile situate pe diagonalele feţelor cubului iniţial. Aflaţi nastfel ca numărul cuburilor colorate să fie2006.Julieta Grigoraş133


Soluţiile problemelor propuse în nr. 2 / 2005Clasele primareP.94. Aflaţi numerele de două cifre cu proprietatea că diferenţa dintre număr şirăsturnatul lui este egală cucelmaimarenumăr scris cu o singură cifră.(Clasa I )Mara Neicu, elevă, HârlăuSoluţie. Dacă unnumăr de două cifre îndeplineşte condiţia din problemă, atuncidiferenţa dintre numărul reprezentat de cifra zecilor şi numărul reprezentat de cifraunităţilor este egală cu1. Numerele care verifică cerinţa sunt: 21, 32, 43, 54, 65, 76,87, 98.P.95. La ora de educaţie fizică, fetele unei clase sunt aliniate de la cea mai înaltăla cea mai scundă, iar băieţii după ele, în aceeaşi ordine. Ana este cea mai înaltă.Eneestecelmaiînalt,iarSorincelmaiscund.ÎntreAnaşi al doilea băiat sunt 15copii, iar între Ene şi Sorin sunt 10 băieţi. Câţi elevi sunt aliniaţi la educaţia fizică?(Clasa I )Înv. Constanţa Cristea şi Inst. Iulian Cristea, IaşiSoluţie. Numărul fetelor este 15−1+1 = 15,iarnumărul băieţilor este 10+1+1 =12. La ora de Educaţie fizică sunt aliniaţi 27 elevi.P.96. Descoperă regulaşi completează căsuţele libere în cazurile:2 5 6 10 5 7 1 3a);3 8 4 14 2 6 13 49 7 8 7 3 3 5 3b)2 5 1 6 01 2 0 .(ClasaaII-a)Inst. Maria Racu, Iaşia bSoluţie. a) Dacă tabelul este de forma , atunci regula este dată dec da + c = b şi b + c = d. În tabelul al treilea completăm căsuţa liberă cu2+7=9,înal patrulea cu 13 − 6=7, iar în ultimul tabel cu 3+4=7, respectiv 4+7=11.b) În acest caz regula este dată dea − c = b şi b − c = d. Tabelele se completeazăcu 3 − 0=3; 5 − 3=2; b =0+2, a = b +2=2+2=4.P.97. În câte moduri putem forma un şir indian compus din 4 băieţi şi 3 feteastfel încât două fetesănusteaunalângăalta,iarşirul să nu înceapă cuofată?(ClasaaII-a)Andrei Burdun, elev, IaşiSoluţie. Şirul poate începe cu cel mult doi băieţi. Avem posibilităţile: bfbfbfb;bfbbfbf; bfbfbbf; bbfbfbf. Şirul poate fi <strong>format</strong> în 4 moduri.P.98. La Crăciun, copii au împodobit bradul cu 35 globuri albe, galbene şi roşii.Andrei a observat că, dacă împarte numărul globurilor albe la cele galbene obţinecâtul 3 şi restul 2, iardacă împarte numărul globurilor roşii la cele galbene obţinecâtul 2 şi restul 3. Câte globuri de fiecare fel au împodobit bradul?(Clasa a III-a)Înv. Rica Bucătaru, IaşiSoluţie. Folosim metoda figurativă.134


abс2Numărul globurilor galbene este[35 − (2+3)]:6=5.353Numărul globurilor albe este 5 × 3+2=17.Numărul globurilor roşii este 5 × 2+3=13.P.99. La concursul de alergare organizat de clasa a II-a, cei mai buni băieţi suntRadu, Cezar, Tudor, Dan şi Mihai. Care a fost clasamentul final, dacă: 1) Radunu a luat locul întâi, 2) Cezar s-a clasat în urma lui Mihai, 3) Radu s-a clasatînaintea lui Mihai, 4) Tudor nu s-a clasat al doilea, 5) Dan este al treilea dupăTudor.(Clasa a III-a)Înv. Constanţa Cristea şi Inst. Iulian Cristea, IaşiSoluţie. Deoarece Tudor nu s-a clasat al doilea şi Dan este al treilea după Tudor,deducem că Tudor s-a clasat pe locul I. Ţinând cont şi de ordinea R—M—C, rezultăclasamentul final TRMDC.P.100. Trei numere naturale au suma 60. Săseaflenumereleştiind că împărţindu-lpe primul la al doilea obţinem câtul 4 şi restul 3, iar al treilea este cu 1 maimare decât dublul celui de-al doilea.(Clasa a III-a)Vasile Solcanu, Bogdăneşti, SuceavaSoluţie. Utilizăm metoda figurativă.3 Al doilea număr esteab =[60− (3 + 1)] : 7 = 8.b60 Primul număr este a =4×8+3 = 32+3 = 35.1сAl treilea număr este c =2× 8+1=17.P.101. Există numere naturale care împărţite la 12 să dea câtul 7, iarîmpărţitela 15 să dearestul2?(ClasaaIV-a)Alexandru-Gabriel Tudorache, elev, IaşiSoluţie. Fie n =12k +7 = 15p +2. Atunci n =3(4k +2)+1 = 3· 5p +2,adică pedeoparten dă laîmpărţirea prin 3 restul 1, pedealtăpartedărestul2,imposibil.P.102. Să searatecăpătratul de latură 36 poate fi acoperit cu piese de forma11 .(ClasaaIV-a)Andrei Burdun, elev, IaşiSoluţie. Prin îmbinarea a două piese putem obţine o piesă dreptunghiularăde forma alăturată. Pentru acoperirea pătratului suntnecesare (36 : 4) × (36 : 2) = 162 piese dreptunghiulare. Pătratulpoate fi acoperit cu 162 × 2 piese iniţiale.P.103. Un dreptunghi are perimetrul de 624 cm, iar lungimea este dublul lăţimii.Poate fi împărţit dreptunghiul într-o reţea de pătrate egale astfel încât suma perimetrelorlor să fie 6656 cm?(ClasaaIV-a)Petru Asaftei, IaşiSoluţie. Lăţimea dreptunghiului este 624:6=104cm. Dacă împărţim dreptunghiulîn n pătrate egale, atunci n poate fi 2, 8, 32, 128, 512, . . . . Suma perimetreloreste 4nl = 6656 cm, de unde nl = 1664 cm. Pentru n =2obţinem l = 832 cm135111


104 cm, fals. Pentru n =8obţinem l = 208 cm > 104 cm, fals. Pentru n =32obţinem l =52cm, ceea ce înseamnă călăţimea poate fi acoperită cudedouăorilatura unui pătrat, ceea ce este fals, deoarece dreptunghiul trebuie acoperit cu 32pătrate. Pentru n = 128 obţinem l =13cm. Atuncidreptunghiulpoatefiacoperitcu (104 : 13) × (208 : 13) = 8 × 16 = 128 pătrate. Pentru n>128 nu avem soluţii.Clasa a V-aV.61. Determinaţi x, y, z ∈ N în fiecare din cazurile:a) x · y = 252z +1 ; b) x2 + y 2 = 252z +1 ;Vasile Solcanu, Bogdăneşti, SuceavaSoluţie. a) Deoarece xy ∈ N, atunci2z +1 | 25, adică 2z +1 ∈ {1, 5, 25}, prin urmarez ∈ {0, 2, 12}. Dacă z =0,atuncixy =25, deci (x, y) ∈ {(1, 25) ; (5, 5) ; (25, 1)}.Dacă z =2,atuncixy =5,deci(x, y) ∈ {(1, 5) ; (5, 1)}. Dacă z =12,atuncixy =1,deci (x, y) =(1, 1). Înconcluzie,(x, y, z) ∈ {(1, 25, 0) ; (5, 5, 0) ; (25, 1, 0) ; (1, 5, 2) ; (5, 1, 2) ; (1, 1, 12)} .b) Obţinem tot z ∈ {0, 2, 12} şi, analizând toate cazurile, găsim că(x, y, z) ∈ {(0, 5, 0) ; (5, 0, 0) ; (3, 4, 0) ; (4, 3, 0) ; (1, 2, 2) ; (2, 1, 2) ; (0, 1, 12) ; (1, 0, 12)} .Să notăm că a) şi b) nu pot avea loc simultan.V.62. Să sescrienumărul 12321 ca diferenţă adouăpătrate perfecte.Andrei-Sorin Cozma, elev, IaşiSoluţie. Observăm că 12321 = 3 2 · 37 2 . Înmulţim relaţia 3 2 +4 2 =5 2 cu 37 2 şiobţinem că 3 2 · 37 2 +4 2 · 37 2 =5 2 · 37 2 , de unde 3 2 · 37 2 =5 2 · 37 2 − 4 2 · 37 2 , deci12321 = 34225 − 21904.V.63. Să se scrie în ordine crescătoare numerele 3 33 ; 3 33 ; 333; 33 3 ; ¡ 3 3¢ 3.Ion Vişan, CraiovaSoluţie. Observăm că3 33 > 3 27 =3 33 = ¡ 3 9¢ 3=81 3 > 33 3 > 27 3 = ¡ 3 3¢ 3 ¡> 32 ¢ 3=9 3 =729> 333,deci 333 < ¡ 3 3¢ 3< 33 3 < 3 33 < 3 33 .V.64. Să se determine x, y, z ∈ N, z 6= 0,încât5 x +5 y = z!, undez! =1·2·3·...·z.Doru Turbatu, IaşiSoluţie. Notăm U 2 (A) ultimele două cifrealenumărului A. Dacă x ≥ 2, y ≥ 2,atunci U 2 (5 x +5 y )=50,întimpceU 2 (z!) 6= 50, ∀z ∈ N. Dacă x =1, y ≥ 2, atunciU 2 (5 x +5 y )=30,întimpceU 2 (z!) 6= 30, ∀z ∈ N. Analog se aratăcănuconvinecazul x ≥ 2, y =1. Rămân de studiat cazurile (x, y) ∈ {(0, 0) , (1, 0) , (0, 1) , (1, 1)}.Analizându-le pe rând, obţinem soluţiile (x, y, z) ∈ {(0, 0, 2) , (1, 0, 3) , (0, 1, 3)}.V.65. Vom numi "număr p" un număr natural care are exact 4 divizori.a) Daţi exemplu de trei numere p consecutive.b) Să searatecănuexistătreinumere p consecutive astfel încât primul dintre elesă fie par.Ovidiu Pop, Satu MareSoluţie. a) De exemplu, putem considera numerele 33, 34, 35.136


) Presupunem prin absurd că există a =2k, b =2k +1, c =2k +2=2(k +1),k ∈ N, treinumere p consecutive. Cum 2k şi 2(k +1) au fiecare exact 4 divizori,numerele k şi k +1 trebuie să fie ambele prime. Singurele numere prime consecutivesunt 2 şi 3, deci k =2,apoia =4, b =5, c =6.Evidentînsăcă 5 nu este număr p.Clasa a VI-aVI.61 Dacă a = x 9 y 3 z 4 t 10 , b = xy 5 z 6 t 8 , c = x 5 y 7 z 10 t 12 şi |a| + a =0,săsearate că |b| + b = |c| + c =0.Cristian - Cătălin Budeanu, IaşiSoluţie. Evident că |a| + a =0⇔ a ≤ 0. Deoarece a = xy ¡ x 4 yz 2 t 5¢ 2≤ 0,rezultă că xy ≤ 0. Atuncib = xy ¡ y 2 z 3 t 4¢ 2 ¡≤ 0, c = xy x 2 y 3 z 5 t 6¢ ≤ 0,prin urmare |b| + b = |c| + c =0.VI.62. Fie a 1 ,a 2 ,...,a n ∈ {−1, 1} astfel încâta 1 a 2 a 3+ a 2a 3 a 4+ ···+ a n−1a n a 1+ a na 1 a 2=0.a 4 a 5 a 2 a 3Să searatecă n se divide cu 4.Ioana Olan, elevă, IaşiSoluţie. Suma dată aren termeni, fiecare dintre ei fiind +1 sau −1. Rezultatulfiind 0, numărul termenilor egali cu +1 este acelaşi cu al celor egali cu −1, prinurmare n =2k. Produsulcelorn fracţii este, pe de o parte, (−1) k (+1) k =(−1) k ,pede alta este a 2 1 a2 2 ···a2 n =+1; rezultă k =2l, l ∈ N. Înconcluzie,n =4l, deci n . 4.Să remarcăm că pentrun =4nu există numerea 1 , a 2 , a 3 , a 4 care să verificeipotezele problemei, însă pentrun =8putem considera numerele −1, 1, 1, 1, 1, 1,1, 1.VI.63. Se poate completa un pătrat 10 × 10 cu numerele de la 1 la 100, aşa încâtpentru fiecare coloană să se poată forma (cu numerele din acea coloană) trei grupe,astfel ca sumele numerelor din fiecare grupă să fie egale?Bogdan Andrei Ciacoi, elev, Gherla100 · 101Soluţie. Suma tuturor numerelor din pătrat este = 5050. Dacăs-ar2puteacompletapătratul în acest fel, atunci datorită faptuluică pe fiecare coloanăsuma numerelor este multiplu de 3, suma numerelor de la 1 la 100 ar fi multiplu de 3.Cum 5050 nu este multiplu de 3, rezultă căpătratul nu se poate completa în moduldorit.VI.64. Pentru fiecare n ∈ N,n ≥ 4, arătaţi că interiorul oricărui patrulaterconvex se poate descompune în reuniune de n triunghiuri dreptunghice cu interioareledisjuncte.Gabriel Popa, IaşiSoluţie. Observăm întâi că interiorul oricărui triunghi poate fi împărţit în douătriunghiuri dreptunghice, ducând înălţimea din vârful unghiului cel mai mare (precauţienecesară dacă triunghiul iniţial este obtuzunghic). Dacă n =4, o diagonalăîmparte interiorul patrulaterului în două triunghiuri, iar după procedeul de mai sus137


obţinem 4 triunghiuri dreptunghice. Dacă se poate face o descompunere în k triunghiuridreptunghice, putem realiza descompunerea în k +1 triunghiuri, ducândînălţimea ipotenuzei în unul din primele k şi cu aceasta rezolvarea este încheiată.VI.65. Fie 4ABC, DE k BC, cuE ∈ (AC) şi D ∈ (AB), iarF ∈ (BD)şi {G} = FE ∩ DC. Demonstraţi că, dacă două dintre următoarele afirmaţii suntadevărate, atunci este adevărată şi a treia:(i) BD =2BF; (ii) AC =2CE; (iii) EF =2FG.Claudiu - Ştefan Popa, IaşiSoluţie. Fie H mijlocul lui [CE], iar{L} = BE ∩AFH, {K} = CD ∩ FH.(i), (ii) ⇒ (iii). Folosim în mod repetat teoremeleliniei mijlocii într-un triunghi/ trapez şi reciprocele lor.D ECum AE = EC şi DE k BC, atunciDE = BCG2 .Apoi,F Kdin DF = FB şi EH = HC, obţinem FH k BC,HLdeci [FL], [FK] vor fi linii mijlocii în 4BDE, respectiv4DBC. AtunciFL = DEBC2 = BC , LK = FK − FL =4BC2 − BC = BC , adică FL = LK = KH. În 4FEC, FH este mediană şi4 4FKKH = 2 , deci K va fi centru de greutate; rezultă că CK este mediană, adică1FG = GE.(i), (iii) ⇒ (ii). În 4CEF, FH şi CG sunt mediane, deci K va fi centru degreutate. Rezultă că FK =2KH şi cum FK = BC DE, KH = , atunci BC =2 22 DE. Conform unei a doua reciproce a teoremei liniei mijlocii în triunghi, mai puţincunoscută, rezultă că [DE] este linie mijlocie în 4ABC (altfel [D 0 E 0 ] ar fi acea liniemijlocie şi cum DE = D 0 E 0 = BC2 , DE k D0 E 0 k BC, amobţine că DEE 0 D 0 ar fiparalelogram, adică DD 0 k EE 0 şi atunci AB k AC, fals!).(ii), (iii) ⇒ (i). Presupunem prin absurd că F nu este mijlocul lui [BD]; fie atunciF 0 acest mijloc, iar {G 0 } = F 0 E ∩ DC. Din (ii), EF 0 =2F 0 G 0 ,deci[GG 0 ] este liniemijlocie în 4EFF 0 , prin urmare GG 0 k BD, adică DC k BD, absurd. Rămâne căare loc (i).Clasa a VII-aVII.61. Fie n ∈ N ∗ fixat şi p, k ∈ N, pn + k 2n + k + p .(În legătură cuV.31 din RecMat - 2/2002).Gigel Buth, Satu MareSoluţie. Avem:(n + p)+(n + p +1)+···+(n + k) =(n + k)(n + k +1)2138−(n + p − 1) (n + p)2=


(2n + k + p)(k − p +1)= . (1)2Inegalitatea dintre media armonică şi cea aritmetică aratăcă(n + p)+(n + p +1)+···+(n + k)k − p +1k − p +1>1n+p + 1n+p+1 + ···+ 1 . (2)n+kDin (1) şi (2) obţinem inegalitatea dorită. Problema V.31 se obţine pentru n =100, p =1, k = 100.VII.62. Fie a ∈ R astfel încât a 10 − a 6 + a 2 =4.Săsearatecă 7 2şi a 4 + 1 a 4 > 2. Din a12 +1 = ¡ a 4 +1 ¢¡ a 8 − a 4 +1 ¢ = a4 +1¡ a 10a 2 − a 6 + a 2¢ =4µa 2 + 1 a 2 ,avema 12 +1 > 4 · 2, deci a 12 > 7. Din a 10 + a 2 = a 6 +4,avema 4 + 1 a 4 =1+ 4 a 6 , de unde 1+ 4 a 6 > 2, decia6 < 4, prin urmare a 12 < 16.VII.63. Fie triunghiurile ABC şi A 0 B 0 C 0 cu AB = A 0 B 0 , BC = B 0 C 0 şi bisectoarele[BE], [B 0 E 0 ] congruente. Să searatecă 4ABC ≡ 4A 0 B 0 C 0 .Petru Asaftei, IaşiSoluţie. A se vedea în acest număr nota Criterii de congruenţă a triunghiurilor,pag. 107-109.VII.64. În triunghiul echilateral ABC considerăm cevienele AM, BN şi CPconcurente într-un punct O, interior triunghiului. Arătaţi că, dacă 4MNP esteechilateral, atunci O este centrul 4ABC.Temistocle Bîrsan, IaşiSoluţie. Unghiurile marcate sunt congruente. Întradevăr,m( A)=60 b ◦ implică m(\ANP ) = 120 ◦ −m(\AP N);Am( \NPM)=60 ◦ implică m( \BPM) = 120 ◦ − m(\AP N),deci \ANP ≡ \BPM. Analog arătăm că \BPN ≡ \CMN;Pvom avea şi egalităţile \AP N ≡ \BMP ≡ \CNM. Searatăuşor că 4ANP ≡ 4BPM ≡ 4CMN, de unde AP =ONBM = CN şi AN = BP = CM. Conform teoremei luiCeva, are loc relaţia <strong>MB</strong>B M CMC · NCNA · PAPB =1,carerevineµ 3 <strong>MB</strong>la =1sau <strong>MB</strong> ≡ MC. Rezultăcă M, N, P sunt mijloacele laturilorMCtriunghiului ABC, deciO este centrul acestuia.VII.65. Fie ABCD paralelogram. O dreaptă variabilă ce trece prin A taiedreptele BC şi CD în F , respectiv G şi taie paralela prin C la BD în E. Săse arate că AE 2 ≤ AF · AG.Cătălin Calistru, Iaşi139


Soluţie. Fie {O} = AC ∩ BD, {P } = AE ∩ BD;cum [OP] este linie mijlocie în 4ACE, vomaveacăAE = 2AP . Din asemănările evidente 4AP D ∼4FPB, 4ABF ∼ 4GCF , 4GCF ∼ 4GDA,obţinem respectiv 1AP = BFPF · BC , 1AF = CFGF · BF ,1AG = CF . De aici,GF · BC1AF + 1AG = CF µ 1GF BF + 1 = CFBC GF= 1AP · CF BF + BC ·FG BF· PFBF .B· BF + BCBF · BC ·A1APPOFEG· PF · BCBFFBDin 4DPA ∼ 4BPF obţinemBC + FB = PF BC + BF⇔AP + PF BF1AF + 1AG = 1AP · CFFG · AFPF · PFBF = 1AP = 2AEşi de aici concluzia rezultă imediat, aplicând inegalitatea mediilor.C=D= AFPF , deciClasa a VIII-aVIII.61. Rezolvaţi în numere naturale ecuaţia x! =y 2z +1.Denisa Florică şi Lucian Tuţescu, CraiovaSoluţie. Pentru x ∈ {0, 1}, obţinem y =0, z ∈ N. Pentru x =2,găsim y =1,z ∈ N sau y ∈ N, z =0.Pentrux ≥ 3 nu mai avem soluţii, deoarece x! se divide cu3, pecândy 2z +1nu (dacă y =3k, atunciy 2z +1=M3+1,iardacă y =3k ± 1,atunci y 2 = M3+1,deciy 2z +1=M3+2).VIII.62. If a, b, c > 0 and √ a + √ b + √ c ≥ 3, provethat2(a + b + c) 2 ≥ 3(ab + bc + ca)+3(a + b + c) .Babis Stergiou, Chalkida, GreeceSoluţie. Inegalitatea de demonstrat revine la2 ¡ a 2 + b 2 + c 2¢ +(ab + bc + ca) ≥ 3(a + b + c) . (1)Condiţia din ipoteză şi inegalitatea Cauchy - Schwarz arată că3(a + b + c) ≥³ √a +√b +√ c´2⇒ a + b + c ≥ 3.Atunci:2 ¡ a 2 + b 2 + c 2¢ +(ab + bc + ca) ≥ a 2 + b 2 + c 2 +2(ab + bc + ca) ==(a + b + c) 2 =(a + b + c)(a + b + c) ≥ 3(a + b + c) ,deci (1) este adevărată. Egalitatea se atinge pentru a = b = c =1.VIII.63. Let a, b be distinct nonzero real numbers. Find all solutions (x, y) ∈ R 2of the equation (x − 1) 2 + y 2 +4aba 2 (x − 1) y =0.+ b2 José Luis Díaz-Barrero, Barcelona, Spain140


Soluţie. Ecuaţia devine succesiv:¡a 2 + b 2¢£¡ x − 1 2¢ + y 2¤ +4ab (x − 1) y =0⇔¡a 2 x 2 + b 2 y 2 + a 2 +2abxy − 2a 2 x − 2aby ¢ ++ ¡ b 2 x 2 + a 2 y 2 + b 2 +2abxy − 2b 2 x − 2aby ¢ =0⇔(ax + by − a) 2 +(bx + ay − b) 2 =0.Evident atunci că ax + by − a = bx + ay − b =0, de unde obţinem x =1,y =0.VIII.64. Să seaflenumărul de pătrate perfecte p astfel încât 2 n ≤ p


IX.62. Rezolvaţi sistemul în necunoscutele x 1 ,x 2 ,...,x n ∈ R ∗ ,n∈ N ∗ :nY2(n − 1) · x k = ¡ nX1+x 2 1¢· x 2 k = ¡ n−1X1+x 2 n¢· x 2 k.k=1k=2Silviu Boga, SuceavaSoluţie. Considerând o soluţie (x 1 ,x 2 ,...,x n ), din prima egalitate obţinemÃX n!Ã n!YnX· x 2 1 − 2(n − 1) x k · x 1 + x 2 k =0. (1)k=2x 2 k·Discriminantul este 4 = (n − 1) ·k=2µnQ P nx k¸2−k=2x 2 kk=2k=2k=1şi, conform inegalităţii mediilor,4 ≤ 0. Însă x 1 ∈ R ∗ prin urmare trebuie să avem4 ≥ 0, fapt care se realizeazăcând x 2 2 = x 2 3 = ··· = x 2 n. Analog obţinem că x 2 1 = x 2 2 = ··· = x 2 n−1şi atuncix 2 1 = x 2 2 = ··· = x 2 n = t ∈ R ∗ ± (n − 1) · tn−1+. Înlocuim în (1) şi găsim că x 1 =(n − 1) · t 2 =±t n−3 , de unde t =1. În concluzie, soluţiile sistemului au forma (x 1 ,x 2 ,...,x n ) cu|x k | =1,k ∈ 1,n.IX.63. Determinaţi x, y ∈ Z pentru care x 3 + y 3 ≤ x + y ≤ x 5 + y 5 .Romeo Ilie, BraşovSoluţie. Notăm x + y = s, xy = p. Dacă s>0, avem:(x + y) ¡ x 2 − xy + y 2¢ ≤ x + y ≤ (x + y) ¡ x 4 − x 3 y + x 2 y 2 − xy 3 + y 4¢ ⇔(x + y) 2 − 3xy ≤ 1 ≤ ¡ x 2 + y 2¢ 2− x 2 y 2 − xy ¡ x 2 + y 2¢ ⇔s 2 − 3p ≤ 1 ≤ ¡ s 2 − 2p ¢ 2 ¡− p s 2 − 2p ¢ − p 2 .Atunci s 2 − 2p ≤ 1+p, deci1 ≤ ¡ s 2 − 2p ¢ 2 ¡− p s 2 − 2p ¢ − p 2 ≤ (1 + p) 2 − p (1 + p) − p 2 =µ= −p 2 + p +1=− p − 1 2+2 5 4 ≤ 5 4 .Deoarece 1 ≤−p 2 +p+1 ≤ 5 4 şi −p2 +p+1 ∈ Z, atunci −p 2 +p+1 = 1, deci p ∈ {0, 1}.Ne amintim că s>0 şi s 2 − 3p ≤ 1 şi obţinem soluţiile (x, y) ∈ {(0, 1) ; (1, 0) , (1, 1)}.Analog, dacă s


−−−→O A C + −−−→ CH A + −−−→ H C A + −−−→ AO C = −−−→ O A C + −−−→ ³ −−−→AO C + O A B + −−−→ ³ −−−→BO C´+ O A D + −−−→ DO C´⇔−−−→CH A + −−−→ H C A =2 −−−−→ O A O C ⇔ 2 −−−−→ O C O A = AH −−−→C + −−−→ H A C.Analog se deduce că 2 −−−−→ O B O D = −−−→ DH B + −−−→ H D B.Atunci−→AC = −−−−→ H C H A ⇔ −−−→ AH C = −−−→ CH A ⇔ AH −−−→C + −−−→ H A C = −→ 0 ⇔ O C = O A ⇔ABCD patrulater inscriptibil ⇔ O B = O D ⇔ −−−→ DH B + −−−→ H D B = −→ 0 ⇔ −−→ BD = −−−−→ H D H B .IX.65. Fie a, b ∈ R astfel încât sin x +sina ≥ cos x · cos b, ∀x ∈ R. Săsearatecă a − b ∈ Z.πAdrian Zanoschi, IaşiSoluţie. Dacă sin a log 4 4 x 1+ ⇔4µ x à à !13√ x !2log 3µ1+ > log34 1+ .4Această ultimă inegalitate rezultă dinobservaţia că 1 3 > 3 √ 24 ,iarlog 3 a =log 3 4×× log 4 a>log 4 a>log 4 b,deîndatăcea>b.X.62. Se dă şirul (z n ) n∈N⊂ C cu proprietatea că z n+2 = z n + iz n+1 , ∀n ∈ N,şi se notează cuV mulţimea termenilor săi. Dacă U 3 ⊂ V ,săsearatecă V = U 12 .(Am notat U k = © z ∈ C | z k =1 ª .)Monica Nedelcu, Iaşi143


Soluţie. Observăm că z n+3 = z n+1 + iz n+2 = z n+1 + i (z n + iz n+1 )=iz n ,prinurmare z n+12 = iz n+9 = i 2 z n+6 = i 3 z n+3 = i 4 z n = z n ,decişirulesteperiodicdeperioadă 12. Înplus,iz ∈ V pentru orice z ∈ V .Pentru a arăta că V = U 12 , este suficient să demonstrăm că U 12 ⊂ V .FieU 12 = {z k | z k =cos 2kπ122kπ+ i sin ,k = 0, 11}.12Evident că {z 0 ,z 4, z 8 } = U 3 ⊂ V ,apoiz 3 = iz 0 ∈ V , z 6 = iz 3 ∈ V , z 9 = iz 6 ∈ V ,z 7 = iz 4 ∈ V , z 10 = iz 7 ∈ V , z 1 = iz 10 ∈ V , z 11 = iz 8 ∈ V , z 2 = iz 11 ∈ V ,z 5 = iz 2 ∈ V şi demonstraţia este încheiată.X.63. Un număr de n jucători aruncă succesivomonedăavândfeţele s şi b;jocul este câştigat de acela care obţine primul faţa s. Săseafleprobabilitateacajucătorul de pe locul k (1 ≤ k ≤ n) să câştige jocul în primele np + k aruncări alemonedei, unde p ∈ N ∗ este dat.Petru Asaftei, IaşiSoluţie. Problema extinde, folosind aceeaşi idee, cazul n =2prezent în unelemanuale. Notăm cu A i evenimentul ca jucătorul de pe locul k să câştige jocul înexact ni + k aruncări ale monedei; pentru realizarea lui A i ,feţele monedei trebuie săapară înordinea bb...b| {z }ni+k−1oris, deci P (A i )=sunt incompatibile în totalitatea lor, iar A 1 ∪ A 2 ∪ ···∪ A p reprezintă evenimentul acărui probabilitate este cerută deproblemă. Avem:P (A) =P (A 1 )+P (A 2 )+···+ P (A p )= 1 2 k + 11+ ···+k+m= 1 2 k ·12 np − 1=12 n − 1µ 121 1 − 2np·2n(p−1)+k 1 − 2 n . ni+k. Evenimentele A 1 ,A 2 ,...,A p22 k+np =X.64. Fie f : R ∗ 2005 ·|x|→ (0, ∞), f (x) =2004 · ¯¯sin 2005 4x¯¯ + 33. Să se+ 2005 ·|x| |x| arate că ¯¯f (x)+1 2¯ −¯¯f (x) − 1 2¯ ≤ 91+|x| 3 , ∀x ∈ R∗ .Ioan Şerdean, OrăştieSoluţie. Construim funcţia g : R ∗ → (0, ∞),g (x) =µ¯¯¯¯f 1+|x|3 (x)+ 1 92¯¯ −¯¯f (x) − 1 2¯, ∀x ∈ R ∗ ;este suficient să arătăm că g (x) ≤ 1, ∀x ∈ R ∗ . Vom demonstra acest lucru în douăetape:i) Dacă |x| > 2, atunci0


deci ¯¯f (x)+1 2¯ = f (x)+1 2 ,iar¯¯¯¯f (x) − 1 2¯ = 1 − f (x). Rezultă că2à !g (x)+ 1+|x|3 · 2f (x) ≤ 1+|x|3 4· 2 ·99 |x| 3 = 8 19 |x| 3 +1 ≤ 8 µ 19 8 +1 =1.ii) Dacă 0 < |x| ≤ 2, folosind faptul că |a| − |b| ≤ |a − b|, avem:g (x) ≤ 1+|x|3· ¯9 ¯f (x)+1 2 − f (x)+1 2¯ = 1+|x|3 ≤ 1+8 =1.9 9X.65. Fie C un cerc de rază 1 şi fie P 1 ,P 2 ,...,P n puncte ale discului corespunzător.Arătaţi că existăunsemicercînchisS ⊂ C astfel încât MP 1 +MP 2 +···+MP n ≥≥ n, ∀M ∈ S.Adrian Zahariuc, elev, BacăuSoluţie. FieO centrul cercului, iar G centruldegreutatealsistemuluidepuncteP 1 ,P 2 ,...,P n . Vom demonstra că MP 1 + MP 2 + ···+ MP n ≥ nMG, pentruoricepunct M din plan. Raportăm planul complex la un reper cu originea în M; fiegafixul lui G, p i afixele punctelor P i ,i= 1,n.Atuncig = p 1 + p 2 + ···+ p n⇒ n ·|g| = |p 1 + p 2 + ···+ p n | ≤ |p 1 | + |p 2 | + ···+ |p n | ⇒nn · MG ≤ MP 1 + MP 2 + ···+ MP n .Rămâne de demonstrat că există un semicerc S ⊂ C astfel încât MG ≥ 1, ∀M ∈ S.Dacă G = O, inegalitatea este adevărată pentruoriceM ∈ C. Dacă G 6= O, fie[AB]diametrul lui C perpendicular pe OG, iarS semicercul delimitat de acesta, "opus"lui G. Atunci m( \MOG) ≥ 90 ◦ , ∀M ∈ S, deciMG ≥ MO =1, ceea ce încheierezolvarea.Clasa a XI-aXI.61. Fie a ∈ R\{1} şi A ∈ M n (C). Dacă există k ∈ N astfel încâtA k+1 = aA k ,săsearatecă I n − A este inversabilă.Gheorghe Iurea, IaşiSoluţie. Fie X 0 ∈ M n,1 (C) osoluţie a sistemului (I n − A) · X = O. AtunciAX 0 = X 0 , deci A k · X 0 = A k+1 · X 0 = X 0 , de unde X 0 = aX 0 şi cum a 6= 1rezultăcă X 0 = O. Astfel, sistemul omogen (I n − A) · X = O are doar soluţia banală şiatunci det (I n − A) 6= 0.XI.62. Fie ABCD un pătrat de centru O, iarM unpunct variabil pe [AB]. Notăm {S} = CM ∩ AD, {E} =SO ∩ MD. Se cere locul geometric al punctului E.Petru Răducanu, IaşiSoluţia 1 (analitică). Raportăm planul la un repercu originea în A, având dreptele AB şi AD axedecoordonate.Putem considera A (0, 0), B (2, 0), C (2, 2), D (0, 2),O (1, 1), iarM (a, 0), cua ∈ (0, 2). Intersectând dreaptaCM cu AD, obţinem că Sµ0,2aa − 2. Atunci ecuaţia145DASLEMOCB


dreptei SO este a +2 · (x − 1) + y − 1=0şi cum DM :2x + a (y − 2) = 0, prina − 2eliminarea parametrului a între cele două ecuaţii găsim ecuaţia locului geometric:x 2 + y 2 − 2y =0. Această ecuaţie reprezintă cercul de diametru [AD], dincaretrebuie reţinut doar semicercul interior pătratului (fără capete), deoarece a ∈ (0, 2).Soluţia 2 (sintetică). În 4AMD cu transversala E−L−S, conform teoremei luiMenelaus obţinem că EMED · LALM · SD =1. Apoi, în 4AMC cu transversala O−L−SSMobţinem LALM · OCOA · SMSM =1.CumSC SM = SDSA = CD EM, atunciAM ED = AM 2AD 2 .Deaici rezultă că AE ⊥ MD,adică AEOD este patrulater inscriptibil, deci E aparţinecercului de diametru [AD].Rămâne să arătăm că orice punct E 0 de pe semicercul interior pătratului aparţinelocului. Fie {M 0 } = DE 0 ∩ AB, {S 0 } = CM 0 ∩ DA, {T } = AC ∩ M 0 D. Atunci:OAOT = ODOT =tg( \OTD) =tg(\AT E 0 );E 0 TE 0 D = E0 TAE 0 : E0 DAE 0 = ctg( \AT E 0 )ctg( \ADM 0 ) = AM 0AD · ctg( \AT E 0 ).Cum S0 DS 0 A = CDAM 0 = ADOA,rezultăcăAM0OT · E0 TE 0 D · S0 DS 0 =1. Din reciproca teoremeiAlui Menelaus în 4AT D urmează că punctele O, E 0 ,S 0 sunt colinare, ceea ce încheiedemonstraţia.XI.63. Fie f,g :[a, b]→ R funcţii continue astfel încât g(a)+f(b) < 1+g(a) f(b).Să searatecăexistă c ∈ (a, b) pentru careµ 3 µ 3 · ¸3f (c) g (c) 2c − a − b 3(2c − a − b) f (c) g (c)+ +=b − c a − c (b − c)(a − c) (b − c) 2 (a − c) 2 .Valeriu Braşoveanu, BârladSoluţie. Florin Popovici, Braşov, dă următoarea generalizare:Fie f,g,h : [a, b] → R trei funcţii continue pe [a, b], astfel încât (g (a) − 1) ·· (f (b) − 1) > 0 şi h (a) =h (b) =0.Dacă n ∈ N ∗ este impar, există c ∈ (a, b) astfelîncât f n (c)(a − c) n + g n (c)(b − c) n +(2c − a − b) n = h (c).Într-adevăr, din ipoteză rezultăcă (g n (a) − 1) (f n (a) − 1) > 0. Considerămfuncţia u :[a, b] → R, u (x) =f n (x)(a − x) n +g n (x)(b − x) n +(2x − a − b) n −h (x).Evident că u este continuă pe[a, b], iaru (a) =(g n (a) − 1) (b − a) n ; u (b) =(1− f n (b)) (b − a) n ⇒⇒ u (a) u (b) =(g n (a) − 1) (1 − f n (b)) (b − a) 2n < 0.Urmează căexistă c ∈ (a, b) astfel încât u (c) =0, de unde concluzia.Problema se obţine pentru n =3şi h (x) =3(2x − a − b)(a − b)(b − x) f (x) g (x).f(x)XI.64. Fie f :(0, 1) → (0, ∞) ofuncţie crescătoare cu lim =1şi f(x) ≤ x,x→0 x1∀x ∈ (0, 1). Definim şirurile (a n ) n≥1, (x n ) n≥1prin a n =(a + b)(2a + b) ···(na + b)146


(a > 0, b > 0) şi x n =[f (a n+1 )+f (a n+2 )+···+ f (a 2n )] 1/n . Săsecalculezelim x n.n→∞Gheorghe Costovici, IaşiSoluţie. Să observăm întâi că problemaesteconsistentă, în sensul că existăfuncţii f ca în enunţ; de exemplu f 1 (x) =x, f 2 (x) =sinx, f 3 (x) =arctgx, f 4 (x) =ln (1 + x) etc.Notăm b n = f (a n+1 )+f (a n+2 )+···+ f (a 2n ) şi observăm că b n → 0:10


obţinem concluzia. Egalitatea se atinge pentru x 1A = x 2A = ··· = x nA =1⇔ x 1 =x 2 = ···= x n .Vom demonstra că x 3 +3n +2≥ 6x, ∀x ∈ R ∗ + ;într-adevăr, acest fapt revine la(x − 1) 2 (x +2)≥ 0, evidentadevărat. Cu notaţia G = n√ x 1 x 2 ...x n ,obţinem:·³x16 n ´3 ³ x1´ ¸·³x2 ´3 ³ x2´ ¸ ·³xn ´3 ³ xn´ ¸≥ +3 +2 +3 +2 ··· +3 +2 ≥A A A AA A³ ´³ ´ ³ ´≥ 6 · x1 6 · x2 ··· 6 · xn =6 n · GnA A A A n ⇒ GnA n ≤ 1 ⇒ G ≤ A,adică tocmai inegalitatea mediilor.Soluţia 2 (Marian Tetiva, Bârlad). Fie x 1 = a 1 A, x 2 = a 2 A,...,x n = a n A,cua 1 + a 2 + ···+ a n = x 1 + x 2 + ···+ x n= n; inegalitatea de demonstrat devine¡Aa31 +3a 1 +2 ¢¡ a 3 2 +3a 2 +2 ¢ ... ¡ a 3 n +3a n +2 ¢ ≤ 6 n ⇔ P (a 1 )·P (a 2 )·· · ··P (a n ) ≤ 6 n ,unde P (x) =x 3 +3x +2.ÎnsăP (a 1 ) · P (a 2 ) ·····P (a n ) ≤ 6 n ⇔ ln P (a 1 )+lnP (a 2 )+···+lnP (a n ) ≤ n ln 6 ⇔⇔ ln P (a µ 1)+lnP (a 2 )+···+lnP (a n ) a1 + a 2 + ···+ a n≤ ln P,nnµ a1 + a 2 + ···+ a ndeoarece P= P (1) = 6. Ultima inegalitate ar fi adevăratăndacă funcţia ln P (x) ar fi concavă. CumP 00 (x) · P (x) − [P 0 (x)] 2 =6x ¡ x 3 +3x +2 ¢ − ¡ 3x 2 +3 ¢ 2== −3 ¡ x 4 − 4x +3 ¢ = −3(x − 1) 2 ¡ x 2 +2x +3 ¢ ≤ 0,demonstraţia este încheiată.Notă. Cum domeniul de definiţie al funcţiei ln P (x) este de forma (x 0 , ∞), cux 0 ∈ (−1, 0), putemspunecă inegalitatea din enunţ arelocpentrux 1 ,x 2 ,...,x n ∈(x 0 , +∞), cux 1 + x 2 + ···+ x n 6=0.Clasa a XII-aXII.61. Să sedeterminefuncţiile derivabile f :(0, ∞) → (0, ∞) care admito primitivă F : (0, ∞) → (0, ∞) pentru care lim F (x) = 2 · lim f (x) = 1 şix&0 x&0F (x) f 0 (x) =F (x) f (x) − f 2 (x)+x − 1, ∀x ∈ (0, ∞).Mihai Haivas, IaşiSoluţie. (dată deFlorin Popovici, Braşov). Cu notaţia u = 1 2 F 2 ,relaţiadin enunţ sescriesubformau 00 (x) − u 0 (x) = x − 1, ∀x ∈ (0, ∞), care este oecuaţie diferenţială liniară deordin2, cu coeficienţi constanţi, neomogenă. Soluţiasa generală esteu (x) =c 1 + c 2 e x − x22 , ∀x ∈ (0, ∞), adică 1 2 F 2 (x) =c 1 + c 2 e x − x22 ,∀x ∈ (0, ∞). Dincondiţia lim F (x) =1obţinem că c 1 + c 2 = 1 . Apoi, derivând înx&0 2ambii membri şi folosind din nou condiţia iniţială, găsim că c 2 = 1 2 ,decic 1 =0.Înconcluzie, F 2 (x) =e x − x 2 , ∀x ∈ (0, ∞).148


Pentru x ∈ (0, 1], avem e x − x 2 ≥ e x − 1 > 0. Pentru x ∈ (1, ∞), avem¡e x − x 2¢ 00= e x − 2 >e− 2 > 0. Apoi, ¡ e x − x 2¢ 0> 0, ∀x ∈ (1, ∞) şi cum e − 1 > 0,deducem că e x − x 2 > 0, ∀x ∈ (1, ∞). Astfel, F (x) = √ e x − x 2 , ∀x ∈ (0, ∞), prinurmare f (x) =F 0 (x) = ex − x 22 √ , ∀x ∈ (0, ∞) este unica soluţie a problemei.e x − x2 XII.62. Fie f : R → R ∗ + ofuncţie continuă,µ iar F primitivaµsa care se anulează1în origine. Să secalculeze: a) lim xF nP 1; b) lim F √k.x→∞ x n→∞k=1Dan Popescu, SuceavaSoluţie. a) Avem:µ 1lim xF F (y) F (y) − F (0)= lim = lim= F 0 (0) = f (0) .x→∞ x y→0 y y→0 y − 0y>0y>0µ 1b) Deoarece lim nF = f (0), pentruorice0 n 1 ,altfelspusF √n > √ a , ∀n >n 1 .nAtuncinXµ 1 Xn 1µ 1nXµ 1 Xn 1µ 1nX 1 Xn 11F √k= F √k + F √k > F √k+ a √ − a √ >k kk=1k=1Xn 1> Fk=1k=n 1+1µ 1 √k+ ak=1şi cum lim a P n 1=+∞, rezultăcă limn→∞k=1 k n→∞k=1k=1k=1nXn11k − a X 1√ , ∀n >n 1knPFk=1µ 1 √k=+∞.XII.63. Fie f,g : R → R două funcţii surjective. Dacă g◦(g − f) este descrescătoareşi există L>1 astfel încât |g (x) − g (y)| ≥ L |x − y|, ∀x, y ∈ R, săsearatecăf areununicpunctfix.Sorin Puşpană, CraiovaSoluţie. Din |g (x) − g (y)| ≥ L |x − y|, ∀x, y ∈ R, rezultă că g este injectivăşi, cum g surjectivă, rezultă g bijectivă; atunci g −1 este o contracţiedeconstantă1< 1, decicontinuă. Rezultă că g este continuă şi cum este şi injectivă, atunci gLeste strict monotonă. Folosind acum faptul că g ◦ (g − f) este descrescătoare, rezultăcă g − f este monotonă, iar g şi g − f au monotonii diferite. Distingem situaţiile:i) Fie g descrescătoare, iar g − f crescătoare; dacă x, y ∈ R, x


În ambele cazuri am obţinut că |f (x) − f (y)| ≥ L ·|x − y| , ∀x, y ∈ R. Cum fsurjectivă, ca şi în cazul lui g, obţinem că f −1 este contracţie de constantă 1 L < 1,deci are un unic punct fix, acesta fiind unicul punct fix şi pentru f.XII.64. Fie M mulţimea funcţiilor f : R → R derivabile de două orişi cuproprietatea că există α ∈ R astfel încât f 00 = αf +1.Arătaţi că existăofamiliedegrupuri {(G α , ∗ α );α ∈ R} ce verifică simultan:1) (G α , ∗ α ) ' ¡ R 2 , + ¢ , ∀α ∈ R;2) {G α ; α ∈ R} este o partiţie a mulţimii M.Temistocle Bîrsan, IaşiSoluţie. Pentru α ∈ R fixat notăm cu G α mulţimea soluţiilor ecuaţiei diferenţialef 00 = αf +1.Ecuaţia caracteristică asociatăesteλ 2 = α, curădăcinile: λ 1,2 = ± √ α,dacă α>0, λ 1,2 = ±i √ −α, dacă α0;G α = © a cos √ −αx + b sin √ −αx − 1/α; a, b ∈ R ª ,dacă α


maxk∈P k (x) = 1 p 1x n + 1 p 2x n−1 + ···+1≤ x n µ 1p 1+ 12p 2+ ···+≤ x n ·12 + 12 · 3 + 12 2 · 5 + µ 17 · 8 + 18 · 9 + ... ¸


Soluţiile problemelor pentru pregătirea concursurilordinnr. 2/2005A. Nivel gimnazialG86. Fie n ≥ 3 un număr natural impar, iar A ⊂ N omulţime cu n 2 − 2n +2elemente. Să searatecăputemalegen numere din mulţimea A cu proprietatea căsuma lor se divide cu n.Soluţie. Scriem A = n−1 Si=0Titu Zvonaru, ComăneştiA i , unde A i conţine toate elementele lui A care daurestul i la împărţirea prin n. Dacă cel puţin o mulţime A i este vidă, cum n 2 −2n+2 =(n − 1) (n − 1) + 1, din principiul cutiei rezultă căexistămăcar o mulţime A j careconţine cel puţin n elemente, iar suma acestora se divide evident cu n. Dacă toatemulţimile A i , i = 0,n− 1, sunt nevide, alegem câte un element din fiecare mulţime.n (n − 1)Suma acestora, modulo n, este0+1+···+(n − 1) ≡ (mod n) şi, cum n2n (n − 1)este impar, n − 1 este par şi rezultă că ≡ 0(modn).2G87. Arătaţi că există o infinitate de numere naturale n pentru care putem găsia i ∈{−1, 1}, i = 1,n astfel încât 2005 =P a i a j .Aflaţi valoarea minimă aluin.1≤i


Soluţie. a) Pentru orice x ∈ R ∗ + \{1}, mulţimea M = {x n | n ∈ Z} are proprietatea(P ), deoarece x n = √ 1 · x 2n ,iar1= √ x n 0 · x−n 0,cun0 ∈ Z arbitrar.b) Fie M omulţime finită cuproprietatea(P ), iara ∈ M cel mai mare elemental acesteia; atunci a este media geometrică a elementelor b, c ∈ M, cub


Soluţie. Răspunsul este 35. Dăm mai întâi un exemplu de colorare cu 34 deculori care nu are proprietatea dorită: împărţim tabla în 4 table 4 × 4, pe cea dinstânga-sus o colorăm în albastru, pe cea din dreapta-jos o colorăm în roşu, iar restulcăsuţelor, în număr de 32, vor fi colorate fiecare cu câte una dintre cele 32 de culorirămase. În acest fel, pe fiecare linie şi pe fiecare coloană vomaveacâte5 culoridiferite.Demonstrăm în continuare că numărul k =35verifică cerinţele problemei. Presupunemprin absurd că arexistaocolorarecu35 de culori care nu are proprietateadorită. Numerotăm culorile de la 1 la 35. Fiel i , respectiv c i ,numărul de linii/coloanepe care se află celpuţin o căsuţă de culoare i. Cumintersecţia a l i linii cu c i coloanePare l i c i elemente, rezultă că 35 l i c i ≥ 64 (∗). Din faptul că pe fiecare linie/coloanăi=1Pavem cel mult 5 culori diferite, rezultă că 35 35Pl i ≤ 40, c i ≤ 40 şi, desigur, l i ≥ 1,i=1i=1c i ≥ 1. Fie x i = l i − 1 ≥ 0, y i = c i − 1 ≥ 0 şi atunci35Pi=1y i = 35 Pi=1c i − 35 ≤ 5. Din(∗) obţinem:35X(x i y i + x i + y i +1)≥ 64 ⇒i=135Xi=135Pi=1x i y i ≥ 19.Px i = 35 l i − 35 ≤ 5,Putem presupune, din simetrie, că max x i ≥ max y i . Cele trei inegalităţi pe care leavem la îndemână nu sunt suficiente pentru a obţine o contradicţie, dar nu lasă decât3 cazuri:i) x i = y i =5şi x k ,y k =0pentru k 6= i (pentru un i fixat); atunci există oculoare care ocupă (nu în întregime!) 6 linii şi 6 coloane, iar restul culorilor aparexact o dată, deci vor rămâne două linii şi două coloane care conţin căsuţe de culoridiferite două câte două, contradicţie cu presupunerea asumată.ii) x i =5, y i =4şi x k ,y k =0pentru k 6= i (pentru un i fixat); acest caz esteevident mai defavorabil decât primul.iii) x i =5, y i =4, y j =1, x k =0pentru k 6= i, y k =0pentru k 6= i, j; niciacumnu putem evita existenţa unei linii sau coloane alcătuită numai din căsuţe de culoridiferite, deoarece 5+max{1, 2} =7< 8.Rezultă că presupunerea făcută estefalsăşi rămâne adevărată concluzia problemei.G92. Fie 4ABC, cu [AB] cea mai scurtă latură. Bisectoarea unghiului Cintersectează paraleladusăprinB la AC în M, iarlaturaAB în C 0 .ParalelaprinC 0 la AC intersectează MA în P şi BC în Q. Arătaţi că înălţimile 4ABC pot filaturi ale unui triunghi dacă şi numai dacă 2AB > P Q.Valentina Blendea şi Gheorghe Blendea, IaşiMSoluţie. Cu notaţiile uzuale, h Pa = 2S Aa , h b =2SC′b , h c = 2S c . Cum c ≤ min {a, b}, atunci h ceste cea mai lungă înălţime, prin urmare h a , h b ,h c pot fi laturi ale unui triunghi dacă şi numaiB Q C154i=1


dacă h c P Q.bPentru ultima echivalenţă, am folosit faptul cunoscut că segmentul ce se spijină pelaturile neparalele ale unui trapez, paralel cu bazele şi care trece prin punctul deintersecţie al diagonalelor, are ca lungime media armonică a bazelor.G93. Fie 4ABC cu m( A) b ≥ 90 ◦ , M — mijlocul lui [BC], iarN punctul decontact al cercului înscris cu BC. Dacă \BAN ≡ \MAC,săsearatecă 4ABC esteisoscel.Doru Buzac, IaşiSoluţie. Dreptele AN şi AM fiind izogonale, putem utiliza relaţia lui Steiner:BNNC · BMMC = AB2 BN, de undeAC2 NC = c2. Pedealtăparte,BN = p − b şi CN = p − c,b2 decip − bp − c = c2b 2 ⇔ b2 (a + c − b) =c 2 (a + b − c) ⇔a ¡ b 2 − c 2¢ + bc (b − c) − ¡ b 3 − c 3¢ =0⇔(b − c) £ a (a + b)+bc − ¡ b 2 + bc + c 2¢¤ =0⇔ (b − c) £ a (b + c) − ¡ b 2 + c 2¢¤ =0.Cum m( A) b ≥ 90 ◦ , atunci b 2 + c 2


I se află pe mediatoarea lui [BC], deci BI = IC şi am arătat că IC = AC; rezultăcă FG = 1 AC. Conform (2), urmeazăcă FG = FD,adică 4FGD este isoscel. În2plus,m(\DFG) = 180 ◦ − m(\BFD) − m( [AF G) = 180 ◦ − 105 ◦ − 15 ◦ =60 ◦ ,deci 4FGD este echilateral.G95. Fie 4ABM dreptunghic în B. Fie C pe [MA] astfel încât MC = AB.Să searatecăîn4 ABC, bisectoarea AD, mediana BE şi înălţimea CF suntconcurente.Dan Brânzei, IaşiSoluţiea 1 (a autorului). Fie {K} = AD∩CF;Avom arăta că punctele B, K, E sunt coliniare, deunde rezultă concluzia. Conform reciprocei teoremeilui Menelaus în 4ADC, coliniaritatea punctelor F K CEB, K, E revine la KAKD = BCBD (1). Fie {D0 } =DAD ∩ BM; atunciB D′ MKAKD = KAKD 0 · KD0KD = CAµ1+ DD0 =CM KD= AC µ1+ BD0 = ACµ1+ BD0AB KC AB D 0 M · D0 M=KC= AC µ1+ ABAB AM · AM = AC µ1+ AB = AMAC AB AC AB .BCPe de altă parte,BD =1+DC BD =1+AC AB = AM , ceea ce încheie justificareaABegalităţii (1).Soluţia 2 (dată deMarius Tiba, elev,Iaşi). Notăm {T } = EB ∩ CF, {U} =EB∩AD. CumCF k BM, cu teorema lui Thales în 4EBD obţinem că ETTB = ECCM .Din teorema bisectoarei în 4AEB avem că EUUB = AE .Însă EC = AE, CM = ABABşi atunci T = U, de unde concluzia problemei.A. Nivel licealL86. CercuriledecentreI a , I b , I c exînscrise 4ABC sunt tangente laturilor[BC], [AC] respectiv [AB] în D, E, F . Bisectoarea interioară aunghiului \BI a Cintersectează laturaBC în M; fie {P } = FE ∩ AM, iarQ ∈ FD, S ∈ DE analogdefinite. Arătaţi că drepteleDP, EQ şi FS sunt concurente.Neculai Roman, Mirceşti, IaşiSoluţie. Cu notaţiile uzuale, avem: BD = AE = p − c, CD = AF = p − b,BF = CE = p − a. Teorema bisectoarei şi teorema sinusurilor în 4BI a C dau<strong>MB</strong>MC = I ABI a C = cos C 2cos B . Aplicăm acum relaţia (R 1 ) din nota Rapoarte determinate2156


de o ceviană şi o secantă într-un triunghi din RecMat 1/2005; obţinem:AFAB · <strong>MB</strong>MC · ACAE · PEPF· p − cb=1⇔PEPF = ABAF · MC<strong>MB</strong> · AEAC ⇔PEPF = cp − b · cos B 2cos C ⇔ PE c(p − c)=PF b(p − b) · cos C 22cos B · cos2 B 22cos 2 C2Analog, QFQD = cos A 2cos C , SDSE = cos B 22cos A , prin urmare PEPF · QFQD · SD =1şi concluziaSE2rezultă din reciproca teoremei lui Ceva.Notă. Vlad Emanuel, elev, Sibiu, evaluează rapoartele necesare pentru aplicareareciprocei teoremei lui Ceva prin considerente de arii.⇔ PEPF = cos C 2cos B .2L87. Să searatecă un tetraedru cu muchiile în progresie geometrică şi în careperechile de muchii opuse sunt perpendiculare, este regulat.Marius Olteanu, Rm. VâlceaNotă. Ulterior publicării în revista noastră, problema a apărut cu nr. O:1088 înG.M. 6/2005. Soluţiaeipoatefigăsită în G.M. 12/2005, pp. 640.Soluţie corectă s-a primit de la Vlad Emanuel, elev, Sibiu.L88. Fie 4ABC de arie S, perimetru 2p, având razele cercurilor circumscris şiînscris R, respectiv r. Notăm k = 8+3√ 3p + 3 r. Fie A, B, C discuri de centru12 4A, B, C şi aceeaşi rază δ


R ≥ 2r (⇔r2R ≤ 1 4 ).L89. Câte drumuri de la A la B există, dacă dinoricepunctdepetraseuneputemdeplasadoarsprestângasauîn jos?Irina Mustaţă, elevă, IaşiSoluţie. Notăm cu a n numărul drumurilor de la Ala B şi încercăm să găsim o relaţiederecurenţă pentrua n . Notăm cu b n−1 numărul drumurilor de la E la B;BGFCDAEn etajenumărul drumurilor de la C, F , respectiv D, pânălaB va fi a n−1 , a n−2 , respectivb n−2 . De la A la C putem ajunge în două moduri, iar de la A la D putem ajungeîntr-un singur mod fără atreceprinC, prin urmarea n =2a n−1 + b n−2 . (1)Cum din E putem pleca fie spre C, fie în jos, apoi stânga, spre D, atuncib n−1 =2a n−1 + b n−2 . (2)Din (1) şi (2) obţinema n =2 n−1 + b n−2 =2a n−1 + a n−2 + b n−3 = ...=2a n−1 + a n−2 + ...+ a 1 +1.(Acel 1 din final reprezintă singurul drum posibil de la G la B.) Trecem n → n +1şi scădem egalităţile obţinute; găsim:a n+1 − a n =2a n − a n−1 ⇔ a n+1 =3a n − a n−1 .Se verifică uşor că a 1 =2, a 2 =5,obţinând astfel o recurenţă liniară omogenă deordin II pentru a n . Mai mult, se demonstrează imediat prin inducţie că a n = F 2n+1 ,unde F k este al k-lea termen din şirul lui Fibonacci.Notă. Soluţie corectă s-a primit de la Vlad Emanuel, elev, Sibiu.L90. Fie n≥1 un număr natural. Să searatecămulţimea A={n+1,n+2,...,2n}se poate partiţiona în trei submulţimi, fiecare cu aceeaşi sumă a elementelor, dacă şinumai dacă n este multiplu de 3, n ≥ 6.Marian Tetiva, BârladSoluţie. Pentru ca A să poată fipartiţionată întreiclaseavândaceeaşi sumăn(3n +1)a elementelor, trebuie ca suma elementelor sale, care este ,să fie multiplu2de 3, ceea ce conduce la 3 | n. Pentru n =3,evidentcă A = {4, 5, 6} nu se poatepartiţiona cum cere enunţul, deci n ≥ 6.Fie acum n =3k, k ≥ 2; dacă k este par, k =2p, p ≥ 1, atunciA = {6p +1,6p +2,...,12p}. ScriemA = {6p +1, 12p} ∪ {6p +2, 12p − 1} ∪ ···∪ {9p, 9p +1},numărul de submulţimi fiind 3p, fiecare cu aceeaşi sumă a elementelor. Claselepartiţiei se obţin reunind p dintre aceste submulţimi (prima clasă), apoi alte p şi laurmă cele p care au rămas.Pentru k impar, k =2m +1, m ≥ 1, avemdepartiţionat mulţimea A = {6m +4,6m +5,...,12m +6}. Remarcăm că suma tuturor elementelor lui A este 3(2m +1)··(9m +5), iar suma elemntelor lui A care dau restul 2 la împărţirea cu 3 este(6m +5)+(6m +8)+···+(12m +5)=(2m +1)(9m +5),158


prin urmare o clasă apartiţiei poate fi construită din numerele care dau restul 2împărţirea cu 3; fieP această mulţime. Mai notăm cu M mulţimea elementelor luiA ce dau restul 1 la împărţirea cu 3 şi cu N mulţimea elementelor lui A ce suntmultipli de 3. Suma elementelor lui M este (2m +1)(9m +5)− (2m +1),iarsumaelementelor lui N este (2m+1)(9m+5)+(2m+1);vomîncercasăschimbăm între eleelemente din N cu elemente din M astfel încât să egalizăm sumele. Întâi, efectuămschimbarea 12m +1 ↔ 12m +6,carecreşte suma elementelor lui M cu 5. Apoi,efectuăm m − 2 schimbări, fiecare mărind suma elementelor lui M cu 2 (şi micşorândcorespunzător suma elementelor lui N):12m − 2 ↔ 12m; 12m − 5 ↔ 12m − 3; ... ; 9m +7↔ 9m +9.Cu aceasta, soluţia problemei este încheiată.Notă. O frumoasă soluţie prin inducţie matematică s-a primit de la VladEmanuel, elev, Sibiu.L91. Punctele planului se colorează în3 culori astfel încât fiecare culoare săfie folosită celpuţin o dată. Spunem despre un triunghi că esteaproape echilateraldacă măsurile unghiurilor sale sunt cel mult 60, 001 ◦ . Arătaţi că existăuntriunghiaproape echilateral cu vârfurile de culori diferite.Adrian Zahariuc, elev, BacăuSoluţie. Să numimalbastru,roşu şi verde cele 3 culori. Vom scrie M(a), M(r)sau M(v) după cum este punctul M albastru, roşu sau verde. Fie A(a), B(r) şi C(v)şi D 1 un disc de centru O 1 şi rază r>0 astfel încât A, B, C ∈ D 1 .Fied>1000000r.Fie D discul de centru O 1 şi rază d + r.Să luăm mai întâi cazul când mulţimea de puncte ¯D nu este monocromatică, undeam notat cu ¯T complementara mulţimii T (faţă de plan). Fie P, Q ∈ ¯D de culoridiferite. Este clar că existăoînşiruire finită de puncte P = M 0 ,M 1 ,...,M n = Qastfel încât [M i M i+1 ] ⊂ ¯D şi |M i M i+1 | ≤ 2r, i = 0,n− 1. CumM 0 şi M n au culoridiferite, rezultă căexistă i ∈ {0, 1, 2,...,n− 1} astfel încât M i şi M i+1 au culoridiferite. În concluzie, există T,S ∈ ¯D cu culori diferite, să zicem T (a), S(r) astfelîncât [TS] ⊂ ¯D şi |TS| ≤ 2r. Fie D 2 discul de rază r şi centru O 2 ,undeO 2 estemijlocul lui [TS]. Este clar că T,S ∈ D 2 şi că |O 1 O 2 | ≥ d. Fie un punct O 3 astfelîncât triunghiul O 1 O 2 O 3 să fie echilateral. Să demonstrăm acum următoareaLemă. Fie O 1 , O 2 şi O 3 vârfurile unui triunghi echilateral de latură l ≥ 1000000rşi D 1 , D 2 şi D 3 , 3 discuri de rază r şi centre O 1 , O 2 ,respectivO 3 .Dacă X ∈ D 1 ,Y ∈ D 2 şi Z ∈ D 3 ,atuncimăsurile unghiurilor 4XY Z sunt cel mult 60, 001 ◦ .Demonstraţie. Avem XY ≤ XO 1 + O 1 O 2 + O 2 Y ≤ l +2r şi O 2 O 3 ≤ O 2 Y +YZ+ ZO 3 ≤ YZ+2r, sauYZ ≥ l − 2r. AtunciXYYZ ≤ l +2rl − 2r ≤ 1000002sin Z< 1, 00001 ⇒ < 1, 00001.999998 sin XRelaţiile analoage se demonstrează într-omanierăasemănătoare etc.Să revenim la problemă. Fie D 3 discul de centru O 3 şi rază r. Fie W ∈ D 3 ,oarecare. Dacă W este albastru, conform lemei, triunghiul CSW este aproape echilateral.Dacă W este roşu, triunghiul CTW este aproape echilateral. În fine, dacă Weste verde, triunghiul ASW este aproape echilateral şi demonstraţia pentru primulcazseîncheie.159


Să luăm acum cazul când mulţimea de puncte ¯D este monocromatică, să zicem cătoate punctele ei sunt albastre. Fie L = {|RV | : R(r),V(v) ∈ R 2 }. Din presupunereacă toate punctele din exteriorul discului ¯D sunt albastre, rezultă cămulţimea L estemărginită. Atunci există l ∈ L astfel încât 1, 000001l >l 0 pentru orice l 0 ∈ L. Fiepunctele R 1 (r), V 1 (v) astfel încât |R 1 V 1 | = l. Fie punctul Q astfel încât QR 1 =QV 1 =1, 000001l. Dacă Q este roşu, OV 1 =1, 000001l contrazice alegerea lui l.Analog, Q verde ne conduce la o contradicţie. Rezultă că Q este albastru. Se verificăimediat că triunghiulQR 1 V 1 este aproape echilateral şi problema este rezolvată.L92. Dacă a i ∈ (1, ∞), ∀i = 1,n, n ≥ 3, iarm, p ∈ N cu m>p≥ 1, săsedemonstreze inegalitateaa m 2 + ···+ a m na m 1 + a m 3 ···+ a m nlog a1a m−p2 + ···+ a m−p +log a2na m−p1 + a m−p3 ···+ a m−p + ···+na m 1 + ···+ a m n−1+log ana m−p1 + ···+ a m−p ≥ np.n−1Gheorghe Molea, Curtea de ArgeşSoluţie. Considerăm o ordonare a numerelor a i ∈ (1, ∞); putem presupune fărăa restrânge generalitatea că a 1 ≥ a 2 ≥ ··· ≥ a n . Atunci a p 1 ≥ ap 2 ≥ ··· ≥ ap n şia m−p1 ≥ a m−p2 ≥ ··· ≥ a m−pn , ∀m >p≥ 1. Aplicăm inegalitatea lui Cebâşev şiinegalitatea mediilor:a m 2 + a m 3 + ···+ a m nn − 1= am−p 2 a p 2 + am−p 3 a p 3+ ···+ am−p n a p nn − 1≥ am−p 2 + a m−p3 + ···+ a m−pn· ap 2 + ap 3 + ···+ ap n≥n − 1n − 1≥ am−p 2 + a m−p3 + ···+ a m−pn·qa n−1 p 2n − 1ap 3 ···ap n ⇒a m 2 + am 3 + ···+ am na n−p2 + a m−p3 + ···+ anm−p ≥q(a n−1 2 a 3 ···a n ) p .Dacă notăm cu S membrul stâng al inegalităţii din enunţ, folosind această inegalitateşi analoagele obţinem:S ≥p £loga1 (a 2 a 3 ···a n )+logn − 1a2(a 1 a 3 ···a n )+log an(a 1 a 2 ···a n−1 ) ¤ ⇔S ≥p X ³logn − 1aia j +log aja i´.i6=jCum log aia j > 0, atunci log aia j +log aja i ≥ 2. Numărul grupelor de forman(n − 1)(log aia j +log aja i ) este şi concluzia problemei urmează imediat.2Notă. Pentru n =3, m =4, p =3,obţinem problema 22814 din G.M. 5-6/1993.Pentru n =3, m =2, p =1obţinem problema C:1605 din G.M. 11/1994.Principial aceeaşi soluţie a dat Vlad Emanuel, elev, Sibiu.L93. Există vreunpolinomf (X, Y ) în două nedeterminate astfel încât{f (m, n) | m, n ∈ Z} ∩ N ∗ = © x 2004k | k ≥ 1 ª ,160≥


¡ √ ¢ n ¡ √ ¢ n1+ 2 + 1 − 2unde x n =?2Gabriel Dospinescu, ParisSoluţie. Răspunsul este afirmativ! De exemplu, putem considera f(X, Y )=X 2004 (2 − (X 2 − 2Y 2 ) 2 ) şi atunci, pentru m, n ∈ Z,f(m, n) > 0 ⇔ 2 > (m 2 − 2n 2 ) 2 ⇔ |m 2 − 2n 2 | =1.Obţinem astfel două ecuaţii Pell; rezolvându-le, obţinem că, de fapt,{m 2004 | există n a.î. |m 2 − 2n 2 | =1} = {x 2004k | k ≥ 1},ceea ce arată că {f(m, n) | m, n ∈ Z} ∩ N ∗ = {x 2004k| k ≥ 1}.L94. Fie A ∈ M n (C) o matrice cu coeficienţi întregi, inversabilă şi astfel încâtmulţimea © A k | k ∈ N ª este finită. Să sedemonstrezecă aceastămulţime are cel mult3 n2 elemente. Rămâne rezultatul adevărat dacă suprimăm condiţia ca elementelematricei să fie întregi?Gabriel Dospinescu, ParisSoluţie. Din ipoteză rezultă căexistă p ≥ 1 astfel încât A p = I n .Săluăm p celmai mic cu această proprietateşi să observăm că dacă p ≤ 3 n2 atunci am terminatprimaparteaproblemei. Să presupunem deci contrariul şi să considerăm matriceledin mulţimea H = {I,A,...,A p−1 }. Ele sunt în mod evident diferite oricare douăşi, mai mult, fiecare element al lui H are proprietatea că X p = I n .Săobservăm cădin faptul că H are cel puţin 3 n2 +1 elemente, există două elemente în H acărordiferenţă esteomatricecutoateelementelemultiplide3 (principiul lui Dirichlet,observând că există 3 n2 matrice cu elemente 0, 1 sau 2). Deci există B ∈ M n (Z) şi1 ≤ i


RSoluţie. Fie T ∈ R ∗ + o perioadă aluif şi notăm F (x) = x (f(t)+a)dt; atunciZ x+(n+1)T0Z TF (x +(n +1)T ) − F (x + nT )=(f (t)+a) dt = f (t) dt + aT ⇒x+nT0Ã Z !TF (x + nT )=F (x)+n f (t) dt + aT , ∀n ∈ N ∗ , ∀x ∈ [0,T).0TRCum f este mărginită, rezultă că F este mărginită pe[0,T], iardacă f(t)dt+aT 6=Ã! 00, rezultăimediatcă lim F (x) =(+∞)sgn TRf (x) dt + aT .x→∞Fie acum a = − 1 TTRf (t) dt; atunci0Z x+TF (x + T )=00(f (t)+a) dt = F (x) , ∀x ∈ R,deci F este periodică deperioaă T . Presupunem prin absurd că F este constantă.Rx 2Atunci F (x 1 )=F (x 2 ), ∀x 1 ,x 2 ∈ R, deci (f (t)+a) dt =0, ∀x 1 ,x 2 ∈ R. Pentrux 1ε>0 fixat, putem găsi δ>0 astfel caZ x2(l s − ε + a)(x 2 − x 1 ) ≤ (f (t)+a) dt ≤ (l s + ε + a)(x 2 − x 1 )x 1pentru x 0 − δ ≤ x 1


Clasele primareProbleme propuse 1P.114. În piramida alăturată unele numere s-au şters de-alungul timpului. Putem să le punem la loc?(Clasa I )Ionela Bărăgan, elevă, Iaşi20 288 12 169531P.115. Dacă din prima ladă iau2mereşilepunînladaadoua,dinadoualadăiau 3 mere şi le pun în lada a treia, iar din lada a treia iau 4 mere şi le pun în primaladă, atunci în fiecare ladă voi avea câte 34 mere. Câte mere au fost în fiecare ladă?(Clasa I )Mariana Nastasia, elevă, IaşiP.116. Luni, mama pune într-un coş câteva mere. Joi, ea găseşte în coş doar 20de mere. Câte mere a pus mama în coş, ştiind că, în fiecare zi din acea săptămână,Mihai, cel mai mare dintre fraţi, împarte fraţilor mai mici cu un măr mai mult ca înziua precedentă şi că joielîmparte5 mere? Câte mere mai rămân în coş sâmbătă,după împărţirea merelor?(Clasa II-a)Inst. Maria Racu, IaşiP.117. În exerciţiul 1¤1¤1¤1¤1¤1¤1 =fiecare căsuţă poate fi înlocuităcu”+”sau ” − ”. Cât poate fi rezultatul acestui exerciţiu?(Clasa II-a)Diana Tănăsoaie, elevă, IaşiP.118. Calculaţi a + b + c + d, ştiind că a × b =5şi c × d =15. Găseşte toateposibilităţile.(Clasa III-a)Înv. Rica Bucătariu, IaşiP.119. Produsul a 10 numere naturale este 40. Aflaţi cea mai mică şi cea maimare valoare a sumei celor 10 numere.(Clasa III-a)Înv. Mirela Buburuzanu, Tomeşti (Iaşi)P.120. Se consideră numerele1,2,3,4,5,... ,49. Careestecelmaimarenumăr de numere pe care putem să alegem dintre acestea astfel încât suma oricărortrei dintre ele să seîmpartăexactla9.(Clasa III-a)Înv. Felicia-Petronela Leanu, Cepleniţa (Iaşi)P.121. Ceasul lui Andrei o ia înainte cu 20 secundepeoră. El a potrivit ceasulluni la ora 8 şi a citit din nou ceasul lunea următoare la aceeaşi oră. Ştiind că înaceastă durată ceasul nu a funcţionat permanent, iar la ultima citire arăta ora 8 h50 min, să seaflecâtnuafuncţionat ceasul.(Clasa IV-a)Paula Borşanu, elevă, IaşiP.122. La Concursul de matematică "F.T.Câmpan", etapa judeţeană, au participat100 elevi de clasa a IV-a, care au avut de rezolvat 3 probleme. Dacă 70eleviaurezolvat bine prima problemă, 69 a doua problemăşi 64 a treia problemă, să searatecă măcar 3 elevi au rezolvat corect toate cele trei probleme.(Clasa IV-a)Anca Cornea, elevă, IaşiP.123. Într-o cutie sunt 34 bile, din care unele cântăresc cu 1 g mai mult. Dacăfiecare bilăcântăreşte un număr natural de grame, iar masa tuturor bilelor este 113 g,1 Se primesc soluţii până ladatade1iunie2007.163


să seaflecâtebilesuntmaigrele.(Clasa IV-a)Petru Asaftei, IaşiClasa a V-aV.71. Comparaţi numerele 3 300003 şi 2 450004 .Lucian Tuţescu, CraiovaV.72. Fie mulţimile A, B astfel încât A ⊂ B, |P (A)| ≥ 60, |P (B)| ≤ 260. Săsedetermine |A| şi |B|. (Prin|X| am notat cardinalul mulţimii X.)Petru Asaftei, IaşiV.73. Să sescrienumărul 2006 2005 ca o sumă deşase pătrate perfecte nenule.Ionel Nechifor, IaşiV.74. Se consideră şirul de numere naturale 1, 1, 2, 5, 12, 27, 58, . . . . Calculaţisuma primilor 100 de termeni ai şirului.Marius Damian, BrăilaV.75. Fie x, k ∈ N, x ≥ 2, k


Clasa a VII-aVII.71. Fie 4ABC, AB < AC şi D ∈ (AC). Fie AE bisectoarea lui \BAC,E ∈ (BD), F mijlocul lui [AD], {O} = AE ∩ BF, {G} = DO ∩ AB. SăsearatecăGD k BC ⇔ AB = CD.Carmen Daniela Tamaş, BârladVII.72. Fie ABCD paralelogram, E ∈ (CD), {M} = AE ∩ BD, {N} = BE ∩AC, {O} = AC ∩ BD. Săsearatecă A MEN =2A MON .Mirela Marin, IaşiVII.73. Fie a


=2yz + a, iar √ x − a + √ b − x = y.Andrei Nedelcu şi Lucian Lăduncă, IaşiIX.72. Fie x, y, z ∈ [1, +∞) aşa încât x[x] = y[y] = z[z] .Săsearatecăqq[x] 2 +[y] 2 +[z] 2 + {x} 2 + {y} 2 + {z} 2 = p x 2 + y 2 + z 2 .Ce egalitate se obţine pentru x, y, z ∈ (−∞, −1]?Dan Plăeşu, IaşiIX.73. Fie a 1 ,a 2 ,...,a n ∈ (0, ∞) cu a 1 a 2 ···a n =1. Pentru orice m ∈ N ∗ ,săsearate că are loc inegalitateaa m 1 + am 2 + ···+ am n ≥ am−1 1 + a2 m−1 + ···+ a m−1n .Marius Tiba, elev, IaşiIX.74. Fie a, b, c lungimile laturilor 4ABC şi m, n ∈ (0, +∞). Considerămpunctele A 0 , B 0 , C 0 astfel încât C ∈ (AA 0 ), A ∈ (BB 0 ), B ∈ (CC 0 ) şi CA 0 = ma + n,AB 0 = mb + n, BC 0 = mc + n. Săsearatecă 4ABC şi 4A 0 B 0 C 0 au acelaşi centrude greutate dacă şi numai dacă 4ABC este echilateral.Dumitru Mihalache, BârladIX.75. Fie ABCD patrulater inscriptibil, {O} = AC ∩ BD, m(\AOD) =60 ◦şi punctele M ∈ (AB), N ∈ (CD). Notăm k = MA<strong>MB</strong> , r = NCND , p = OA½OB . Dacăp ∈ 2k, 2 r , 1 ¾,săsearatecă m( \MON) 6= 90 ◦ .2rMihai Haivas, IaşiClasa a X-aX.71. Fie a, b, c lungimile laturilor unui triunghi. Să searatecă√ √ √a +24bc√ √ b +24ac√ c +24ab√ √ + √ √ + √ √ > 3.b + c a + c a + bLucian Tuţescu, CraiovaX.72. Să se rezolve sistemulx 2 log 2 15 + y 2 log 3 10 + z 2 log 5 6=2(xy + yz + zx) ,x + y + z =5.Marius Damian, BrăilaX.73. Determinaţi funcţiile f : N → µ R cu proprietatea că pentru orice x, y ∈ Nx + y f (x)+f (y)astfel încât 3 | x + y, are loc egalitatea f = .33Eugenia Roşu, elevă, IaşiX.74. Fie d 1 , d 2 două drepte perpendiculare şi l 1 , l 2 dreptele suport ale bisectoarelorcelor două perechi de unghiuri opuse <strong>format</strong>e de ele. Determinaţi 4ABCcu A ∈ d 1 , B ∈ d 2 , C ∈ l 1 şi O, H ∈ l 2 (O, H cu semnificaţiile uzuale).Temistocle Bîrsan, IaşiX.75. a) Fie α ∈ C ∗ . Dacă z 1 , z 2 , z 3 sunt trei numere complexe diferite astfelîncât Re (α¯z 1 )=Re(α¯z 2 )=Re(α¯z 3 ), atunci punctele de afixe z 1 , z 2 , z 3 suntcoliniare. Notăm cu d dreapta pe care sunt situate aceste puncte.166


) Dacă z 0 1, z 0 2, z 0 3 sunt trei numere complexe diferite cu proprietatea că Im (α¯z 0 1)=Im (α¯z 0 2) = Im(α¯z 0 3), atunci punctele cu afixele z 1 , z 2 , z 3 sunt coliniare. Fie d 0dreapta ce le conţine.c) Să searatecădrepteled şi d 0 sunt perpendiculare.Constantin Cocea, IaşiClasa a XI-aXI.71. Fie şirul (x n ) n≥1, unde x 1 > 1, x n+1 = x n − 1limn→∞ xn n.ln x n, ∀n ∈ N ∗ .CalculaţiDan Popescu, SuceavaXI.72. Este posibil ca o funcţie f : R → R,careverifică 1+f(x)+f(x) f(1+x) =0,∀x ∈ R, săfiecontinuăpeR?Dorin Mărgidanu, CorabiaXI.73. Fie f :[0, 1] → R o funcţie nenulă declasă C k+1 pe [0, 1], k ∈ N ∗ ,astfelca f (0) = f (1) = 0. Dacă pentru orice 1 ≤ j ≤ k există a j ∈ {0, 1} astfel încâtf (j) (a j )=0,atunciexistă x 1 ,x 2 ∈ (0, 1) astfel ca f (k+1) (x 1 ) · f (k+1) (x 2 ) < 0.Gheorghe Moroşanu şi Paul Georgescu, IaşiXI.74. Fie a, b, c ∈ R, a 6= 0, n ∈ 2N ∗ .Următoarele afirmaţii sunt echivalente:(1) b 2 − 4ac ≤ 0, (2) det ¡ ¢aA 2 + bA + cI n ≥ 0, ∀A ∈ Mn (R).Marian Ursărescu, RomanXI.75. Fie A, B ∈ M 2 (C). Săsearatecădacă AB − BA comută cuA şi cu B,atunci AB = BA.Dorel Miheţ, TimişoaraClasa a XII-aXII.71. Fie f :[a, b] → R ofuncţie derivabilă cu derivata continuă. Să searateR bcă lim f (x) · sin nx dx =0.n→∞ aDan Radu, BucureştiXII.72. Fie funcţia f :[0, 1] → R, indefinit derivabilă pe[0, 1], cuproprietateacă există M>0 astfel încât ¯¯f (n) (x)¯¯ ≤ M, ∀x ∈ [0, 1] şi ∀n ∈ N. Arătaţi că:a) P pk=1lim(−1)k+1 (n + p)(n + p − 1) ···(n + p − k +2)f (n+p−k) (1)n→∞(−1) p+1 =0, ∀p ∈ N ∗ ;(n + p)(n + p − 1) ···(n +1)P pk=1b) lim(−1)k+1 (n + p)(n + p − 1) ···(n + p − k +2)f (n+p−k) (1)p→∞(−1) p+1 =(n + p)(n + p − 1) ···(n +1)=Z 1XII.73. Să searatecăZ 111+x 2 ln (1 + x)n +(1− x) n1+x n dx = n00x n f (n) (x) dx, ∀n ∈ N.167Z 10Ovidiu Pop, Satu Mareln (1 + x)1+x 2 dx = n π 8 ln 2, n∈ N∗ .


π/4RSă se calculeze apoi ln 1+tg3 x1+3tg 2 x dx.0Gabriel Necula, PlopeniXII.74. Fie (G, ·) un grup comutativ cu proprietatea că există n ∈ N astfel încâtdin x n = y n rezultă x = y, cux, y ∈ G. Dacă f, g sunt două endomorfisme alelui G, atunci ecuaţia f (x) =g ¡ x −1¢ are soluţie unică dacăşi numai dacă funcţiah : G → G, h (x) =f (x n ) g (x n )este injectivă.D. M. Bătineţu-Giurgiu, BucureştiXII.75. Fie A ∈ M n (C) de rang r şi S = {B ∈ M n (C) | ABA = O n }.Arătaţică S este subspaţiu vectorial în M n (C) şi că dim S = n 2 − r 2 .Adrian Reisner, ParisPremiu pe anul 2006 acordatde ASOCIAŢIA “RECREAŢII <strong>MATEMATICE</strong>”Se acordă un premiu în bani în valoare de 100 lei elevuluiZAHARIUC Adrian — Colegiul Naţional "Ferdinand I", Bacăupentru nota O problemă despre suma cifrelor unui numărnaturalînbazedenumeraţieoarecare apărută în acest număr la pagina 113.Acest premiu este acordat de Asociaţia "Recreaţii Matematice" ca urmare acontractului de sponsorizare cu Fundaţia Culturală "Poiana"(director d-l DanTiba).Premii pe anul 2006 acordate unor elevidin Republica Moldova participanţi la faza finalăaONM,Iaşi, 2006Se acordă câte un premiu în valoare de 100 lei elevilorGUZUN IonBOREICO Iuriedin lotul Republicii Moldova, care au obţinut cele mai bune rezultate.Premiile sunt acordate de Asociaţia "Recreaţii Matematice", iarsumadebani provine din contractul de sponsorizare cu Fundaţia Culturală "Poiana"(director d-l Dan Tiba).168


Probleme pentru pregătirea concursurilorA. Nivel gimnazialG106. Fie m ∈ N \{0, 1} şi k ∈ N, k ≤ m. Pentru fiecare x ∈ N, considerămpropoziţiile: x > 1; x > 2; ...; x > m. Aflaţi x ∈ N pentru care k din cele mpropoziţii sunt adevărate, iar celelalte m − k sunt false.Maria Miheţ, TimişoaraG107. Mulţimea A ⊂ N de cardinal n ∈ N ∗ are proprietatea că, oricare ar fipatru elemente ale sale, putem alege două cusuma2 2006 +1.Aflaţi valoarea maximăaluin.Dan Nedeianu, Dr. Tr. SeverinG108. Fie m, n ∈ N ∗ . Săsearatecămulţimea numerelor întregi de modul celmult egal cu n poate fi partiţionată înm submulţimi cu aceeaşi sumă a elementelor,dacă şi numai dacă n +1≥ m.Marian Tetiva, BârladG109. La un concurs se dau şase probleme evaluate cu 1, 2, 3, 4, 5, repectiv6 puncte. Dacă unelevnurezolvăoproblemă, primeşte 1 punct; dacă orezolvă,primeşte punctajul corespunzător. Fiecare elev obţine măcar 11 puncte. Să searatecă oproblemă a fost rezolvată de cel puţin o treime dintre elevi.Gabriel Dospinescu, student, ParisG110. Fie mulţimile A = {k + √ n | k ∈ Z,n∈ N} şi B =(0, 1/10). Arătaţi căA ∩ B este infinită.Petru Asaftei, IaşiG111. Fie 0


a) S 1 − S 2 =4 p S AEN · S BFQ ; b) S 3 ≥ S 1 ; c) 1 S 3= 1 S 3− 1 S 3.Claudiu Ştefan Popa, IaşiB. Nivel licealL106. Fie I centrul cercului înscris în 4ABC. Dreptele AI, BI şi CI intersecteazăa doua oară cercurile circumscrise triunghiului BCI, CAI şi ABI în A 0 , B 0 ,respectiv C 0 .Dacănotăm cu |XY Z| perimetrul 4XY Z,să se demonstreze căBC|BCA 0 | + CA|CAB 0 | + AB|ABC 0 | =1.Titu Zvonaru, ComăneştiL107. Fie M 1 N două puncte situate în interiorul 4ABC, având distanţele pânăla laturile AB, BC, CA egale cu 3, 2, 7, respectiv 9 2 , 5, 52. Dacă raza cerculuicircumscris 4ABC este R =8,să se calculeze MN.Vlad Emanuel, elev, SibiuL108. Să searatecăînorice4ABC are loc inegalitatea3 √ 32 − (sin A +sinB +sinC) ≥ 4√ 3sin 2 π − 3A .12Marian Tetiva, BârladL109. Se dau numerele reale pozitive subunitare a 1 ,a 2 ,...,a 2n 2 −n, n ∈ N\{0, 1}.Să se demonstreze inegalitatea (sumarea se face prin permutări circulare)"Xa 2n−112 n − 1a 2n−12 + a 2n−13 + ...+ a 2n−1


L114. Considerăm o parabolăşi două drepte secante parabolei, paralele între ele,dar neparalele cu axa de simetrie a parabolei. Folosind doar rigla negradată, să seconstruiască tangenta la parabolă care este paralelă cudrepteledate.Titu Zvonaru, ComăneştiL115. Determinaţi P ∈ R[X], grad P ≥ 2, astfel încât funcţia f : R → R,f(x) =p({x})+{p(x)} să fieperiodică (unde p este funcţia polinomială ataşată luiP ,iar{·} desemnează parteafracţionară).Paul Georgescu and Gabriel Popa, IaşiTraining problems for mathematical contestsJunior highschool levelG106. Let m ∈ N\{0, 1} and k ∈ N, k ≤ m. For any x ∈ N, consider theassertions: x>1; x>2; ... ; x>m. Find x ∈ N such that k out of the massertionsaretruewhiletheotherm − k are false.Maria Miheţ, TimişoaraG107. The set A ⊂ N of cardinal number n ∈ N has the property that, from anyfour of its elements, we can choose two elements whose sum is 2 2006 +1. Find themaximum value of n.Dan Nedeianu, Drobeta-Turnu SeverinG108. Let m, n ∈ N ∗ . Show that the set of integer numbers of absolute valueat most equal to n can be partitioned into m subsets with the same sum of theirelements if and only if n +1≥ m.Marian Tetiva, BârladG109. Six problems are proposed for a school contest and they are respectivelyevaluated with 1, 2, 3, 4, 5, 6 points. If a schoolchild does not solve a problem hereceives 1 point for it; if he (she) solves it he gets the corresponding points. Everyschoolchild receives at least 11 points. Show that a problem was solved by at leastone third of the participating schoolchildren.Gabriel Dospinescu, ParisG110. Let us consider the sets A = {k + √ n | k ∈ Z, n∈ N} and B =(0, 1/10).Show that A ∩ B is infinite.Petru Asaftei, IaşiG111. Let 0


espect to line AB. The perpendicular line at C ∈ (AB) on AB meets C 1 at E andC 2 at D. IfAD ∩ C 1 = {F }, show that AE is tangent to the circle circumscribed to∆DEF.Alexandru Negrescu, highschool student, BotoşaniG114. Let ABCD be a parallelogram that is not a rhomb with m(\BAD) =60 ◦ . If M,N ∈ (AC), P ∈ (BC) and Q ∈ (CD) are such that [BM, [DN,[AP and [AQ are the bisectrix lines of the angles \ABC, \ADC, \BAC and respectively\DAC then MP is orthogonal on NQ.Andrei Nedelcu, IaşiG115. Let MNPQbethesquareinscribedinthesquareABCD,withM ∈ (AB),N ∈ (AD), P ∈ (CD) and Q ∈ (BC) and let {E } = PN ∩ AB, {F } = PQ∩ AB.We denote by S 1 , S 2 , S 3 , the areas of the square ABCD, ofthesquareMNPQ,andof ∆PEF, respectively. Show that:a) S 1 − S 2 =4 p S AEN · S BFQ ; b) S 3 ≥ S 1 ; c) 1 = 1 − 1 .S 3 S 3 S 3Claudiu-Ştefan Popa, IaşiHighschool levelL106. Let I be the centre of the circle inscribed in the arbitrary triangle ABC.The straight lines AI, BI, CI meet for the second time the circles circumscbribedto the triangles BCI, CAI and ABI at the points A 0 , B 0 and respectively C 0 .Ifwedenote by |XY Z| the perimeter of the triangle XY Z,showthatBC|BCA 0 | + CA|CAB 0 | + AB|ABC 0 | =1.Titu Zvonaru, ComăneştiL107. Let M, N be two points situated in the interior of ∆ ABC such theirdistances up to the sides AB, BC, CA are respectively equal to 3, 2, 7 and 9 2 , 5, 5 2 .If the radius of the circumscribed circle to ∆ABC is R =8, determine the distancefrom M to N.Vlad Emanuel, highschool student, SibiuL108. Show that the following inequality holds in any ∆ABC :3 √ 32 − (sin A +sinB +sinC) ≥ 4√ 3sin 2 π − 3A .12Marian Tetiva, BârladL109. The real positive and less than 1 numbers a 1 ,a 2 ,...,a 2n2 −n, n ∈ N \{0, 1}are given. Prove the following inequality (with summation being performed by circularpermutations):Xa 2n−112 n − 1a 2n−12 + a 2n−13 + ...+ a 2n−1


(A problem connected with another problem proposed at OBM 2005).Titu Zvonaru, Comăneşti and Bogdan Ioniţă, BucureştiL111. We are given m distinct natural numbers contained in the set {1, 2,...,n}.Show that it is possible to select a couple of them, with their sum equal to S, suchm(m +1)that 0 ≤ S − ≤ n + √ 2n − m.2Adrian Zahariuc, highschool student, BacăuL112. For n ∈ N, wedenotea(n) the number of ways under which the numbern can be written as a sum of an even number of powers of 2 and let b(n) denotethe number of ways under which n can be written as the sum of an odd number ofpowers of 2. Show that a(n) =b(n), ∀n ≥ 2.Adrian Zahariuc, highschool student, BacăuL113. Determine the real numbers a, b such that the set A = {a n + b n | n ∈ N ∗ }is finite.Gheorghe Iurea, IaşiL114. We consider a parabola and two parallel straight lines that meet theparabola but are not parallel to its axis of symmetry. Using a non-measuring rule,build the tangent to the parabola which is parallel to the given lines.Titu Zvonaru, ComăneştiL115. Determine the polynomials P ∈ R[X] of degree ≥ 2 so that the functionf : R → R, f(x) =p({x})+{p(x)} be periodical (where p is the polynomial functioncorresponding to P ,and{·} denotes the franctional part function).Paul Georgescu and Gabriel Popa, IaşiASOCIAŢIA “RECREAŢII <strong>MATEMATICE</strong>”La data de 14.02.2005 a luat fiinţă ASOCIAŢIA “RECREAŢII MATE-MATICE”, cusediulînIaşi (str. Aurora, nr. 3, sc. D, ap. 6), având ca scop sprijinireaactivităţilor de matematică specifice învăţământului preuniversitar, organizareaşi desfăşurarea de activităţi care să contribuie la dezvoltarea gustului pentrumatematică în rândurile elevilor, profesorilor şi iubitorilor de matematică şi stimulareapreocupărilor şi cercetătorilor originale.Obiectivele majore pentru atingerea scopului propus sunt:1. editarea unei reviste destinată elevilor şi profesorilor — revista "RecreaţiiMatematice";2. fondarea unei biblioteci de matematică elementară—biblioteca "RecreaţiiMatematice";3. alcătuirea unei colecţii de cărţi de matematică elementară, cărţi de referinţăşi aflate la prima apariţie — Colecţia "Recreaţii Matematice".Poate deveni membru al Asociaţiei, printr-o simplă completare a unei cerei tip,orice perosană care aderă laobiectiveleacesteiaşi sprijină realizarealor.173


Pagina rezolvitorilorBRAŞOVLiceul "N. Titulescu". ClasaaX-a.ANTONACHE Alexandru: VIII(61,62,64),IX(61,70), X.67; BARBU Cosmin: VIII(61-64), IX(61,70), X.69; BARDĂ Dan:VIII(61-64), IX(61,70), X.69; BRÂNZEA Alexandru: VIII(61-64), IX.61; BUT-NARIU Anda: VIII(61-64), IX.61; DAVID Dana: VIII(61,64), IX(61,70), X.69; FÜZISandor: VIII(61-64), IX.61; GHICA Alexandra: VIII(61,63,64), IX(61,63), X.67;ILIE Andreea: VIII(61,62,64), IX(61,70), X.67; MUREŞIANU Raul: VIII(61-64),IX.61; OBANCEA Dragoş: VIII.64, IX(68,70), X(67,69); SZÖCS Daniel: VIII(61-64), IX(61,70), X.69; ŞEITAN Marius: VIII(61-64), IX.61; ŞOIM Mihaela: VIII(61-64), IX(61,68,70), X.67; VASILE Cătălin: VIII(61-64), IX.61; VÂRDOL Radu:VIII(61-64), IX.61. Clasa a XI-a. DEMÉNY Ana-Maria: IX(63,65,68,70), X(67,69);MUNTEAN Vlad: IX(61,63,65), X(62,69).CARANSEBEŞLiceul teoretic "Traian Doda". Clasa a IV-a. SZALO Ildiko: P(94-101).Liceul pedagogic "G. D. Loga". Clasa a VIII-a. VLAD Adina Alexandra:VI(61-65), VII(61,64), VIII(61,62), IX.63.CRAIOVAColegiul Naţional "Carol I". ClasaaVI-a. STANCIU Ioan: V(66-70), VI(66-70), VIII.68, G(96,97).HÂRLĂUŞcoala "P. Rareş". Clasa a III-a (înv. BUDACEA Maria). NEICU Mara:P(100,104,105,108,110).Liceul "Ştefan cel Mare". Clasa a VII-a. CURCĂ Ioana: VI(66,67,69,70),VII(68,70); PINTILII Anda: VI(66,67,69,70), VII(68,70).IAŞIŞcoala nr. 3 "Al. Vlahuţă". ClasaaIII-a(inst. MAXIM Gabriela). BACIUIonela-Lavinia: P(94,96,98,100,103-105,108); CELMARE Raluca: P(94,96,98,100,103-105,108); GHEMU Laura: P(94,96,98,100,103-105,108); NEAGU Ramona: P(94,96,98,100,103-105,108); POPOVICI Ionuţ: P(94,96,98,100,103-105,108); RUSU IoanaAndreea: P(94,96,98,100,103-105,108); RUSU Mădălina Andreea: P(94,96,98,100,103-105,108); SAVA Vlad: P(94,96,98,100,103-105,108); VECHIU Mădălina: P(94,96,98,100,103-105,108), ZANCHI Georgiana-Alexandra: P(94,96,98,100,103-105,108).Clasa a III-a (înv. MĂRIUŢĂ Valentina). BÂRLĂDEANU Andrei: P(98,100,103-105,108); CULEA Alina: P(98,100,103-105,108); HADARAG Ana-Maria: P(98,100,103-105,108); NĂSTASE Cosmin: P(98,100,103-105,108); POPA Iulian: P(98,100,103-105,108); PROCA Ancuţa-Ioana: P(98,100,103-105,108); TÂNCU Alexandra:P(98,100,103-105,108).Şcoala nr. 13 "Alexandru cel Bun". Clasa a II-a (inst. COJOCARIU Ana).AGAFIŢEI Elena-Roxana: P(104-107,109); CARAMALĂU Andra: P(104-107,109);CĂLIN Andreea-Claudia: P(104-107,109); COJOCARIU Andreea: P(104-107,109);DUDUMAN Luisa-Ştefania: P(104-107,109); LELEU Alexandrina-Ştefana: P(104-107,109); LUPAŞCU Diana-Maria: P(104-107,109); MANOLACHE Mădălina-Andre-174


ea: P(104-107,109); MIHĂILĂ Narcisa-Lorena: P(104-107,109); PASCU Gabriela:P(104-107,109); PĂDURARU Tiberiu-Ştefan: P(104-107,109); RĂDUCEA Marin-Andrei: P(104-107,109); SAVIN Cristina-Simona: P(104-107,109); ŞTEFAN Bogdan-Vasile: P(104-107,109); ŞTIUBEI Cosmin-Ionuţ: P(104-107,109).Şcoala nr. 26 "G. Coşbuc". Clasa I (inst. RACU Maria). APACHIŢEI Aura-Georgiana: P(94-97,105,110); BURA Emma-Andreea: P(94-97,105,110); FILIP Ingrid-Ştefania:P(94-97,105,110); GRĂDINARIU Georgiana: P(94-97,105,110); HRIS-CU Ovidiu-Constantin: P(94-97,105,110); HUZA Mădălina: P(94-97,105,110); MA-XIM Alexandra-Camelia: P(94-97,105,110); VASILE Bogdan-Andrei: P(94-97,105,110). ClasaaIV-a(înv. BUCATARIU Rica). AELENEI George-Ciprian: P(104,105,108-111); SCUTARU Ionela-Cristina: P(104-112).Colegiul Naţional "Emil Racoviţă". ClasaaII-a(inst. CĂLINESCU Rodica).PETREA Mădălina: P(104-107,110); UNGUREANU George: P(104,105,107,108,110). ClasaaV-a. ENEA Maria: V(66-70); ŢURCANU Andreea Bianca: V(66-70); Clasa a VI-a. TUDORACHE Alexandru Gabriel: V(66-70), VI(66-70).Colegiul Naţional "C. Negruzzi". ClasaaV-a.PĂVĂLOI Alexandru: V(61-65).Clasa a VII-a. TIBA Marius: VII(66-68,70), G(96,97,100,103).PETROŞANILiceul de in<strong>format</strong>ică. ClasaaX-a. PĂLIŢĂ Florian: VIII(66,68,69), IX.68,X.68, G(97,102,103).SIBIUColegiul Naţional "Gh. Lazăr". Clasa a XI-a. VLAD Emanuel: X(61,63-65),XI(62,63), L(86,87,89,90,92).SUCEAVAŞcoala generală nr. 3. Clasa a II-a (înv. TABARCEA Silvestru). FECHETŞtefan: P(94-97,99).Premii acordate rezolvitorilorASOCIAŢIA "RECREAŢII <strong>MATEMATICE</strong>" în colaborare cu redacţiarevistei RECREAŢII <strong>MATEMATICE</strong> acordă câteodiplomă şi un premiu încărţi pentru trei apariţii la rubrica Pagina rezolvitorilor următorilor elevi:Liceul "N. Titulescu", BraşovBARDĂ Dan(cl. a X-a): 1/2005(5pb), 2/2005(5pb), 2/2006(7pb);BUTNARIU Anda (cl. a X-a): 1/2005(6pb), 2/2005(5pb), 2/2006(5pb);SZÖCS Daniel (cl. a X-a): 1/2005(6pb), 2/2005(5pb), 2/2006(7pb);Liceul "Ştefan cel Mare", HârlăuCIUCĂ Ioana(cl. a VII-a): 2/2005(5pb), 1/2006(5pb), 2/2006(6pb);PINTILII Anda (cl. a VII-a): 2/2005(5pb), 1/2006(5pb), 2/2006(6pb);175


Colegiul Naţional "Gh. Lazăr", SibiuVLAD Emanuel (cl. a XI-a): 2/2005(5pb), 1/2006(7pb), 2/2006(12pb);Şcoala nr. 3 "Al. Vlahuţă", IaşiRUSU Mădălina-Andreea (cl. a III-a): 2/2005(6pb), 1/2006(5pb), 2/2006(8pb);SAVA Vlad (cl. a III-a): 2/2005(6pb), 1/2006(5pb), 2/2006(8pb);Şcoala nr. 13 "Alexandru cel Bun", IaşiAGAFIŢEI Elena-Roxana (cl. a III-a): 2/2005(5pb), 1/2006(5pb), 2/2006(5pb);CARAMALĂU Andra (cl. a III-a): 2/2005(5pb), 1/2006(5pb), 2/2006(5pb);CĂLIN Andreea-Claudia (cl. a III-a): 2/2005(5pb), 1/2006(5pb), 2/2006(5pb);COJOCARIU Andreea (cl. a III-a): 2/2005(5pb), 1/2006(5pb), 2/2006(5pb);DUDUMAN Luisa-Ştefania (cl. a III-a): 2/2005(5pb), 1/2006(5pb), 2/2006(5pb);LELEU Alexandrina-Ştefana (cl. a III-a): 2/2005(5pb), 1/2006(5pb), 2/2006(5pb);LUPAŞCU Diana-Maria (cl. a III-a): 2/2005(5pb), 1/2006(5pb), 2/2006(5pb);MANOLACHE Mădălina-Andreea (cl. a III-a): 2/2005(5pb), 1/2006(5pb),2/2006(5pb);PASCU Gabriela (cl. a III-a): 2/2005(5pb), 1/2006(5pb), 2/2006(5pb);PĂDURARU Tiberiu-Ştefan (cl. a III-a): 2/2005(5pb), 1/2006(5pb), 2/2006(5pb);RĂDUCEA Marin-Andrei (cl. a III-a): 2/2005(5pb), 1/2006(5pb), 2/2006(5pb);SAVIN Cristina-Simona (cl. a III-a): 2/2005(5pb), 1/2006(5pb), 2/2006(5pb);ŞTEFAN Bogdan-Vasile (cl. a III-a): 2/2005(5pb), 1/2006(5pb), 2/2006(5pb);ŞTIUBEI Cosmin-Ionuţ (cl. a III-a): 2/2005(5pb), 1/2006(5pb), 2/2006(5pb);ASOCIAŢIA “RECREAŢII <strong>MATEMATICE</strong>”Lista membrilor asociaţiei - continuarea listei din nr. 1/2006.Numele şi prenumele Locul de muncă Data înscrierii40 Calistru Cătălin Univ. Tehn. "Gh. Asachi", Iaşi 22.07.200541 Nechita Vasile Col. Naţ. "C. Negruzzi", Iaşi 08.08.200542 Kern Edgar S. C. SELGROS, Braşov 11.08.200543 Bătineţu-Giurgiu Maria Acad. Tehnică Militară, Buc. 11.08.200544 Bătineţu-Giurgiu Dumitru Col. Naţ. "Matei Basarab", Buc. 11.08.200545 Picioroagă Gabriel Univ. of Southern, Denmark 27.08.200546 Anişca Răzvan Lakehead University, Canada 08.09.200547 Cheţan Corneliu CITRIX, INC., USA 02.10.200548 Maftei V. Ioan Col. Naţ. "Sf. Sava", Buc. 09.10.200549 Gheorhiţă V. Vitalii Univ. Tehn. "Gh. Asachi", Iaşi 20.01.200650 Ionescu Marius Vasile Dartmounth College, USA 25.03.200651 Ciacoi Bogdan Andrei Lic. T. "Ana Ipătescu", Gherla, (elev) 27.03.2006176


<strong>Revista</strong> semestrială RECREAŢII <strong>MATEMATICE</strong> este editată deASOCIAŢIA “RECREAŢII <strong>MATEMATICE</strong>”. Apare la datele de 1 martie şi1 septembrie şi se adresează elevilor, profesorilor, studenţilor şi tuturor celorpasionaţi de matematica elementară.În atenţia tuturor colaboratorilorMaterialele trimise redacţiei spre publicare (note şi articole, chestiuni demetodică, probleme propuse etc.) trebuie prezentate îngrijit, clar şi concis; eletrebuie să prezinte interes pentru un cerc cât mai larg de cititori. Se recomandă catextele să nu depăşească patru pagini. Evident, ele trebuie să fie originale şi sănu fi apărut sau să fi fost trimise spre publicare altor reviste. Rugăm ca materialeletehnoredactate să fie însoţite de fişierele lor.Problemele destinate rubricilor: Probleme propuse şi Probleme pentrupregătirea concursurilor vor fi redactate pe foi separate cu enunţ şi demonstraţie/rezolvare(câte una pe fiecare foaie) şi vor fi însoţite de numele autorului, şcoalaşi localitatea unde lucrează/învaţă.Redacţia va decide asupra oportunităţii publicării materialelor primite.În atenţia elevilorNumele elevilor ce vor trimite redacţiei soluţii corecte la problemele dinrubricile de Probleme propuse şi Probleme pentru pregatirea concursurilorvor fi menţionate în Pagina rezolvitorilor. Se va ţine seama de regulile:1. Pot trimite soluţii la minimum cinci probleme propuse în numărulprezent şi cel anterior al revistei; pe o foaie va fi redactată soluţia unei singureprobleme.2. Elevii din clasele VI-XII au dreptul să trimită soluţii la problemelepropuse pentru clasa lor, pentru orice clasă mai mare, din două clase mai mici şiimediat anterioare. Elevii din clasa a V-a pot trimite soluţii la problemele propusepentru clasele a IV-a, a V-a şi orice clasă mai mare, iar elevii claselor I-IV pottrimite soluţii la problemele propuse pentru oricare din clasele primare şi orice clasămai mare. Orice elev poate trimite soluţii la problemele de concurs (tip G şi L).3. Vor fi menţionate următoarele date personale: numele şi prenumele,clasa, şcoala şi localitatea.4. Plicul cu probleme rezolvate se va trimite prin poştă (sau va fi adusdirect) la adresa Redacţiei:Prof. dr. Temistocle BîrsanStr. Aurora, nr. 3, sc. D, ap. 6,700 474, IaşiJud. IAŞIE-mail: tbirsan@math.tuiasi.ro


CUPRINSMendel Haimovici şi Şcoala matematică din Iaşi ........................................................... 89Florica T. Câmpan ................................................................................................................ 91Eclipsele de Soare.................................................................................................................. 93Olimpiada 57 .......................................................................................................................... 96ARTICOLE ŞI NOTEA. REISNER – Pseudoinversă şi inversă generalizată ale unei aplicaţii liniare............... 97C. ŢIGĂERU – O soluţie parţială a unei probleme a lui N. Papacu ............................ 102M. APETRII şi C.-C. BUDEANU– Criterii de congruenţă a triunghiurilor ...................... 107D. M. BĂTINEŢU-GIURGIU – Asupra problemei 3639din Gazeta Matematică, v. XXXIII (1927-1928)............................ 110NOTA ELEVULUIA. ZAHARIUC – O problemă despre suma cifrelor unui număr naturalîn baze de numeraţie oarecare......................................................... 113Al. NEGRESCU – Generalizări ale unor inegalităţi din RecMat...................................... 117CHESTIUNI METODICEM. TETIVA – Cea mai bună inegaliate de acest tip … .............................................. 120CORESPONDENŢESocietatea de Ştiinţe Matematice din R. Moldova ............................................................ 124CONCURSURI ŞI EXAMENEConcursul de Matematică "Al. Myller", Ediţia a IV-a, Iaşi............................................. 126Concursul de matematică "Florica T. Câmpan" ...................................................................... 130PROBLEME ŞI SOLUŢIISoluţiile problemelor propuse în nr. 2/2005........................................................................ 134Soluţiile problemelor pentru pregătirea concursurilor din nr. 2/2005 ............................... 152Probleme propuse................................................................................................................... 163Probleme pentru pregătirea concursurilor ............................................................................ 169Training problems for mathematical contests ..................................................................... 171Pagina rezolvitorilor ............................................................................................................ 174ASOCIAŢIA “RECREAŢII <strong>MATEMATICE</strong>”5 lei

Hooray! Your file is uploaded and ready to be published.

Saved successfully!

Ooh no, something went wrong!